Real Analysis (1) (All Lectures)
Real Analysis (1) (All Lectures)
Topics Page
Notations 2.1.1:
(1) ℝ: The set of all real numbers ⇒ ℝ = (−∞, ∞) = {𝑥: −∞ < 𝑥 < ∞}
(2) ℚ: The set of rational numbers.
(3) ℕ: The set of all natural numbers, that is ℕ = {1,2,3, … }
The Trichotomy Property 2.1.2 ()الخاصية الثالثية على األعداد الحقيقية:
If 𝑎 ∈ ℝ, then 𝑎 > 0, 𝑎 = 0, or 𝑎 < 0.
Theorem 2.1.3:
(1) If 𝑎 ∈ ℝ such that 0 ≤ 𝑎 ≤ 𝜀, ∀𝜀 > 0, then 𝑎 = 0.
(2) If 𝑎 ∈ ℝ such that 0 ≤ 𝑎 < 𝜀, ∀𝜀 > 0, then 𝑎 = 0.
(3) If 𝑎 ∈ ℝ such that −𝜀 ≤ 𝑎 ≤ 𝜀, ∀𝜀 > 0, then 𝑎 = 0.
(4) If 𝑎 ∈ ℝ such that −𝜀 < 𝑎 < 𝜀, ∀𝜀 > 0, then 𝑎 = 0.
Proof: We prove part (1). The proofs of the other parts (2)&(3) are similar.
Suppose that 0 ≤ 𝑎 ≤ 𝜀, ∀𝜀 > 0. The proof is by contradiction: Assume that 𝑎 ≠ 0.
Since 0 ≤ 𝑎 ≤ 𝜀, ∀𝜀 > 0 ⇒ 𝑎 ≥ 0……① and 𝑎 ≤ 𝜀, ∀𝜀 > 0……..②
𝑎 𝑎
Since 𝑎 ≠ 0, then ① implies that 𝑎 > 0. So, in ② take 𝜀 = 2 we have 𝑎 ≤ 2
1
⇒ 1 ≤ 2 which is impossible ⇒ We have a contradiction ⇒ Our assumption is false
⇒ 𝑎 = 0.
2
Example 2.1.4:
1
(1) If 𝑎 ∈ ℝ such that 0 ≤ 𝑎 ≤ 𝑛 , ∀𝑛 ∈ ℕ, then 𝑎 = 0.
1
(2) If 𝑎 ∈ ℝ such that 0 ≤ 𝑎 < 𝑛 , ∀𝑛 ∈ ℕ, then 𝑎 = 0.
1 1
(3) If 𝑎 ∈ ℝ such that − 𝑛 < 𝑎 < 𝑛 , ∀𝑛 ∈ ℕ, then 𝑎 = 0.
Solution: We prove part (1). The proofs of parts (2)&(3) are exercises.
Let 𝜀 > 0 be given.
1 1
Since 𝜀 > 0 and lim = 0, then ∃𝑛0 ∈ ℕ such that 𝑛 < 𝜀
𝑛→∞ 𝑛 0
1 1
But 0 ≤ 𝑎 ≤ 𝑛 , ∀𝑛 ∈ ℕ ⇒ 0 ≤ 𝑎 ≤ 𝑛 , since 𝑛0 ∈ ℕ
0
1
But 𝑛 < 𝜀 ⇒ 0 ≤ 𝑎 < 𝜀 and since 𝜀 is arbitrary we have 0 ≤ 𝑎 < 𝜀, ∀𝜀 > 0
0
Theorem 3 ⇒ 𝑎 = 0.
Corollary 2.1.5:
(1) If 𝑎, 𝑏 ∈ ℝ such that 𝑎 ≤ 𝑏 + 𝜀, ∀𝜀 > 0, then 𝑎 ≤ 𝑏.
(2) If 𝑎, 𝑏 ∈ ℝ such that 𝑎 < 𝑏 + 𝜀, ∀𝜀 > 0, then 𝑎 ≤ 𝑏.
1
(3) If 𝑎, 𝑏 ∈ ℝ such that 𝑎 ≤ 𝑏 + 𝑛 , ∀𝑛 ∈ ℕ, then 𝑎 ≤ 𝑏.
1
(4) If 𝑎, 𝑏 ∈ ℝ such that 𝑎 < 𝑏 + 𝑛 , ∀𝑛 ∈ ℕ, then 𝑎 ≤ 𝑏.
Proof: We prove part (1). The proofs of the other parts are similar.
Suppose that 𝑎 ≤ 𝑏 + 𝜀, ∀𝜀 > 0. The proof is by contradiction:
Assume that 𝑎 > 𝑏. (we must have a contradiction)
𝑎 > 𝑏 ⇒ 𝑎 − 𝑏 > 0 ⇒ 𝑎 − 𝑏 ≥ 0……..①
𝑎 ≤ 𝑏 + 𝜀, ∀𝜀 > 0 ⇒ 𝑎 − 𝑏 ≤ 𝜀, ∀𝜀 > 0……..②
①&② ⇒ 0≤ 𝑎 − 𝑏 ≤ 𝜀, ∀𝜀 > 0
Theorem 3 ⇒ 𝑎 − 𝑏 = 0 ⇒ 𝑎 = 𝑏 which is a contradiction ⇒ Our assumption is false
⇒ 𝑎 ≤ 𝑏.
Definition 2.1.6:
(1) The geometric mean of two positive real numbers 𝑎 and 𝑏 is √𝑎𝑏.
𝑎+𝑏
(2) The arithmetic mean of two real numbers 𝑎 and 𝑏 is .
2
3
Proof: (𝑎 − 𝑏)2 ≥ 0 ⇔ ⏟
𝑎2 − 2𝑎𝑏 + 𝑏 2 ≥ 0 ⇔ 𝑎2 + 2𝑎𝑏 + 𝑏 2 ≥ 4𝑎𝑏 ⇔ ⏟
(𝑎 + 𝑏)2 ≥ 4𝑎𝑏
4𝑎𝑏 اجمع للطرفين نأخذ الجذر التربيعي للطرفين
𝑎+𝑏
⇔ √(𝑎 + 𝑏)2 ≥ √4𝑎𝑏 ⇔ 𝑎 + 𝑏 ≥ 2√𝑎𝑏 ⇔ √𝑎𝑏 ≤ 2
𝑎+𝑏
For Equality: √𝑎𝑏 = iff (𝑎 − 𝑏)2 = 0 iff 𝑎 = 𝑏.
2
Definition 2.2.1: The absolute value of a real number 𝑎 ∈ ℝ, written as |𝑎|, is defined by
𝑎 , 𝑎≥0
|𝑎| = { }
−𝑎 , 𝑎 < 0
𝑥 , 𝑥≥0
As a function: Let 𝑓(𝑥) = |𝑥|, 𝑥 ∈ ℝ ⇒ 𝑓(𝑥) = { }
−𝑥 , 𝑥 < 0
𝑥 2 − 2𝑥 − 3 , 𝑥 ≤ −1 or 𝑥 ≥ 3
|𝑥 2 − 2𝑥 − 3| = { }
3 + 2𝑥 − 𝑥 2 , −1 < 𝑥 < 3
Theorem 2.2.3: Let 𝑎, 𝑏 ∈ ℝ. Then
𝑎 |𝑎|
(1) |𝑎𝑏| = |𝑎| |𝑏| (2) |𝑏| = |𝑏|, where 𝑏 ≠ 0
Proof: We prove part (1). The proof of part (2) is similar.
① ② ③ ④
Proof: We prove parts (1) and (3). The proofs of parts (2) and (4) are similar.
(1) Since 𝑥 ∈ ℝ ⇒ We have two cases: 𝑥 ≥ 0 or 𝑥 < 0
Case 1: Let 𝑥 ≥ 0 ⇒ |𝑥| = 𝑥. Now,
|𝑥| ≤ 𝑎 ⇔ 𝑥 ≤ 𝑎 ⇔ 𝑥 ⏟≤ 𝑎 and ⏟ 𝑥 ≥ −𝑎 ⇔ −𝑎 ≤ 𝑥 ≤ 𝑎
proposition Tautology
Case 2: Let 𝑥 < 0 ⇒ |𝑥| = −𝑥. Now,
|𝑥| ≤ 𝑎 ⇔ −𝑥 ≤ 𝑎 ⇔ −𝑎 ≤ 𝑥 ⇔ ⏟ −𝑎 ≤ 𝑥 and 𝑥
⏟≤ 𝑎 ⇔ −𝑎 ≤ 𝑥 ≤ 𝑎
proposition Tautology
(3) Since 𝑥 ∈ ℝ ⇒ We have two cases: 𝑥 ≥ 0 or 𝑥 < 0
Case 1: Let 𝑥 ≥ 0 ⇒ |𝑥| = 𝑥. Now: |𝑥| ≥ 𝑎 ⇔ 𝑥 ≥ 𝑎 ⇔ 𝑥
⏟≥ 𝑎 or ⏟
𝑥 ≤ −𝑎
proposition cotradiction
Case 2: Let 𝑥 < 0 ⇒ |𝑥| = −𝑥. Now,
|𝑥| ≥ 𝑎 ⇔ −𝑥 ≥ 𝑎 ⇔ 𝑥 ≤ −𝑎 ⇔ 𝑥 ⏟≥ 𝑎 or ⏟
𝑥 ≤ −𝑎
contradiction proposition
Example 2.2.6: Solve the following equations and inequalities ()متباينات
(1) |𝑥 + 1| + |𝑥 − 2| = 7 (2) |𝑥| + |𝑥 + 1| < 2
(3) |2𝑥 − 1| < 𝑥 + 1 (4) |𝑥 − 1| ≤ |𝑥|
Solution: We solve parts (1) and (2). The solution of parts (3) and (4) are exercise.
6
(1)
(2)
7
) في المثلث يكون مجموع طولي أي ضلعين أكبر من أو يساوي طول الضلع الثالثThe triangle inequality( المتباينة المثلثية
Example 2.2.15: Let 0 < 𝑎 < 1 and 0 < 𝜀 < min{𝑎, 1 − 𝑎}. Show that 𝑉𝜀 (𝑎) ⊆ (0,1)
Proof: Let 𝑥 ∈ 𝑉𝜀 (𝑎) ⇒ |𝑥 − 𝑎| < 𝜀 ⇒ |𝑥 − 𝑎| < min{𝑎, 1 − 𝑎}. So,
|𝑥 − 𝑎| < 𝑎 ⇒ −𝑎 < 𝑥 − 𝑎 < 𝑎 ⇒ 0 < 𝑥 < 2𝑎 ⇒𝑥 > 0…….①
|𝑥 − 𝑎| < 1 − 𝑎 ⇒ −(1 − 𝑎) < 𝑥 − 𝑎 < 1 − 𝑎 ⇒ −1 + 2𝑎 < 𝑥 < 1 ⇒ 𝑥 < 1…….②
①&② ⇒ 0 < 𝑥 < 1 ⇒ 𝑥 ∈ (0,1). So, 𝑉𝜀 (𝑎) ⊆ (0,1)
Example 2.2.16: Let 𝑎, 𝑏 ∈ ℝ and 𝜀 > 0. Show if 𝑥 ∈ 𝑉𝜀 (𝑎) and 𝑦 ∈ 𝑉𝜀 (𝑏), then 𝑥 + 𝑦 ∈ 𝑉2𝜀 (𝑎 + 𝑏)
Proof: 𝑥 ∈ 𝑉𝜀 (𝑎) ⇒ |𝑥 − 𝑎| < 𝜀 and 𝑦 ∈ 𝑉𝜀 (𝑎) ⇒ |𝑦 − 𝑏| < 𝜀
|𝑥 + 𝑦 − (𝑎 + 𝑏)| = ⏟
|(𝑥 − 𝑎) + (𝑦 − 𝑏)| ≤ |𝑥 − 𝑎| + |𝑦 − 𝑏| < 𝜀 + 𝜀
by the triangle inequality
Definition 2.3.1:
(1) A nonempty set 𝑆 is called bounded (written as bdd) above if ∃𝑢 ∈ ℝ such that
𝑥 ≤ 𝑢, ∀𝑥 ∈ 𝑆, that is 𝑆 ⊆ (−∞, 𝑢].
The real number 𝑢, if exists, is called an upper bound (written as UB)
[𝑢 is not UB of a set 𝑆] iff [∃𝑥 ∈ 𝑆 such that 𝑥 > 𝑢]
(2) A real number 𝑢∗ is called a supremum (or a least UB) of a set 𝑆 (written sup 𝑆 = 𝑢∗ ) if
the following two conditions hold:
𝐶1 : 𝑢∗ is an UB of 𝑆.
𝐶2 : if 𝑢 is an UB of 𝑆, then 𝑢∗ ≤ 𝑢.
(4) A real number 𝑣 ∗ is called an infimum (or a greatest LB) of a set 𝑆 (written inf 𝑆 = 𝑣 ∗ ) if
the following two conditions hold:
𝐶1 : 𝑣 ∗ is a LB of 𝑆.
𝐶2 : if 𝑣 is a LB of 𝑆, then 𝑣 ∗ ≥ 𝑣.
Example 2.3.2:
1 1 1 1
(1) Consider the set 𝑆 = {𝑛 : 𝑛 ∈ ℕ} ⇒ 𝑆 = {1, 2 , 3 , 4 , … } (sketch the graph of 𝑆 on the real
line if possible).
sup 𝑆 = 1 and the set of all UB’s of 𝑆 is [1, ∞) ⇒ 𝑆 is bdd above
inf 𝑆 = 0 and the set of all LB’s of 𝑆 is (−∞, 0] ⇒ 𝑆 is bdd below
⇒ 𝑆 is a bdd set.
11
(2) Consider the set (−1, ∞). (sketch the graph of 𝑆 on the real line if possible).
sup (−1, ∞) = ∞ ⇒ sup (−1, ∞) does not exist ⇒ The interval (−1, ∞) has no UB.
⇒ The interval (−1, ∞) is not bdd above
inf (−1, ∞) = −1 ⇒ The set of all LB’s of (−1, ∞) is the set (−∞, −1]
⇒ (−1, ∞) is bdd below ⇒ (−1, ∞) is not a bdd set (since it is not bdd above)
1−√5 1+√5
⇒ 𝑆 = {𝑥 ∈ ℝ: 𝑥 + 1 > 𝑥 2 } = ( , )
2 2
1+√5 1−√5
⇒ sup 𝑆 = and inf 𝑆 =
2 2
and
inf 𝑓(𝑥) = inf{𝑓(𝑥): 𝑥 ∈ 𝐷} = inf (Range 𝑓)
𝑥∈𝐷
Example 2.3.5:
(1) 𝑓(𝑥) = −𝑥 2 + 𝑥 + 1. Find sup 𝑓(𝑥), inf 𝑓(𝑥), sup 𝑓(𝑥) and inf 𝑓(𝑥).
𝑥∈ℝ 𝑥∈ℝ 𝑥∈[1,2) 𝑥∈[1,2)
3 2
(2) Let 𝑓(𝑥) = 𝑥 + 3𝑥 − 2. Find sup 𝑓(𝑥) and inf 𝑓(𝑥).
𝑥∈[−2,0.5) 𝑥∈[−2,0.5)
Solution:
(1) To find Range(𝑓) we give a sketch the graph of 𝑓(𝑥)
5
When Domain (𝑓) = ℝ ⇒ Range(𝑓) = (−∞, 4]
5 5 5
⇒ sup 𝑓(𝑥) = sup (−∞, 4] = 4 and inf 𝑓(𝑥) = inf (−∞, 4] = −∞
𝑥∈ℝ 𝑥∈ℝ
Suppose that 𝐶2′′ holds ⇒ ∀𝜀 > 0, ∃𝑥𝜀 ∈ 𝑆 such that 𝑢∗ − 𝜀 < 𝑥𝜀 …….③
Let 𝑢 < 𝑢∗ . Take 𝜀 = 𝑢∗ − 𝑢 ⇒ 𝜀 > 0 so by ② ∃𝑥𝜀 ∈ 𝑆 such that 𝑢∗ − 𝜀 < 𝑥𝜀
⇒ 𝑢∗ − (𝑢∗ − 𝑢) < 𝑥𝜀 ⇒ 𝑢 < 𝑥𝜀 ⇒ 𝑢 is not an UB of 𝑆 ⇒ 𝐶2′ holds. So, 𝐶2′′ ⇒ 𝐶2′ ……④
By ③&④ we have 𝐶2′ ⇔ 𝐶2′′ .
Theorem 2.3.9:
(1) If 𝑢0 is an UB of a set 𝑆 and 𝑢0 ∈ 𝑆, then sup 𝑆 = 𝑢0 .
(2) If 𝑣0 is a LB of a set 𝑆 and 𝑣0 ∈ 𝑆, then inf 𝑆 = 𝑣0 .
Proof:
(1) We will show that 𝑢0 satisfies 𝐶1 and 𝐶2 in the definition of supremum.
In the definition taking 𝑢∗ = 𝑢0 we have:
𝑢0 is an UB of a set 𝑆 ⇒ 𝐶1 holds
Let 𝑢 be an UB of 𝑆. Want to show that 𝑢 ≥ 𝑢0 :
Since 𝑢 is an UB of 𝑆 ⇒ 𝑢 ≥ 𝑥, ∀𝑥 ∈ 𝑆……①
But 𝑢0 ∈ 𝑆. So, in ② take 𝑥 = 𝑢0 ⇒ 𝑢 ≥ 𝑢0 ⇒ 𝐶2 holds
14
Remark 2.3.12:
2
(1) ℚ: The set of rational numbers. For example: 0, −7,9, 5 , √16 are rational numbers.
1
(2) ℚ𝑐 : The set of irrational numbers. For example: √2, √3, 𝜋, are irrational numbers
√2
(3) ℤ: The set of integers ⇒ ℤ ⊂ ℚ
(4) ℝ = ℚ ∪ ℚ𝑐 and ℚ ∩ ℚ𝑐 = ∅ (the empty set)
(5) 𝑎 ∈ ℝ ⇔ 𝑎 ∈ ℚ or 𝑎 ∈ ℚ𝑐
(6) If 𝑥 ∈ ℝ, then ∃𝑚 ∈ ℤ such that 𝑚 − 1 ≤ 𝑥 < 𝑚.
The Density Theorem 2.3.13: Between any two real numbers there is a rational number
If 𝑥, 𝑦 ∈ ℝ such that 𝑥 < 𝑦, then ∃𝑟 ∈ ℚ such that 𝑥 < 𝑟 < 𝑦.
(any interval has a rational number inside it)
1
Proof: 𝑥 < 𝑦 ⇒ 𝑦 − 𝑥 > 0 ⇒ 𝑦−𝑥 > 0
1
The Archimedean Property 2.3.11 ⇒ ∃𝑛 ∈ ℕ such that 𝑛 > 𝑦−𝑥 ⇒ 𝑛𝑦 − 𝑛𝑥 > 1
15
⇒ 𝑛𝑥 + 1 < 𝑛𝑦..…..①
Since 𝑛𝑥 ∈ ℝ ⇒ ∃𝑚 ∈ ℤ such that 𝑚 − 1 ≤ 𝑛𝑥 < 𝑚 ⇒ 𝑛𝑥 < 𝑚………..②
②: ⇒ 𝑚 − 1 ≤ 𝑛𝑥 ⇒ 𝑚 ≤ 𝑛𝑥 + 1. So, by ① we have 𝑚 < 𝑛𝑦……….③
𝑚
②&③ ⇒ 𝑛𝑥 < 𝑚 < 𝑛𝑦 ⇒ 𝑥 < < 𝑦……④
𝑛
𝑚
So, take 𝑟 = and since 𝑚, 𝑛 ∈ ℤ with 𝑛 ≠ 0 we have 𝑟 ∈ ℚ with 𝑥 < 𝑟 < 𝑦 (by ④)
𝑛
Corollary 2.3.14: Between any two real numbers there is an irrational number
If 𝑥, 𝑦 ∈ ℝ such that 𝑥 < 𝑦, then ∃𝑟 ′ ∈ ℚ𝑐 such that 𝑥 < 𝑟 ′ < 𝑦.
(any interval has an irrational number inside it)
𝑥 𝑦
Proof: 𝑥 < 𝑦 ⇒ <
√2 √2
𝑥 𝑦
Applying the density Theorem on the interval ( , ): ∃𝑟 ∈ ℚ such that
√2 √2
𝑥 𝑦
<𝑟< ⇒ 𝑥 < 𝑟√2 < 𝑦. Let 𝑟 ′ = 𝑟√2 ⇒ 𝑥 < 𝑟 ′ < 𝑦
√2 √2
Recall that:
(5) sup 𝑆 = 𝑢∗ if the following two conditions hold:
𝐶1 : 𝑢∗ is an UB of 𝑆.
𝐶2 : if 𝑢 is an UB of 𝑆, then 𝑢∗ ≤ 𝑢.
(6) inf 𝑆 = 𝑣 ∗ if the following two conditions hold:
𝐶1 : 𝑣 ∗ is a LB of 𝑆.
𝐶2 : if 𝑣 is a LB of 𝑆, then 𝑣 ∗ ≥ 𝑣.
Remark:
: يكون البرهان باختيار طرف من أطراف العالقة ثم نقوم بتطبيق الخطوات الخمس التاليةinf 𝑆 أوsup 𝑆 إلثبات عالقة ما فيها
① ② ③ ④ ⑤
⏞𝐶1 Definition ⇒ ⏞
⇒ ⏞ ⏞ ⇒ خطوة اإلنتقال
Definition ⇒ ⏞𝐶2
Very Important
Theorem 2.4.3: Let 𝐴 be a set of real numbers. Then
(1) sup(−𝐴) = − inf 𝐴, that is inf 𝐴 = − sup(−𝐴).
(2) inf (−𝐴) = − sup 𝐴, that is sup 𝐴 = − inf(−𝐴).
(3)
(4)
Proof:
(1) Let sup(−𝐴) = 𝑢∗ and inf 𝐴 = 𝑣 ∗ .
Want to show that 𝑢∗ = −𝑣 ∗ by showing that 𝑢∗ ≥ −𝑣 ∗ and 𝑢∗ ≤ −𝑣 ∗ :
Step 1: sup(−𝐴) = 𝑢∗ :
𝐶1 on −𝐴: 𝑢∗ is an UB of −𝐴 ⇒ −𝑎 ≤ 𝑢∗ , ∀𝑎 ∈ 𝐴 (by definition)
⇒ 𝑎 ≥ −𝑢∗ , ∀𝑎 ∈ 𝐴
⇒ −𝑢∗ is a LB of 𝐴 (by definition)
⇒ 𝑣 ∗ ≥ −𝑢∗ (by 𝐶2 on 𝐴)
⇒ 𝑢∗ ≥ −𝑣 ∗ ……①
Step 2: inf 𝐴 = 𝑣 ∗ :
𝐶1 on 𝐴: 𝑣 ∗ is a LB of 𝐴 ⇒ 𝑎 ≥ 𝑣 ∗ , ∀𝑎 ∈ 𝐴 (by definition)
⇒ −𝑎 ≤ −𝑣 ∗ , ∀𝑎 ∈ 𝐴
⇒ 𝑥 ≤ −𝑣 ∗ , ∀𝑥 ∈ −𝐴
⇒ −𝑣 ∗ is an UB of −𝐴 (by definition)
⇒ −𝑣 ∗ ≥ 𝑢∗ (by 𝐶2 on −𝐴)
⇒ −𝑣 ∗ ≥ 𝑢∗ ……②
①&②: ⇒ 𝑢∗ = −𝑣 ∗ ⇒ sup(−𝐴) = − inf 𝐴
(2) Let 𝐵 = −𝐴 ⇒ 𝐴 = −𝐵 ⇒ inf(−𝐴) = inf 𝐵 = − sup(−𝐵) = − sup 𝐴 (by part (1))
17
Definition 2.4.4: Let 𝐴 be a set of real numbers and let 𝑏 ∈ ℝ. The set 𝑏𝐴 is defined by
𝑏𝐴 = {𝑏𝑎: 𝑎 ∈ 𝐴}
Example 2.4.5:
(5) If 𝐴 = {1,2,4}, then 0.1𝐴 = {0.1,0.2,0.4)}
(6) 2ℤ = 2{0, ±1, ±2, ±3, … } = {0, ±2, ±4, ±6, … } = The set of even integers.
Very Important
Proof:
(1) Let 𝑏 > 0, sup(𝑏𝐴) = 𝑢∗ and sup 𝐴 = 𝑤 ∗ .
Want to show that 𝑢∗ = 𝑏𝑤 ∗ by showing that 𝑢∗ ≥ 𝑏𝑤 ∗ and 𝑢∗ ≤ 𝑏𝑤 ∗ :
Step 1: sup(𝑏𝐴) = 𝑢∗ :
𝐶1 on 𝑏𝐴: 𝑢∗ is an UB of 𝑏𝐴 ⇒ 𝑏𝑎 ≤ 𝑢∗ , ∀𝑎 ∈ 𝐴 (by definition)
𝑢∗
⇒𝑎≤ , ∀𝑎 ∈ 𝐴 (since 𝑏 > 0)
𝑏
𝑢∗
⇒ is an UB of 𝐴 (by definition)
𝑏
𝑢∗
⇒ 𝑤∗ ≤ (by 𝐶2 on 𝐴)
𝑏
⇒ 𝑢∗ ≥ 𝑏𝑤 ∗ ……①
Step 2: sup 𝐴 = 𝑤 ∗:
𝐶1 on 𝐴: 𝑤 ∗ is an UB of 𝐴 ⇒ 𝑎 ≤ 𝑤 ∗ , ∀𝑎 ∈ 𝐴 (by definition)
⇒ 𝑏𝑎 ≤ 𝑏𝑤 ∗ , ∀𝑎 ∈ 𝐴 (since 𝑏 > 0)
⇒ 𝑏𝑤 ∗ is an UB of 𝑏𝐴 (by definition)
⇒ 𝑢∗ ≤ 𝑏𝑤 ∗ (by 𝐶2 on 𝐴)
⇒ 𝑢∗ ≤ 𝑏𝑤 ∗ ……②
Definition 2.4.7: Let 𝐴 be a set of real numbers and let 𝑏 ∈ ℝ. Define the set 𝐴 + 𝑏 by
𝐴 + 𝑏 = {𝑎 + 𝑏: 𝑎 ∈ 𝐴}
Example 2.4.8:
(7) If 𝐴 = {1,2,4}, then 𝐴 + 0.5 = {1 + 0.5,2 + 0.5,4 + 0.5} = {1.5,2.5,4.5}
(8) 𝐴 − 1 = 𝐴 + (−1) = {1 − 1,2 − 1,4 − 1} = {0,1,3}
Theorem 2.4.9: Let 𝐴 be a set of real numbers and let 𝑏 ∈ ℝ. Then
(1) sup(𝐴 + 𝑏) = (sup 𝐴) + 𝑏
(2) inf(𝐴 + 𝑏) = (inf 𝐴) + 𝑏
Proof:
(1) Let sup(𝐴 + 𝑏) = 𝑢∗ and sup 𝐴 = 𝑤 ∗
Want to show that 𝑢∗ = 𝑤 ∗ + 𝑏 by showing: 𝑢∗ ≥ 𝑤 ∗ + 𝑏 and 𝑢∗ ≤ 𝑤 ∗ + 𝑏:
Step 1: sup(𝐴 + 𝑏) = 𝑢∗ :
𝐶1 on 𝐴 + 𝑏: 𝑢∗ is an UB of 𝐴 + 𝑏 ⇒ 𝑎 + 𝑏 ≤ 𝑢∗ , ∀𝑎 ∈ 𝐴 (by definition)
⇒ 𝑎 ≤ 𝑢∗ − 𝑏, ∀𝑎 ∈ 𝐴
⇒ 𝑢∗ − 𝑏 is an UB of 𝐴 (by definition)
⇒ 𝑤 ∗ ≤ 𝑢∗ − 𝑏 (by 𝐶2 on 𝐴)
⇒ 𝑤 ∗ + 𝑏 ≤ 𝑢∗ ……①
Step 2: sup 𝐴 = 𝑤 ∗:
𝐶1 on 𝐴: 𝑤 ∗ is an UB of 𝐴 ⇒ 𝑎 ≤ 𝑤 ∗ , ∀𝑎 ∈ 𝐴 (by definition)
⇒ 𝑎 + 𝑏 ≤ 𝑤 ∗ + 𝑏, ∀𝑎 ∈ 𝐴
⇒ 𝑤 ∗ + 𝑏 is an UB of 𝐴 + 𝑏 (by definition)
⇒ 𝑢∗ ≤ 𝑤 ∗ + 𝑏 (by 𝐶2 on 𝐴 + 𝑏)
⇒ 𝑢∗ ≤ 𝑤 ∗ + 𝑏……②
①&②: ⇒ 𝑢∗ = 𝑤 ∗ + 𝑏 ⇒ sup(𝐴 + 𝑏) = (sup 𝐴) + 𝑏.
19
(2) inf(𝐴 + 𝑏) = ⏟
− sup[−(𝐴 + 𝑏)] = − sup(−𝐴 − 𝑏) = ⏟
−[sup(−𝐴) − 𝑏]
by Theorem 3 by part (1)
Solution:
6𝑛+1 1 1
(1) sup { : 𝑛 ∈ ℕ} = sup {3 + 2𝑛 : 𝑛 ∈ ℕ} = 3 + sup {2𝑛 : 𝑛 ∈ ℕ}
2𝑛
1 1 1 7
= 3 + sup { : 𝑛 ∈ ℕ} = 3 + (1) =
2 𝑛 2 2
2𝑛2 +𝑛−1
(2) inf { : 𝑛 ∈ ℕ} = inf{2𝑛 − 1: 𝑛 ∈ ℕ} = inf{2𝑛: 𝑛 ∈ ℕ} − 1
𝑛+1
Solution:
1 1 1 1
(1) sup {√𝑛 + 1 − √𝑛 ∶ 𝑛 ∈ ℕ} = sup { ∶ 𝑛 ∈ ℕ} = sup { , , ,…}
√𝑛+1+√𝑛 √2+1 √3+√2 √4+√3
1
= .
√2+1
1 1 1 1
(2) inf {√𝑛 + 1 − √𝑛 ∶ 𝑛 ∈ ℕ} = inf { ∶ 𝑛 ∈ ℕ} = inf { , , ,…}
√𝑛+1+√𝑛 √2+1 √3+√2 √4+√3
= 0.
20
Definition 2.4.12: Let 𝐴 and 𝐵 be two sets of real numbers. Define the sets 𝐴 + 𝐵 by
𝐴 + 𝐵 = {𝑎 + 𝑏: 𝑎 ∈ 𝐴, 𝑏 ∈ 𝐵}
Example 2.4.13: If 𝐴 = {−1,0} and 𝐵 = {−3,0,1}.
(1) Find 𝐴 + 𝐵 (2) Find 𝐴 + 𝐴 (3) Find 2𝐴 (4) Is 𝐴 + 𝐴 = 2𝐴
Solution:
(1) 𝐴 + 𝐵 = {−1 + (−3), −1 + 0, −1 + 1,0 + (−3), 0 + 0,0 + 1} = {−4, −1,0, −3,0,1}
⇒ 𝐴 + 𝐵 = {−4, −3, −1,0,1}
(2) 𝐴 + 𝐴 = {−1 + (−1), −1 + 0,0 + (−1), 0 + 0} = {−2, −1, −1,0} = {−2, −1,0}
(3) 2𝐴 = {−2,0}
(4) 𝐴 + 𝐴 ≠ 2𝐴
Theorem 2.4.14: Let 𝐴 and 𝐵 be two sets of real numbers. Then
(1) sup(𝐴 + 𝐵) = sup 𝐴 + sup 𝐵
(2) inf(𝐴 + 𝐵) = inf 𝐴 + inf 𝐵
(3) sup(𝐴 + 𝐴) = 2 sup 𝐴
(4) inf(𝐴 + 𝐴) = 2 inf 𝐴
Proof:
(1) Let sup(𝐴 + 𝐵) = 𝑢∗ , sup 𝐴 = 𝑣 ∗ and sup 𝐵 = 𝑤 ∗ . Want to show that
𝑢∗ = 𝑣 ∗ + 𝑤 ∗ by showing that 𝑢∗ ≥ 𝑣 ∗ + 𝑤 ∗ and 𝑢∗ ≤ 𝑣 ∗ + 𝑤 ∗ :
Step 1: sup(𝐴 + 𝐵) = 𝑢∗ :
𝐶1 on 𝐴 + 𝐵: 𝑢∗ is an UB of 𝐴 + 𝐵 ⇒ 𝑎 + 𝑏 ≤ 𝑢∗ , ∀𝑎 ∈ 𝐴, ∀𝑏 ∈ 𝐵 (by definition)
⇒ [𝑎 ≤ 𝑢∗ − 𝑏, ∀𝑎 ∈ 𝐴], ∀𝑏 ∈ 𝐵
⇒ [𝑢∗ − 𝑏 is an UB of 𝐴],∀𝑏 ∈ 𝐵 (by definition)
⇒ [𝑣 ∗ ≤ 𝑢∗ − 𝑏], ∀𝑏 ∈ 𝐵 (by 𝐶2 on 𝐴)
⇒ [𝑏 ≤ 𝑢∗ − 𝑣 ∗ ], ∀𝑏 ∈ 𝐵
⇒ 𝑢∗ − 𝑣 ∗ is an UB of 𝐵 (by definition)
⇒ 𝑤 ∗ ≤ 𝑢∗ − 𝑣 ∗ (by 𝐶2 on 𝐵)
⇒ 𝑤 ∗ + 𝑣 ∗ ≤ 𝑢∗ …….①
Step 2: sup 𝐴 = 𝑣 ∗ and sup 𝐵 = 𝑤 ∗ :
𝐶1 on 𝐴: 𝑣 ∗ is an UB of 𝐴 ⇒ 𝑎 ≤ 𝑣 ∗ , ∀𝑎 ∈ 𝐴……...②
𝐶1 on 𝐵: 𝑤 ∗ is an UB of 𝐵 ⇒ 𝑏 ≤ 𝑤 ∗ , ∀𝑏 ∈ 𝐵….....③
②+③: ⇒ 𝑎 + 𝑏 ≤ 𝑣 ∗ + 𝑤 ∗ , ∀𝑎 ∈ 𝐴, ∀𝑏 ∈ 𝐵
⇒ 𝑣 ∗ + 𝑤 ∗ is an UB of 𝐴 + 𝐵 ⇒ 𝑢∗ ≤ 𝑣 ∗ + 𝑤 ∗ ……..④
①&④: ⇒ 𝑢∗ = 𝑣 ∗ + 𝑤 ∗ ⇒ sup(𝐴 + 𝐵) = sup 𝐴 + sup 𝐵
21
= − [sup(−𝐴)
⏟ + sup(−𝐵)] = − sup(−𝐴) − sup(−𝐵)
by part (1)
= ⏟ inf 𝐴 + ⏟ inf 𝐵
by Theorem 3 by Theorem 3
(−1)𝑛 5
(2) inf { − 𝑚+1 − 3: 𝑚, 𝑛 ∈ ℕ}
2𝑛
Solution:
(−1)𝑛 5 (−1)𝑛 −5
(1) sup { − 𝑚+1 − 3: 𝑚, 𝑛 ∈ ℕ} = sup { + 𝑚+1 : 𝑚, 𝑛 ∈ ℕ} − 3
2𝑛 2𝑛
(−1)𝑛 −5
= sup { : 𝑛 ∈ ℕ} + sup { : 𝑚 ∈ ℕ} − 3
2𝑛 𝑚+1
1 (−1)𝑛 1
= sup { : 𝑛 ∈ ℕ} − 5 inf { : 𝑚 ∈ ℕ} − 3
2 𝑛 𝑚+1
1 1 1 1 1 1 1
= sup {−1, , − , , … } − 5 inf { , , , … } − 3
2 2 3 4 2 3 4
1 1
= ( ) − 5(0) − 3
2 2
= −2.75
(−1)𝑛 5 (−1)𝑛 −5
(2) inf { − 𝑚+1 − 3: 𝑚, 𝑛 ∈ ℕ} = inf { + 𝑚+1 : 𝑚, 𝑛 ∈ ℕ} − 3
2𝑛 2𝑛
(−1)𝑛 −5
= inf { : 𝑛 ∈ ℕ} + inf { : 𝑚 ∈ ℕ} − 3
2𝑛 𝑚+1
1 (−1)𝑛 1
= inf { : 𝑛 ∈ ℕ} − 5 sup { : 𝑚 ∈ ℕ} − 3
2 𝑛 𝑚+1
1 1 1 1 1 1 1
= inf {−1, , − , , … } − 5 sup { , , , … } − 3
2 2 3 4 2 3 4
1 1
= (−1) − 5 ( ) − 3
2 2
= −6
Exercises 2.4.16 (Pages 44-45): 1,2,8
22
1 1
Example 2.5.6: Let 𝐼𝑛 = [−1 + 𝑛 , 1 − 𝑛], 𝑛 ∈ ℕ. Find ∪ 𝐼𝑛 and ∩ 𝐼𝑛 if they exist.
1 1 2 2 3 3
Solution: 𝐼1 = [0,0] = {0}, 𝐼2 = [− 2 , 2] , 𝐼3 = [− 3 , 3] , 𝐼4 = [− 4 , 4] , …
⇒ ∪ 𝐼𝑛 = (𝑎, 𝑏) = (−1,1).
24
1
Example 2.5.7: Let 𝐼𝑛 = (0, 𝑛), 𝑛 ∈ ℕ. Find ∪ 𝐼𝑛 and ∩ 𝐼𝑛 if they exist.
1 1 1
Solution: 𝐼1 = (0,1), 𝐼2 = (0, 2) , 𝐼3 = (0, 3) , 𝐼4 = (0, 4) , …
1 1
Example 2.5.8: Let 𝐼𝑛 = (− 𝑛 , 𝑛], 𝑛 ∈ ℕ. Find ∪ 𝐼𝑛 and ∩ 𝐼𝑛 if they exist.
1 1 1 1 1 1
Solution: 𝐼1 = (−1,1], 𝐼2 = (− 2 , 2] , 𝐼3 = (− 3 , 3] , 𝐼4 = (− 4 , 4] , …
⇒ ∪ 𝐼𝑛 = [0,1)
25
1 1
Example 2.5.11: Let 𝐼𝑛 = (− 𝑛 , 1 + 𝑛), 𝑛 ∈ ℕ. Find ∪ 𝐼𝑛 and ∪ 𝐼𝑛 if they exist.
1 3 1 4 1 5
Solution: 𝐼1 = (−1,2), 𝐼2 = (− 2 , 2) , 𝐼3 = (− 3 , 3) , 𝐼4 = (− 4 , 4) , …
Definition 3.1.1: A sequence of real numbers (or a sequence in ℝ) is a function 𝑋: ℕ → ℝ and this
function is denote by the notations 𝑋, (𝑥𝑛 ), or (𝑥𝑛 : 𝑛 ∈ ℕ).
It is convenient to list the terms of a sequence as 𝑋 = (𝑥𝑛 : 𝑛 ∈ ℕ) = (𝑥1 , 𝑥2 , 𝑥3 , … )
Example 3.1.2: Plot the graph of the sequence 𝑋 = (2.5, −1,0, 4, 3, −1, 1, 1, 0.5, … ).
Solution: 𝑥1 = 2.5, 𝑥2 = −1, 𝑥3 = 0, 𝑥4 = 4, 𝑥5 = 3, 𝑥6 = −1, 𝑥7 = 1, 𝑥8 = 1, 𝑥9 = 0.5 , …
y
4.5
4
3.5
3
2.5
2
1.5
1
0.5 x
-0.5 1 2 3 4 5 6 7 8 9
-1
-1.5
Remark 3.1.3:
(1) Sequences may be denoted by other letters, such as 𝑌 = (𝑦𝑘 ), 𝑍 = (𝑧𝑗 ) and so on.
(2) Sequences are often defined by giving a formula for the 𝑛𝑡ℎ term 𝑥𝑛 . For example:
1 1 1 1 1
𝑋 = ( , , , , … ) = ( : 𝑛 ∈ ℕ)
2 4 6 8 2𝑛
1
or more simply 𝑋 = (2𝑛).
𝑛 1 2 3
(3) For example: If (𝑥𝑛 ) = (𝑛+1) ⇒ (𝑥𝑛 ) = (2 , 3 , 4 , … )
Definition 3.1.4: If 𝑏 ∈ ℝ, the sequence 𝐵 = (𝑏, 𝑏, 𝑏, … ), all of whose terms equal 𝑏, is called the
constant sequence 𝑏.
The constant sequence 1 is the sequence (1,1,1, … )
The constant sequence 0 is the sequence (0,0,0, … ).
27
⇒ 𝑓𝑛+1 | = 𝑓2 + 𝑓1 = 1 + 1 = 2 ⇒ 𝑓3 = 2
𝑛=2
⇒ 𝑓𝑛+1 | = 𝑓3 + 𝑓2 = 2 + 1 = 3 ⇒ 𝑓4 = 3
𝑛=3
⇒ 𝑓5 = 𝑓4 + 𝑓3 = 3 + 2 = 5, 𝑓6 = 𝑓5 + 𝑓4 = 5 + 3 = 8, …
⇒ (𝑓𝑛 ) = (1, 1, 2, 3, 5, 8, 13, 21, 34, 55, … )
Question 3.1.8: What is 𝑓13 in the Fibonacci sequence (𝑓𝑛 )?
① ②
that ⏞
|𝑥𝑛 − 𝑥| < 𝜀, ∀𝑛 ≥ 𝐾𝜀 .
Remark 3.1.10:
If (𝑥𝑛 ) converges to 𝑥 we say that 𝑥 is a limit of (𝑥𝑛 ) and we write lim 𝑥𝑛 = 𝑥
𝑛→∞
If a sequence has no limit, that is lim 𝑥𝑛 does not exist, we say that the sequence (𝑥𝑛 )
𝑛→∞
is divergent.
28
Remark 3.1.11:
(1) The notation 𝐾𝜀 is used to emphasize that the choice of 𝐾 depends on the value of 𝜀.
However, it is often convenient to write 𝐾 instead of 𝐾𝜀 .
(2) In most cases, a ‘‘small’’ value of 𝜀 will usually require a ‘‘large’’ value of 𝐾 to guarantee
that |𝑥𝑛 − 𝑥| < 𝜀, ∀𝑛 ≥ 𝐾.
Theorem 3.1.12: Let (𝑥𝑛 ) be a sequence of real numbers, and let 𝑥 ∈ ℝ. The following statements
are equivalent.
(a) (𝑥𝑛 ) converges to 𝑥.
(b) ∀𝜀 > 0, ∃𝐾 ∈ ℕ such that |𝑥𝑛 − 𝑥| < 𝜀, ∀𝑛 ≥ 𝐾.
(c) ∀𝜀 > 0, ∃𝐾 ∈ ℕ such that 𝑥 − 𝜀 < 𝑥𝑛 < 𝑥 + 𝜀, ∀𝑛 ≥ 𝐾.
(d) ∀𝜀, ∃𝐾 ∈ ℕ such that 𝑥𝑛 ∈ (𝑥 − 𝜀, 𝑥 + 𝜀), ∀𝑛 ≥ 𝐾.
(e) ∀𝜀, ∃𝐾 ∈ ℕ such that 𝑥𝑛 ∈ 𝑉𝜀 (𝑥), ∀𝑛 ≥ 𝐾.
Proof:
[(𝑥𝑛 ) converges to 𝑥] iff [∀𝜀 > 0, ∃𝐾 ∈ ℕ such that |𝑥𝑛 − 𝑥| < 𝜀, ∀𝑛 ≥ 𝐾]
iff [∀𝜀 > 0, ∃𝐾 ∈ ℕ such that −𝜀 < 𝑥𝑛 − 𝑥 < 𝜀, ∀𝑛 ≥ 𝐾]
iff [∀𝜀 > 0, ∃𝐾 ∈ ℕ such that 𝑥 − 𝜀 < 𝑥𝑛 < 𝑥 + 𝜀, ∀𝑛 ≥ 𝐾]
iff [∀𝜀 > 0, ∃𝐾 ∈ ℕ such that 𝑥𝑛 ∈ (𝑥 − 𝜀, 𝑥 + 𝜀), ∀𝑛 ≥ 𝐾]
iff [∀𝜀 > 0, ∃𝐾 ∈ ℕ such that 𝑥𝑛 ∈ 𝑉𝜀 (𝑥), ∀𝑛 ≥ 𝐾].
Theorem Uniqueness of Limit 3.1.13: A sequence in ℝ can have at most one limit.
Proof: Suppose that 𝑥 ′ and 𝑥 ′′ are both limits of (𝑥𝑛 ). Want to show that 𝑥 ′ = 𝑥 ′′ .
Let 𝜀 > 0 be given.
𝜀
Since lim 𝑥𝑛 = 𝑥 ′ ⇒ ∃𝐾 ′ ∈ ℕ such that |𝑥𝑛 − 𝑥 ′ | < 2 , ∀𝑛 ≥ 𝐾 ′ ……..①
𝑛→∞
𝜀
Since lim 𝑥𝑛 = 𝑥 ′′ ⇒ ∃𝐾 ′′ ∈ ℕ such that |𝑥𝑛 − 𝑥 ′′ | < , ∀𝑛 ≥ 𝐾 ′′ ……..②
2
-2 -1 1 2 3 4 5 6 7 8 9 10 11 12 13 14 15 16 17 18 19 20 21
-0.5
Solution: First observe that lim 𝑥𝑛 = 1.5 ⇒ Want to find the smallest 𝐾 ∈ ℕ such that
𝑛→∞
18
⇒ |𝑥𝑛 − 𝑥| ≤ ……①
𝑛
18 18 18
Let <𝜀 ⇔ <𝑛 ⇔𝑛> ……….②
𝑛 𝜀 𝜀
18
By the Archimedean Property ∃𝐾 ∈ ℕ such that 𝐾 > .
𝜀
18 18
If 𝑛 ≥ 𝐾 ⇒ 𝑛 > . So, by ② we have < 𝜀, ∀𝑛 ≥ 𝐾.
𝜀 𝑛
Solution:
Let 𝜀 > 0 be given. Take 𝑥𝑛 = √𝑛 + 1 − √𝑛 and 𝑥 = 0.
?
⏞ ∈ ℕ such that |𝑥𝑛 − 𝑥| < 𝜀, ∀𝑛 ≥ 𝐾.
Want to find 𝐾
√𝑛+1+√𝑛
|𝑥𝑛 − 𝑥| = |√𝑛 + 1 − √𝑛 − 0| = √𝑛 + 1 − √𝑛 = (√𝑛 + 1 − √𝑛) ×
√𝑛+1+√𝑛
1 1
= < …………①
√𝑛+1+√𝑛 √𝑛
1 1 1
Let < 𝜀 ⇔ √𝑛 > 𝜀 ⇔ 𝑛 > 𝜀2…….②
√𝑛
1
By the Archimedean Property ∃𝐾 ∈ ℕ such that 𝐾 > 𝜀2 .
1 1
If 𝑛 ≥ 𝐾 ⇒ 𝑛 > 𝜀2 and by ② we have < 𝜀, ∀𝑛 ≥ 𝐾
√𝑛
ln(ε)
By the Archimedean Property take 𝐾 ∈ ℕ such that 𝐾 > ln(𝑏).
ln(ε)
If 𝑛 ≥ 𝐾 ⇒ 𝑛 > ln(𝑏) , ∀𝑛 ≥ 𝐾 and by ① we have |𝑥𝑛 − 𝑥| < 𝜀, ∀𝑛 ≥ 𝐾
1
Example 3.1.19: Using definition, show that lim =0
𝑛→∞ 𝑛
?
1
⏞ ∈ ℕ such that
Solution: Let 𝜀 > 0 be given. Take 𝑥𝑛 = 𝑛 and 𝑥 = 0. Want to find 𝐾
|𝑥𝑛 − 𝑥| < 𝜀, ∀𝑛 ≥ 𝐾.
1 1 1
|𝑥𝑛 − 𝑥| < 𝜀 ⇔ | − 0| < 𝜀 ⇔ < 𝜀 ⇔ 𝑛 > 𝜀 ……….①
𝑛 𝑛
1 1
By the Archimedean Property take 𝐾 ∈ ℕ such that 𝐾 > 𝜀 : If 𝑛 ≥ 𝐾 ⇒ 𝑛 > 𝜀 , ∀𝑛 ≥ 𝐾
1
⇒ ① holds ⇒ |𝑥𝑛 − 𝑥| < 𝜀, ∀𝑛 ≥ 𝐾 ⇒ lim =0
𝑛→∞ 𝑛
Example 3.1.20: Using definition, show that the sequence ((−1)𝑛 ) = (−1,1, −1,1, … ) diverges.
Solution: The proof by contradiction.
Assume that the sequence is convergent to a real number 𝑥 ⇒ lim 𝑥𝑛 = 𝑥.
𝑛→∞
⇒ The sequence has two different limits which is a contradiction (since the limit is unique)
⇒ Our assumption is false ⇒ The sequence diverges.
32
Solution:
lim 𝑥𝑛 = 0 ⇔ ∀𝜀 > 0, ∃𝐾 ∈ ℕ such that |𝑥𝑛 − 0| < 𝜀, ∀𝑛 ≥ 𝐾
𝑛→∞
Also, lim 𝑥𝑛 ≠ 1 because lim 𝑥𝑛 does not exist (by Example 3.1.20 (𝑥𝑛 ) = ((−1)𝑛 ) diverges)
𝑛→∞ 𝑛→∞
The terms 𝑥1 , 𝑥2 , … , 𝑥𝑚 were dropped from the sequence 𝑋 to get the 𝑚-tail 𝑋𝑚 .
Example 3.1.24: The 3-tail of the sequence 𝑋 = (2, 4, 6, 8, … , 2𝑛, … ) is the sequence:
𝑋3 = (𝑥4 , 𝑥5 , … ) = (8, 10, 12, … )
Lemma 3.1.25: Let 𝑋 = (𝑥𝑛 : 𝑛 ∈ ℕ) be a sequence of real numbers and let 𝑚 ∈ ℕ. Then the 𝑚-
tail 𝑋𝑚 = (𝑥𝑚+𝑛 : 𝑛 ∈ ℕ) converges to 𝑥 if and only if 𝑋 converges to 𝑥.
In this case, lim 𝑥𝑛 = lim 𝑥𝑚+𝑛 = 𝑥.
𝑛→∞ 𝑛→∞
33
Definition 3.1.26: A sequence (𝑥𝑛 ) is called ultimately has a certain property if some tail of (𝑥𝑛 )
has this property.
Example 3.1.27:
(1) The sequence 𝑋 = (3, 4, 5, 5, … , 5, … ) is ultimately constant since the tail
𝑋2 = (5, 5, 5, … . ) is constant.
(2) The sequence 𝑌 = (3, 5, 3, 5, 3, 5, … ) is not ultimately constant since it doesn’t has a
constant tail.
Theorem 3.1.28 (Comparison Theorem of sequences):
Let (𝑥𝑛 ) be a sequence of real numbers and let 𝑥 ∈ ℝ. If (𝑎𝑛 ) is a sequence of positive real numbers
with lim 𝑎𝑛 = 0 and if for some constant 𝐶 > 0 and some 𝐾 ∈ ℕ we have
𝑛→∞
1
Example 3.1.29: Show that if 𝑎 > 0, then lim =0
𝑛→∞ 1+𝑛𝑎
Solution:
1 1 1 1 1
| − 0| = < = ( ) , ∀𝑛 ≥ 1 … … … ①
1 + 𝑎𝑛 1 + 𝑛𝑎 𝑛𝑎 𝑎 𝑛
1
But lim = 0……….②
𝑛→∞ 𝑛
1
①&②& Comparison Theorem 26 ⇒ lim = 0.
𝑛→∞ 1+𝑛𝑎
Definition 3.2.1: A sequence (𝑥𝑛 ) of real numbers is said to be bounded if ∃𝑀 > 0 such that
|𝑥𝑛 | ≤ 𝑀, ∀𝑛 ∈ ℕ.
Thus, the sequence (𝑥𝑛 ) is bounded if and only if the set {𝑥𝑛 : 𝑛 ∈ ℕ} is a bounded subset of ℝ
⇒ (𝑥𝑛 ) is bounded if and only if ∃𝑀 > 0 such that −𝑀 ≤ 𝑥𝑛 ≤ 𝑀, ∀𝑛 ∈ ℕ.
Remark 3.2.3:
(1) The convers of Theorem 3.2.2 is not true. Consider the following counterexample:
Let 𝑥𝑛 = (−1)𝑛 ⇒ |𝑥𝑛 | ≤ 2, ∀𝑛 ∈ ℕ ⇒ (𝑥𝑛 ) is bounded. But (𝑥𝑛 ) = ((−1)𝑛 ) diverges
(2) The contraposition of Theorem 3.2.2 is the following:
If a sequence (𝑥𝑛 ) is unbounded, then (𝑥𝑛 ) is divergent.
For example:
The sequence (𝑛) = (1,2,3,4, … ) diverges (since it is unbounded)
The sequence ((−1)𝑛 𝑛) = (−1,2, −3,4, −5,6, … ) diverges (since it is unbounded).
Theorem 3.2.4:
(1) Let (𝑥𝑛 ) be a sequence that converges to 𝑥, and let 𝑐 ∈ ℝ. Then (𝑐𝑥𝑛 ) converges to 𝑐𝑥, that
is lim (𝑐𝑥𝑛 ) = 𝑐 lim 𝑥𝑛
𝑛→∞ 𝑛→∞
(2) Let (𝑥𝑛 ) and (𝑦𝑛 ) be sequences that converge to 𝑥 and 𝑦, respectively, and let 𝑐 ∈ ℝ. Then
a) (𝑥𝑛 + 𝑦𝑛 ) converges to 𝑥 + 𝑦, that is lim (𝑥𝑛 + 𝑦𝑛 ) = lim 𝑥𝑛 + lim 𝑦𝑛
𝑛→∞ 𝑛→∞ 𝑛→∞
b) (𝑥𝑛 − 𝑦𝑛 ) converges to 𝑥 − 𝑦, that is lim (𝑥𝑛 − 𝑦𝑛 ) = lim 𝑥𝑛 − lim 𝑦𝑛
𝑛→∞ 𝑛→∞ 𝑛→∞
c) (𝑥𝑛 𝑦𝑛 ) converges to 𝑥𝑦, , that is lim 𝑥𝑛 𝑦𝑛 = ( lim 𝑥𝑛 ) ( lim 𝑦𝑛 )
𝑛→∞ 𝑛→∞ 𝑛→∞
(3) If (𝑥𝑛 ) converges to 𝑥 and (𝑦𝑛 ) is a sequence of nonzero real numbers that converges to 𝑦 and
𝑥 𝑥
if 𝑦 ≠ 0, then the quotient sequence (𝑦𝑛) converges to 𝑦, that is
𝑛
𝑥𝑛 𝑛→∞lim 𝑥𝑛
lim =
𝑛→∞ 𝑦𝑛 lim 𝑦𝑛
𝑛→∞
35
Proof:
(1) Want to show that lim(𝑐𝑥𝑛 ) = 𝑐𝑥 that is ∀𝜀 > 0,∃𝐾 ∈ ℕ such that
|𝑐𝑥𝑛 − 𝑐𝑥| < 𝜀, ∀𝑛 ≥ 𝐾.
Let 𝜀 > 0 be given
Since lim 𝑥𝑛 = 𝑥 ⇒ If 𝜀1 > 0, then ∃𝐾 ∈ ℕ such that |𝑥𝑛 − 𝑥| < 𝜀1 , ∀𝑛 ≥ 𝐾……①
𝑛→∞
𝜀 𝜀
Take 𝜀1 = |𝑐|+1 so ① ⇒ |𝑥𝑛 − 𝑥| < |𝑐|+1 , ∀𝑛 ≥ 𝐾……②
𝜀 |𝑐|
So, |𝑐𝑥𝑛 − 𝑐𝑥| = |𝑐||𝑥𝑛 − 𝑥| < |𝑐| |𝑐|+1 = |𝑐|+1 𝜀 ≤ 𝜀, ∀𝑛 ≥ 𝐾
⇒ |𝑐𝑥𝑛 − 𝑐𝑥| < 𝜀, ∀𝑛 ≥ 𝐾
∴ (𝑐𝑥𝑛 ) converges to 𝑐𝑥 and lim (𝑐𝑥𝑛 ) = 𝑐 lim 𝑥𝑛
𝑛→∞ 𝑛→∞
(2)
(a) Want to show that lim (𝑥𝑛 + 𝑦𝑛 ) = 𝑥 + 𝑦 that is ∀𝜀 > 0,∃𝐾 ∈ ℕ such that
𝑛→∞
|(𝑥𝑛 + 𝑦𝑛 ) − (𝑥 + 𝑦)| < 𝜀, ∀𝑛 ≥ 𝐾.
Let 𝜀 > 0 be given
𝜀 𝜀
In the definition of lim 𝑥𝑛 = 𝑥 take 2 > 0: ∃𝐾1 ∈ ℕ such that |𝑥𝑛 − 𝑥| < 2 , ∀𝑛 ≥ 𝐾1 ……③
𝑛→∞
𝜀 𝜀
In the definition of lim 𝑦𝑛 = 𝑦 take 2 > 0: ∃𝐾2 ∈ ℕ such that |𝑦𝑛 − 𝑦| < 2 , ∀𝑛 ≥ 𝐾2...…④
𝑛→∞
Let 𝐾 = max( 𝐾1 , 𝐾2 ) ⇒ 𝐾 ∈ ℕ
If 𝑛 ≥ 𝐾 ⇒ 𝑛 ≥ 𝐾1 and 𝑛 ≥ 𝐾2
𝜀
From ③: |𝑥𝑛 − 𝑥| < 2 , ∀𝑛 ≥ 𝐾 ………⑤
𝜀
From ④: |𝑦𝑛 − 𝑦| < 2 , ∀𝑛 ≥ 𝐾 ………⑥
𝜀 𝜀
Now, |(𝑥𝑛 + 𝑦𝑛 ) − (𝑥 + 𝑦)| ≤
⏟ |𝑥𝑛 − 𝑥| + |𝑦𝑛 − 𝑦| <
⏟ 2 + 2 = 𝜀, ∀𝑛 ≥ 𝐾
by the triangle inequality
by ⑤&⑥
⇒ |(𝑥𝑛 + 𝑦𝑛 ) − (𝑥 + 𝑦)| < 𝜀, ∀𝑛 ≥ 𝐾
∴ (𝑥𝑛 + 𝑦𝑛 ) converges to 𝑥 + 𝑦, that is lim (𝑥𝑛 + 𝑦𝑛 ) = lim 𝑥𝑛 + lim 𝑦𝑛
𝑛→∞ 𝑛→∞ 𝑛→∞
(b) lim (𝑥𝑛 − 𝑦𝑛 ) = lim (𝑥𝑛 + (−𝑦𝑛 )) = (−𝑦𝑛 ) = lim 𝑥𝑛 − lim 𝑦𝑛
𝑛→∞ 𝑛→∞ ⏟ lim 𝑥𝑛 + lim
𝑛→∞ ⏟ 𝑛→∞ 𝑛→∞ 𝑛→∞
by part (a) by part (1)
(c) Want to show that lim 𝑥𝑛 𝑦𝑛 = 𝑥𝑦 that is ∀𝜀 > 0,∃𝐾 ∈ ℕ such that
𝑛→∞
|𝑥𝑛 𝑦𝑛 − 𝑥𝑦| < 𝜀, ∀𝑛 ≥ 𝐾.
|𝑥𝑛 𝑦𝑛 − 𝑥𝑦| = |𝑥𝑛 𝑦𝑛 − 𝑥𝑦𝑛 + 𝑥𝑦𝑛 − 𝑥𝑦| = |(𝑥𝑛 − 𝑥)𝑦𝑛 + 𝑥(𝑦𝑛 − 𝑦)|
≤ |(𝑥𝑛 − 𝑥)𝑦𝑛 | + |𝑥(𝑦𝑛 − 𝑦)|
= |𝑥𝑛 − 𝑥||𝑦𝑛 | + |𝑥||𝑦𝑛 − 𝑦|………⑦
Let 𝜀 > 0 be given
(𝑦𝑛 ) converges ⇒ (𝑦𝑛 ) bounded (see Theorem 3.2.2) ⇒ ∃𝑀 > 0 such that
|𝑦𝑛 | ≤ 𝑀, ∀𝑛 ∈ ℕ……….⑧
𝜀
In the definition of lim 𝑥𝑛 = 𝑥 take 2𝑀 > 0: ∃𝐾1 ∈ ℕ such that
𝑛→∞
𝜀
|𝑥𝑛 − 𝑥| < , ∀𝑛 ≥ 𝐾1 ……⑨
2𝑀
𝜀
In the definition of lim 𝑦𝑛 = 𝑦 take 2(|𝑥|+1) > 0: ∃𝐾2 ∈ ℕ such that
𝑛→∞
𝜀
|𝑦𝑛 − 𝑦| < , ∀𝑛 ≥ 𝐾2 ...… ⑩
2(|𝑥|+1)
Let 𝐾 = max( 𝐾1 , 𝐾2 ) ⇒ 𝐾 ∈ ℕ
36
If 𝑛 ≥ 𝐾 ⇒ 𝑛 ≥ 𝐾1 and 𝑛 ≥ 𝐾2, so
𝜀
⑨ ⇒ |𝑥𝑛 − 𝑥| < 2𝑀 , ∀𝑛 ≥ 𝐾……⑪
𝜀
⑩ ⇒ |𝑦𝑛 − 𝑦| < 2(|𝑥|+1) , ∀𝑛 ≥ 𝐾...… ⑫
Now, |𝑥𝑛 𝑦𝑛 − 𝑥𝑦| = |𝑥𝑛 𝑦𝑛 − 𝑥𝑦𝑛 + 𝑥𝑦𝑛 − 𝑥𝑦|
= |(𝑥𝑛 − 𝑥)𝑦𝑛 + 𝑥(𝑦𝑛 − 𝑦)|
≤ |(𝑥𝑛 − 𝑥)𝑦𝑛 | + |𝑥(𝑦𝑛 − 𝑦)| (by the triangle inequality)
= |𝑥𝑛 − 𝑥||𝑦𝑛 | + |𝑥||𝑦𝑛 − 𝑦|
𝜀 𝜀
< 2𝑀 𝑀 + |𝑥| 2(|𝑥|+1) (by ⑧, ⑨&⑩)
𝜀 |𝑥| 𝜀
= 2 + (|𝑥|+1) 2
𝜀 𝜀
≤2+2
= 𝜀, ∀𝑛 ≥ 𝐾
⇒ |𝑥𝑛 𝑦𝑛 − 𝑥𝑦| < 𝜀, ∀𝑛 ≥ 𝐾
∴ (𝑥𝑛 𝑦𝑛 ) converges to 𝑥𝑦, that is lim (𝑥𝑛 𝑦𝑛 ) = ( lim 𝑥𝑛 ) ( lim 𝑦𝑛 )
𝑛→∞ 𝑛→∞ 𝑛→∞
(3) Is an exercise
Theorem 3.2.7:
(1) If (𝑥𝑛 ) and (𝑦𝑛 ) are convergent sequences and if 𝑥𝑛 ≤ 𝑦𝑛 , ∀𝑛 ∈ ℕ (or 𝑥𝑛 < 𝑦𝑛 , ∀𝑛 ∈ ℕ), then
lim 𝑥𝑛 ≤ lim 𝑦𝑛 .
𝑛→∞ 𝑛→∞
(2) If (𝑥𝑛 ) is a convergent sequence and 𝑎 ≤ 𝑥𝑛 ≤ 𝑏, ∀𝑛 ∈ ℕ (or 𝑎 < 𝑥𝑛 < 𝑏, ∀𝑛 ∈ ℕ), then 𝑎 ≤
lim 𝑥𝑛 ≤ 𝑏.
Proof:
(1) Let 𝑧𝑛 = 𝑦𝑛 − 𝑥𝑛 , 𝑛 ∈ ℕ ⇒ 𝑧𝑛 ≥ 0, ∀𝑛 ∈ ℕ (since 𝑥𝑛 ≤ 𝑦𝑛 , ∀𝑛 ∈ ℕ)
⇒ lim 𝑧𝑛 ≥ 0 ⇒ lim (𝑦𝑛 − 𝑥𝑛 ) ≥ 0 ⇒ lim 𝑦𝑛 − lim 𝑥𝑛 ≥ 0 ⇒ lim 𝑦𝑛 ≥ lim 𝑥𝑛
𝑛→∞ 𝑛→∞ 𝑛→∞ 𝑛→∞ 𝑛→∞ 𝑛→∞
⇒ 𝑎 ≤ lim 𝑥𝑛 ≤ 𝑏
𝑛→∞
Squeeze Theorem 3.2.8: Let (𝑥𝑛 ), (𝑦𝑛 ), and (𝑧𝑛 ) be sequences of real numbers such that
𝑥𝑛 ≤ 𝑦𝑛 ≤ 𝑧𝑛 , ∀𝑛 ∈ ℕ. If (𝑥𝑛 ) and (𝑧𝑛 ) are convergent with lim 𝑥𝑛 = lim 𝑧𝑛 = 𝑤. Then (𝑦𝑛 ) is
𝑛→∞ 𝑛→∞
convergent and lim 𝑦𝑛 = 𝑤.
𝑛→∞
Let 𝐾 = max(𝐾1 , 𝐾2 ) ⇒ 𝐾 ∈ ℕ.
If 𝑛 ≥ 𝐾 ⇒ 𝑛 ≥ 𝐾1 and 𝑛 ≥ 𝐾2
Remark 3.2.9:
(1) A sequence (𝑥𝑛 ) converges to 𝑥 iff any tail (𝑥𝑚+𝑛 ) of (𝑥𝑛 ) also converges to 𝑥, that is:
lim 𝑥𝑛 = 𝑥 iff lim 𝑥𝑚+𝑛 = 𝑥, ∀𝑚 ∈ ℕ.
𝑛→∞ 𝑛→∞
(2) Theorems 3.2.5,3.3.7 and 3.2.8 also true for the tails of sequences, that is:
Theorem 3.2.10: Let (𝑥𝑛 ) is a convergent sequence.
(1) If 𝑥𝑛 ≥ 0, ∀𝑛 ≥ 𝐾 (or 𝑥𝑛 > 0, ∀𝑛 ≥ 𝐾), then lim 𝑥𝑛 ≥ 0.
𝑛→∞
(2) If 𝑥𝑛 ≤ 0, ∀𝑛 ≥ 𝐾 (or 𝑥𝑛 < 0, ∀𝑛 ≥ 𝐾), then lim 𝑥𝑛 ≤ 0.
𝑛→∞
Theorem 3.2.11:
(1) If (𝑥𝑛 ) and (𝑦𝑛 ) are convergent sequences and if 𝑥𝑛 ≤ 𝑦𝑛 , ∀𝑛 ≥ 𝐾 (or 𝑥𝑛 ≤ 𝑦𝑛 , ∀𝑛 ≥ 𝐾),
then lim 𝑥𝑛 ≤ lim 𝑦𝑛 .
𝑛→∞ 𝑛→∞
(2) If (𝑥𝑛 ) is a convergent sequence and 𝑎 ≤ 𝑥𝑛 ≤ 𝑏, ∀𝑛 ≥ 𝐾 (or 𝑎 < 𝑥𝑛 < 𝑏, ∀𝑛 ≥ 𝐾),
then 𝑎 ≤ lim 𝑥𝑛 ≤ 𝑏.
Squeeze Theorem 3.2.12: Let (𝑥𝑛 ), (𝑦𝑛 ), and (𝑧𝑛 ) be sequences of real numbers such that
𝑥𝑛 ≤ 𝑦𝑛 ≤ 𝑧𝑛 , ∀𝑛 ≥ 𝐾. If (𝑥𝑛 ) and (𝑧𝑛 ) are convergent with lim 𝑥𝑛 = lim 𝑧𝑛 = 𝑤. Then
𝑛→∞ 𝑛→∞
(𝑦𝑛 ) is convergent and lim 𝑦𝑛 = 𝑤.
𝑛→∞
𝑥
Take 𝜀 = 𝐿 − 𝑟 in the definition of lim 𝑥𝑛+1 = 𝐿 ⇒ ∃𝐾 ∈ ℕ such that
𝑛→∞ 𝑛
𝑥𝑛+1
| − 𝐿| < 𝐿 − 𝑟, ∀𝑛 ≥ 𝐾
𝑥𝑛
𝑥
⇒ −(𝐿 − 𝑟) < 𝑥𝑛+1 − 𝐿 < 𝐿 − 𝑟, ∀𝑛 ≥ 𝐾
𝑛
𝑥𝑛+1 𝑥𝑛+1
⇒ −𝐿 + 𝑟 < − 𝐿, ∀𝑛 ≥ 𝐾 ⇒ 𝑟 < , ∀𝑛 ≥ 𝐾
𝑥𝑛 𝑥𝑛
⇒ 𝑥𝑛+1 > 𝑟𝑥𝑛 , ∀𝑛 ≥ 𝐾…..④
For 𝑛 = 𝐾 ∶ ④ ⇒ 𝑥𝐾+1 > 𝑟𝑥𝐾
For 𝑛 = 𝐾 + 1 ∶ ④ ⇒ 𝑥𝐾+2 > 𝑟𝑥𝐾+1 ⇒ 𝑥𝐾+2 > 𝑟 2 𝑥𝐾
For 𝑛 = 𝐾 + 2 ∶ ④ ⇒ 𝑥𝐾+3 > 𝑟𝑥𝐾+2 ⇒ 𝑥𝐾+3 > 𝑟 3 𝑥𝐾
For 𝑛 = 𝐾 + 3 ∶ ④ ⇒ 𝑥𝐾+4 < 𝑟𝑥𝐾+3 ⇒ 𝑥𝐾+4 > 𝑟 4 𝑥𝐾
⋮
For general 𝑛 ≥ 𝐾 ∶ ④ ⇒ 𝑥𝑛 > 𝑟 𝑛−𝐾 𝑥𝐾 = 𝐶𝑟 𝑛 ,……..⑤
where 𝐶 = 𝑟 −𝐾 𝑥𝐾
Since 𝑟 > 1 and 𝐶 > 0⇒ lim 𝐶𝑟 𝑛 = ∞ ……….⑥
𝑛→∞
⑤&⑥⇒ lim 𝑥𝑛 = ∞ ⇒ (𝑥𝑛 ) diverges.
𝑛→∞
(3) To show that the test fails if 𝐿 = 1:
1
1 𝑥𝑛+1 𝑛
Take 𝑥𝑛 = 𝑛 ⇒ (𝑥𝑛 ) converges and 𝐿 = lim = lim 𝑛+1
1 = lim =1
𝑥𝑛 𝑛+1
𝑛
𝑥𝑛+1 𝑛+1
Take 𝑥𝑛 = 𝑛 ⇒ (𝑥𝑛 ) diverges and 𝐿 = lim = lim =1
𝑥𝑛 𝑛
Proof:
(1) Let 0 ≤ 𝐿 < 1 ⇒ ∃𝑟 ∈ (𝐿, 1) that is 𝐿 < 𝑟 < 1 ⇒ 𝑟 − 𝐿 > 0
Take 𝜀 = 𝑟 − 𝐿 in the definition of lim 𝑛√𝑥𝑛 = 𝐿 ⇒ ∃𝐾 ∈ ℕ such that
𝑛→∞
𝑛
| √𝑥𝑛 − 𝐿| < 𝑟 − 𝐿, ∀𝑛 ≥ 𝐾
𝑛
⇒ −(𝑟 − 𝐿) < √𝑥𝑛 − 𝐿 < 𝑟 − 𝐿, ∀𝑛 ≥ 𝐾
⇒ 𝑛√𝑥𝑛 − 𝐿 < 𝑟 − 𝐿, ∀𝑛 ≥ 𝐾 ⇒ 𝑛√𝑥𝑛 < 𝑟, ∀𝑛 ≥ 𝐾
⇒ 𝑥𝑛 < 𝑟 𝑛 , ∀𝑛 ≥ 𝐾……①
But 𝑥𝑛 > 0, ∀𝑛 ∈ ℕ ⇒ 0 < 𝑥𝑛 < 𝑟 𝑛 , ∀𝑛 ≥ 𝐾…………②
Since 0 < 𝑟 < 1 we have lim 𝑟 𝑛 = 0
𝑛→∞
①&② ⇒ lim 0 ≤ lim 𝑥𝑛 ≤ lim 𝑟 𝑛 (by the squeeze theorem) ⇒ 0 ≤ lim 𝑥𝑛 ≤ 0
𝑛→∞ 𝑛→∞ 𝑛→∞ 𝑛→∞
⇒ lim 𝑥𝑛 = 0.
𝑛→∞
40
Solving Problems:
sin 𝑛
(Q 3.2.15): Show that the sequence ( ) converges and find its limit.
𝑛
1 sin 𝑛 1
Solution: −1 ≤ sin 𝑛 ≤ 1, ∀𝑛 ∈ ℕ ⇒ −
⏟𝑛 ≤ ⏟ ≤⏟ , ∀𝑛 ∈ ℕ
𝑛 𝑛
𝑥𝑛 𝑦𝑛 𝑧𝑛
1 1
Since lim (− 𝑛) = lim =0
𝑛→∞ 𝑛→∞ 𝑛
sin 𝑛 sin 𝑛
⇒( ) converges and lim = 0 (by the squeeze theorem)
𝑛 𝑛→∞ 𝑛
(−1)𝑛
(Q 3.2.16) Show that the sequence ( ) converges and find its limit.
𝑛
1 (−1)𝑛 1
Solution: −1 ≤ (−1)𝑛 ≤ 1, ∀𝑛 ∈ ℕ ⇒ −
⏟𝑛 ≤ ⏟ ≤⏟ , ∀𝑛 ∈ ℕ
𝑛 𝑛
𝑥𝑛 𝑦𝑛 𝑧𝑛
1 1 (−1)𝑛 (−1)𝑛
Since lim (− 𝑛) = lim =0⇒( ) converges and lim = 0 (by the squeeze theorem)
𝑛→∞ 𝑛→∞ 𝑛 𝑛 𝑛→∞ 𝑛
Note
0 ±∞
(1) Some of the indeterminate Forms are: ∞ − ∞, 0 , ±∞ , 0∞, ∞0 , 00 , 1∞
𝑓(𝑥) 0 ±∞ 𝑓(𝑥) 𝑓 ′ (𝑥)
(2) L’Hospital’s Rule: If lim = 0 or ⇒ lim = lim
𝑛→∞ 𝑔(𝑥) ±∞ 𝑛→∞ 𝑔(𝑥) 𝑛→∞ 𝑔′ (𝑥)
41
2
(Q 3.2.17) Show that the sequence ((𝑛!)1/𝑛 ) converges and find its limit.
Solution: 𝑛! = 1
⏟× 2 × ⋯ × (𝑛 − 1) × 𝑛
𝑛−times
𝑛 × 𝑛 × ⋯ × 𝑛 × 𝑛 = 𝑛𝑛 ⇒ 1 ≤ 𝑛! ≤ 𝑛𝑛
⇒ 1 ≤ 𝑛! ≤ ⏟
𝑛−times
2 2 2 2
⇒ 11/𝑛 ≤ (𝑛!)1/𝑛 ≤ (𝑛𝑛 )1/𝑛 ⇒ 1 ≤ (𝑛!)1/𝑛 ≤ 𝑛1/𝑛 ……….①
lim 1 = 1………②
𝑛→∞
1 ln 𝑛 ln 𝑛 1/𝑛 1
1/𝑛 ln 𝑛𝑛 lim lim lim
lim 𝑛 = lim 𝑒 = lim 𝑒 𝑛 = 𝑒 𝑛→∞ 𝑛 = 𝑒 𝑛→∞ 1 = 𝑒 𝑛→∞𝑛 = 𝑒 0 = 1…③
𝑛→∞ 𝑛→∞ 𝑛→∞
1
1/𝑛2
①&②&③: ⇒ ((𝑛!) ) converges and lim (𝑛!)𝑛2 = 1 (by the squeeze theorem)
𝑛→∞
𝑎𝑛+1 𝑏𝑛+1
𝑎𝑛+1 +𝑏𝑛+1 + 𝑛 𝑎𝑟 𝑛 +𝑏 lim (𝑎𝑟 𝑛 +𝑏) 𝑎×0+𝑏
𝑏𝑛 𝑏 𝑛→∞
lim = lim 𝑛 𝑛 = lim = = =𝑏
𝑛→∞ 𝑎𝑛 +𝑏𝑛 𝑛→∞ 𝑎𝑛 +𝑏𝑛 𝑛→∞ 𝑟 𝑛 +1 lim (𝑟 𝑛 +1)
𝑛→∞
0+1
𝑏 𝑏
𝑛! 𝑛𝑛
(Q 3.2.19) Find lim and lim if exist.
𝑛→∞ 𝑛𝑛 𝑛→∞ 𝑛!
Solution:
𝑛! 𝑛!
lim 𝑛𝑛 : Let 𝑥𝑛 = 𝑛𝑛
𝑛→∞
𝑥𝑛+1 (𝑛+1)! 𝑛𝑛 (𝑛+1)(𝑛!) 𝑛𝑛 𝑛 𝑛 1
⇒ 𝐿 = lim = lim (𝑛+1)𝑛+1
× = lim × = lim (𝑛+1) = 𝑒
𝑛→∞ 𝑥𝑛 𝑛→∞ 𝑛! 𝑛→∞ (𝑛+1)(𝑛+1)𝑛 𝑛! 𝑛→∞
𝑛!
0 ≤ 𝐿 < 1 ⇒ The Ratio Test applies ⇒ lim =0
𝑛→∞ 𝑛𝑛
𝑛! 𝑛!
lim : Let 𝑦𝑛 = 𝑛𝑛
𝑛→∞ 𝑛𝑛
𝑦𝑛+1
𝐿 = lim = 𝑒 ⇒ 𝐿 > 1 ⇒ The Ratio Test applies
𝑛→∞ 𝑦𝑛
𝑛𝑛 𝑛𝑛
⇒ ( 𝑛! ) is unbounded and so lim =∞
𝑛→∞ 𝑛!
42
(Q 3.2.20) Let (𝑥𝑛 ) be a sequence of real numbers. Prove that if lim 𝑥𝑛 = 𝑥, then lim |𝑥𝑛 | = |𝑥|, that
𝑛→∞ 𝑛→∞
is
Proof: Let 𝜀 > 0 be given want to find 𝐾 ∈ ℕ such that ||𝑥𝑛 | − |𝑥|| < 𝜀, ∀𝑛 ≥ 𝐾
Observe that: from the triangle inequality ||𝑥𝑛 | − |𝑥|| ≤ |𝑥𝑛 − 𝑥| ………①
By the definition of lim 𝑥𝑛 = 𝑥: Take 𝜀 > 0 ⇒ ∃𝐾 ∈ ℕ such that |𝑥𝑛 − 𝑥| < 𝜀, ∀𝑛 ≥ 𝐾……..②
Proof: Let 𝜀 > 0 be given want to find 𝐾 ∈ ℕ such that |√𝑥𝑛 − √𝑥. | < 𝜀, ∀𝑛 ≥ 𝐾
By the definition of lim 𝑥𝑛 = 𝑥: Take 𝜀 2 > 0 ⇒ ∃𝐾 ∈ ℕ such that |𝑥𝑛 − 𝑥| < 𝜀 2 , ∀𝑛 ≥ 𝐾…….②
Example 3.3.2:
(1) The following sequences are increasing (and so they are monotone):
(1,2,3, … ), (1,2,2,3,3,3, … ), (𝑎, 𝑎2 , 𝑎3 , … ), where 𝑎 > 1
(2) The following sequences are decreasing (and so they are monotone):
1 1 1 1 1
(1, 2 , 3 , … ) , (1, 2 , 22 , 23 , … ) , (𝑏, 𝑏 2 , 𝑏 3 , … ), where 0 < 𝑏 < 1
(3) The following sequences are not monotone:
1 1
(1, −1,1, −1, … ), (1,2, , 3,4, , … )
2 4
(4) The following sequences are not monotone but they are ultimately monotone:
1 1
(7,6,2,1,2,3, … ) , (−2,0,1, , , … )
⏟
⏟ 2 3
the tail 𝑋 increasing
3
the tail 𝑌2 decreasing
Recall that:
A sequence (𝑥𝑛 ) is called ultimately has a certain property if some tail of (𝑥𝑛 ) has this property.
Remark 3.3.3:
(1) A sequence (𝑥𝑛 ) is increasing iff (−𝑥𝑛 ) is decreasing.
(2) A sequence (𝑥𝑛 ) is decreasing iff (−𝑥𝑛 ) is increasing.
(3) Recall that if 𝑢∗ is a real number, then sup 𝑆 = 𝑢∗ iff the following two conditions hold:
𝐶1 : 𝑢∗ is an UB of 𝑆.
𝐶2′ : if 𝑢 < 𝑢∗ , then 𝑢 is not an UB of 𝑆.
Proof:
Suppose that (𝑥𝑛 ) converges ⇒ (𝑥𝑛 ) is bounded (by Theorem 3.2.2)
Suppose that (𝑥𝑛 ) is monotone and bounded.
Since (𝑥𝑛 ) is monotone ⇒ (𝑥𝑛 ) is increasing or decreasing:
Case 1: Let (𝑥𝑛 ) be increasing ⇒ 𝑥𝑛 ≤ 𝑥𝑛+1 , ∀𝑛 ∈ ℕ
44
Since (𝑥𝑛 ) is bounded, then by the Completeness Theorem we have (𝑥𝑛 ) has a
supremum. So, let sup{𝑥𝑛 : 𝑛 ∈ ℕ} = 𝑥.
By using the squeeze theorem we will show that lim 𝑥𝑛 = 𝑥
𝑛→∞
Since sup{𝑥𝑛 : 𝑛 ∈ ℕ} = 𝑥
Condition 𝐶1 ⇒ 𝑥 is an UB of {𝑥𝑛 : 𝑛 ∈ ℕ} ⇒ 𝑥𝑛 ≤ 𝑥, ∀𝑛 ∈ ℕ
⇒ 𝑥𝑛 − 𝑥 ≤ 0, ∀𝑛 ∈ ℕ…………..①
1
Since 𝑥 − 𝑛 < 𝑥, ∀𝑛 ∈ ℕ and 𝑥 = sup{𝑥𝑛 : 𝑛 ∈ ℕ}. Then:
1
Condition 𝐶2′ ⇒ ∀𝑛 ∈ ℕ, 𝑥 − 𝑛 is not an UB of {𝑥𝑛 : 𝑛 ∈ ℕ}
1
⇒ ∃𝐾 ∈ ℕ such that 𝑥 − 𝑛 < 𝑥𝐾 ……….②
But (𝑥𝑛 ) is increasing ⇒ 𝑥𝐾 ≤ 𝑥𝑛 , ∀𝑛 ≥ 𝐾……..③
1 1
②&③: ⇒ 𝑥 − 𝑛 < 𝑥𝑛 , ∀𝑛 ≥ 𝐾 ⇒ − 𝑛 < 𝑥𝑛 − 𝑥, ∀𝑛 ≥ 𝐾………④
1
①&④: ⇒ − 𝑛 < 𝑥𝑛 − 𝑥 < 0, ∀𝑛 ≥ 𝐾…………⑤
1
The squeeze theorem ⑤ ⇒ lim (− 𝑛) ≤ lim (𝑥𝑛 − 𝑥) ≤ 0 ⇒ 0 ≤ lim (𝑥𝑛 − 𝑥) ≤ 0
𝑛→∞ 𝑛→∞ 𝑛→∞
⇒ lim(𝑥𝑛 − 𝑥) = 0 ⇒ lim 𝑥𝑛 = 𝑥 ⇒ (𝑥𝑛 ) converges and lim 𝑥𝑛 = sup{𝑥𝑛 : 𝑛 ∈ ℕ}
Case 2: Let (𝑥𝑛 ) be decreasing ⇒ (−𝑥𝑛 ) is increasing
Since (𝑥𝑛 ) is bounded ⇒ (−𝑥𝑛 ) is bounded
∴ (−𝑥𝑛 ) is increasing bounded sequence
Case 1 ⇒ (−𝑥𝑛 ) converges and lim (−𝑥𝑛 ) = sup{−𝑥𝑛 : 𝑛 ∈ ℕ}
𝑛→∞
⇒ (𝑥𝑛 ) converges and − ( lim 𝑥𝑛 ) = − inf{𝑥𝑛 : 𝑛 ∈ ℕ}
𝑛→∞
⇒ (𝑥𝑛 ) converges and lim 𝑥𝑛 = inf{𝑥𝑛 : 𝑛 ∈ ℕ}
𝑛→∞
Remark 3.3.5: The contraposition of the MCT 3.3.4 is: Let (𝑥𝑛 ) be a monotone sequence. Then
(𝑥𝑛 ) diverges iff (𝑥𝑛 ) is unbounded.
1
Example 3.3.6: Using the MCT 3.3.4, show that the sequence ( ) converges and find its limit.
√𝑛+1
1 1 1 1
Solution: ( )=( , , , … ) ⇒ The sequence is decreasing………①
√𝑛+1 √2 √3 √4
1 1
Also, 0 < ≤ , ∀𝑛 ∈ ℕ ⇒ The sequence is bounded………..②
√𝑛+1 √2
1 1 1 1 1 1
MCT ⇒ ( ) converges and lim = inf { : 𝑛 ∈ ℕ} = inf { , , ,…} = 0
√𝑛+1 𝑛→∞ √𝑛+1 √𝑛+1 √2 √3 √4
⇒ 𝑥 − 1 = √𝑥 − 1 ⇒ (𝑥 − 1) = 𝑥 − 1 ⇒ 𝑥 2 − 2𝑥 + 1 = 𝑥 − 1
2
⇒ 𝑥 2 − 3𝑥 + 2 = 0 ⇒ (𝑥 − 1)(𝑥 − 2) = 0
⇒ 𝑥 = 1 or 𝑥 = 2……….⑤
Observe that: ⏟
2 ≤ 𝑥𝑛 ≤ 𝑥1 , ∀𝑛 ∈ ℕ ⇒ 2 ≤ lim 𝑥𝑛 ≤ 𝑥1
𝑛→∞
the study of boundedness
⇒ 2 ≤ 𝑥 ≤ 𝑥1 ………….⑥
⑤&⑥: ⇒ 𝑥 = 2 ⇒ lim 𝑥𝑛 = 2
𝑛→∞
Remark 3.3.9: There are other examples in the text book, the students should study them.
Definition 3.4.1: Let 𝑋 = (𝑥𝑛 ) be a sequence and (𝑛1 , 𝑛2 , 𝑛3 , … ) be a sequence of natural numbers
such that 𝑛1 < 𝑛2 < 𝑛3 < ⋯. The sequence 𝑋 ′ = (𝑥𝑛𝑘 ) = (𝑥𝑛1 , 𝑥𝑛2 , 𝑥𝑛3 , … ) is
called a subsequence of 𝑋 = (𝑥𝑛 ).
𝑋 𝑋 هي حدود في المتتالية′ ) من المتتالية 𝑋 إذا كانت جميع حدودsubsequence( 𝑋 متتالية جزئية′ تكون المتتالية
.مع المحافظة على الترتيب
1 1 1 1 1 1 1 1
Example 3.4.2: Let 𝑋 = (𝑛) = (1, 2 , 3 , 4 , … ) ⇒ 𝑥𝑛 = 𝑛 ⇒ 𝑥1 = 1, 𝑥2 = 2 , 𝑥3 = 3 , 𝑥4 = 4,…
𝑥2 𝑥 4 𝑥6
1 ⏞1 ⏞ 1 ⏞ 1
(1) (2𝑛) = ( 2 , 4 , 6 , … ) is a subsequence of 𝑋.
𝑥3 𝑥5 𝑥2 𝑥6 𝑥24
𝑥1
⏞
1 ⏞ 1 ⏞
1 ⏞1 ⏞ 1
(2) The following are subsequences of 𝑋: ( ⏞
1 , 3 , 5 , … ) , ( 2! , 3! , ,…)
4!
𝑥2
𝑥1
⏞
1 1
1
⏞ 1 1
(3) The following are not subsequences of 𝑋: (0,
⏟ 1, 3 , 5 , … ) , ( 2 , 1 , 4 , 3 , … )
0 is not a term in 𝑋 ⏟
The ordering is
different than that of 𝑋
Remark 3.4.3:
(1) Every sequence is a subsequence of itself.
Proof: Let 𝑋 = (𝑥𝑛 ) = (𝑥1 , 𝑥2 , 𝑥3 , … ) and take the natural numbers:
𝑛1 = 1, 𝑛2 = 2, 𝑛3 = 3,…,𝑛𝑘 = 𝑘,… ⇒ 𝑛1 < 𝑛2 < 𝑛3 < ⋯.
So, 𝑋 ′ = (𝑥𝑛1 , 𝑥𝑛2 , 𝑥𝑛3 , … ) is a subsequence of 𝑋. But 𝑋 ′ = (𝑥1 , 𝑥2 , 𝑥3 , … ) ⇒ 𝑋 ′ = 𝑋.
Recall that
(𝑥𝑛 ) converges to 𝑥 iff any tail 𝑋𝑚 = (𝑥𝑚+𝑛 ) converges to
that is: lim 𝑥𝑛 = 𝑥 ⇔ lim 𝑥𝑚+𝑛 = 𝑥
𝑛→∞ 𝑛→∞
Example 3.4.7:
(1) Show that the sequence (sin(𝑛)) diverges.
(2) Show that the sequence (cos(𝑛)) diverges.
To have a contradiction, we will construct two subsequences of (sin(𝑛)) with different limits
as follows:
𝜋 5𝜋
Let 𝐼𝑘 = [ 6 + 2(𝑘 − 1)𝜋, 6 + 2(𝑘 − 1)𝜋] , 𝑘 ∈ ℕ
4𝜋 2𝜋
⇒ Length of 𝐼𝑘 = 6 = 3 > 2, ∀𝑘 ∈ ℕ
⇒ ∃𝑛𝑘 ∈ ℕ such that 𝑛𝑘 ∈ 𝐼𝑘 , ∀𝑘 ∈ ℕ
⇒ (sin(𝑛𝑘 )) is a subsequence of (sin(𝑛)) and by ①⇒ lim sin(𝑛𝑘 ) = 𝑥 ………②
𝑘→∞
1
Since ≤ sin(𝑛𝑘 ) ≤ 1, ∀𝑘 ∈ ℕ
2
1 1
⇒ 2 ≤ lim sin(𝑛𝑘 ) ≤ 1 and by ②: 2 ≤ 𝑥 ≤ 1 ………③
𝑘→∞
7𝜋 11𝜋
Let 𝐽𝑘 = [ + 2(𝑘 − 1)𝜋, + 2(𝑘 − 1)𝜋] , 𝑘 ∈ ℕ
6 6
4𝜋 2𝜋
⇒ Length of 𝐽𝑘 = 6 = 3 > 2, ∀𝑘 ∈ ℕ
⇒ ∃𝑚𝑘 ∈ ℕ such that 𝑚𝑘 ∈ 𝐽𝑘 , ∀𝑘 ∈ ℕ
⇒ (sin(𝑚𝑘 )) is a subsequence of (sin(𝑛)) and by ①: lim sin(𝑚𝑘 ) = 𝑥 ………④
𝑘→∞
1
Since −1 ≤ sin(𝑚𝑘 ) ≤ − 2 , ∀𝑘 ∈ ℕ
1 1
⇒−1 ≤ lim sin(𝑚𝑘 ) ≤ − 2 and by ②: −1 ≤ 𝑥 ≤ − 2 ………⑤
𝑘→∞
③&⑤:
1 1
⇒ 2 ≤ 𝑥 ≤ 1 and −1 ≤ 𝑥 ≤ − 2 which is impossible (since the limit of a sequence is unique)
⇒ We have a contradiction ⇒ Our assumption is false
⇒ (sin(𝑛)) diverges.
51
Example 3.4.8: Let 𝑋 = (𝑥𝑛 ) be a sequence of real numbers. There are two important subsequences
of 𝑋 which are (𝑥2𝑛 ) = (𝑥2 , 𝑥4 , 𝑥6 , … ) and (𝑥2𝑛−1 ) = (𝑥1 , 𝑥3 , 𝑥5 , … ).
Theorem 3.4.9: A sequence (𝑥𝑛 ) converges to 𝑥 iff [(𝑥2𝑛 ) and (𝑥2𝑛−1 ) both converge to 𝑥].
That is lim 𝑥𝑛 = 𝑥 iff [ lim 𝑥2𝑛 = 𝑥 and lim 𝑥2𝑛−1 = 𝑥]
𝑛→∞ 𝑛→∞ 𝑛→∞
Proof (Optional):
Suppose that (𝑥𝑛 ) converges to 𝑥 ⇒ All subsequences of 𝑋 converges to 𝑥 (by Theorem 3.4.4)
⇒ 𝑋 ′ = (𝑥2𝑛 ) and 𝑋 ′′ = (𝑥2𝑛−1 ) both converges to 𝑥
Suppose that both 𝑋 ′ = (𝑥2𝑛 ) and 𝑋 ′′ = (𝑥2𝑛−1 ) converges to 𝑥
To show that 𝑋 = (𝑥𝑛 ) converges to 𝑥 we use definition:
Let 𝜀 > 0 be given. Want to show that ∃𝐾 ∈ ℕ such that |𝑥𝑛 − 𝑥| < 𝜀, ∀𝑛 ≥ 𝐾
lim 𝑥2𝑛 = 𝑥 ⇒ ∃𝐾1 ∈ ℕ such that |𝑥2𝑛 − 𝑥| < 𝜀, ∀𝑛 ≥ 𝐾1
lim 𝑥2𝑛−1 = 𝑥 ⇒ ∃𝐾2 ∈ ℕ such that |𝑥2𝑛−1 − 𝑥| < 𝜀, ∀𝑛 ≥ 𝐾2
Let 𝐾 = max(2𝐾1 , 2𝐾2 − 1).
If 𝑛 ≥ 𝐾 ⇒ 𝑛 ≥ 2𝐾1 & 𝑛 ≥ 2𝐾2 − 1
But 𝑛 is even or 𝑛 is odd ⇒ 𝑛 = 2𝑘1 or 𝑛 = 2𝑘1 − 1 for some 𝑘1 , 𝑘2 ∈ ℕ
If 𝑛 = 2𝑘1 ⇒ 2𝑘1 ≥ 2𝐾1 ⇒ 𝑘1 ≥ 𝐾1 ⇒ |𝑥2𝑘1 − 𝑥| < 𝜀
⇒ |𝑥𝑛 − 𝑥| < 𝜀
If 𝑛 = 2𝑘2 − 1 ⇒ 2𝑘2 − 1 ≥ 2𝐾2 − 1 ⇒ 𝑘2 ≥ 𝐾2
⇒ |𝑥2𝑘2 −1 − 𝑥| < 𝜀 ⇒ |𝑥𝑛 − 𝑥| < 𝜀
In either case we have |𝑥𝑛 − 𝑥| < 𝜀, ∀𝑛 ≥ 𝐾 ⇒ (𝑥𝑛 ) converges to 𝑥.
Corollary 3.4.10:
(1) A sequence (𝑥𝑛 ) diverges iff [(𝑥2𝑛 ) or (𝑥2𝑛−1 ) diverges].
(2) A sequence (𝑥𝑛 ) diverges iff lim 𝑥2𝑛 ≠ lim 𝑥2𝑛−1.
𝑛→∞ 𝑛→∞
Monotone Subsequence Theorem (MST) 3.4.11: Every sequence has a monotone subsequence.
lim 𝑥𝑛 = sup inf 𝑥𝑚 that is lim 𝑥𝑛 = sup inf {𝑥𝑛 , 𝑥𝑛+1 , 𝑥𝑛+2 , … }
𝑛→∞ 𝑛∈ℕ 𝑚≥𝑛 𝑛→∞ 𝑛∈ℕ
52
(2) The limit superior of (𝑥𝑛 ) (denoted by lim 𝑥𝑛 or lim sup 𝑥𝑛 ) is defined by
𝑛→∞ 𝑛→∞
lim 𝑥𝑛 = inf sup 𝑥𝑚 that is lim 𝑥𝑛 = inf sup {𝑥𝑛 , 𝑥𝑛+1 , 𝑥𝑛+2 , … }
𝑛→∞ 𝑛∈ℕ 𝑚≥𝑛 𝑛→∞ 𝑛∈ℕ
Remark 3.4.15: Let 𝑋 = (𝑥𝑛 ) be a sequence and let 𝑋𝑚 = (𝑥𝑚+𝑛 ) be the 𝑚-tail of 𝑋,𝑚 = 0,1,2, …,
where 𝑋0 = 𝑋. Then
lim 𝑥𝑛 = sup {inf 𝑋0 , inf 𝑋1 , inf 𝑋2 , … }
𝑛→∞
lim 𝑥𝑛 = inf {sup 𝑋0 , sup 𝑋1 , sup 𝑋2 , … }
𝑛→∞
Theorem 3.4.16: Let (𝑥𝑛 ) be a bounded sequence of real numbers and let 𝑆 be the set of all
subsequential limits of (𝑥𝑛 ). Then
(𝑥𝑛 ) converges to 𝑥 iff lim 𝑥𝑛 = lim 𝑥𝑛 = 𝑥.
𝑛→∞ 𝑛→∞
1 1 1 1 1 1
Example 3.4.17: Let (𝑥𝑛 ) = (−1,1, − 2 , 2 , − 3 , 3 , − 4 , 4 , … )
(1) Find lim 𝑥𝑛 and lim 𝑥𝑛
𝑛→∞ 𝑛→∞
(2) Using part (1) determine whether the sequence (𝑥𝑛 ) converges or diverges. If it converges
find its limit.
Solution:
(1)
inf 𝑥𝑚 = inf 𝑋𝑚 sup 𝑥𝑚 = sup 𝑋𝑚
𝑚≥𝑛 𝑚≥𝑛
1 1 1 1 1 1
𝑋0 = (−1,1, − , , − , , − , , … ) −1 1
2 2 3 3 4 4
1 1 1 1 1 1 1
𝑋1 = (1, − , , − , , − , , … ) − 1
2 2 3 3 4 4 2
1 1 1 1 1 1 1 1
𝑋2 = (− , , − , , − , , … ) −
2 2 3 3 4 4 2 2
1 1 1 1 1 1 1
𝑋3 = ( , − , , − , , … ) −
2 3 3 4 4 3 2
1 1 1 1 1 1
𝑋4 = (− , , − , , … ) −
3 3 4 4 3 3
1 1 1 1 1
𝑋5 = ( , − , , … ) −
3 4 4 4 3
1 1 1 1
lim 𝑥𝑛 = sup {inf 𝑋0 , inf 𝑋1 , inf 𝑋2 , … } = sup {−1, − , − , − , − , … } = 0
𝑛→∞ 2 2 3 3
1 1 1 1
lim 𝑥𝑛 = inf {sup 𝑋0 , sup 𝑋1 , sup 𝑋2 , … } = inf {1,1, , , , , … } = 0
𝑛→∞ 2 2 3 3
(2) Since lim 𝑥𝑛 = lim 𝑥𝑛 = 0 ⇒ The sequence (𝑥𝑛 ) converges to 0, that .
𝑛→∞ 𝑛→∞
53
1 1 1
Example 3.4.18: Let (𝑥𝑛 ) = (−1,1, −1, 2 , −1, 3 , −1, 4 , … )
(1) Find lim 𝑥𝑛 and lim 𝑥𝑛
𝑛→∞ 𝑛→∞
(2) Use part (1) to show that the sequence (𝑥𝑛 ) diverges.
Solution:
(3)
inf 𝑥𝑚 = inf 𝑋𝑚 sup 𝑥𝑚 = sup 𝑋𝑚
𝑚≥𝑛 𝑚≥𝑛
1 1 1 1 −1 1
𝑋0 = (−1,1, −1, , −1, , −1, , −1, , … )
2 3 4 5
1 1 1 1 −1 1
𝑋1 = (1, −1, , −1, , −1, , −1, , … )
2 3 4 5
1 1 1 1 1
𝑋2 = (−1, , −1, , −1, , −1, , … ) −1
2 3 4 5 2
1 1 1 1 1
𝑋3 = ( , −1, , −1, , −1, , … ) −1
2 3 4 5 2
1 1 1 1
𝑋4 = (−1, , −1, , −1, , … ) −1
3 4 5 3
1 1 1 1
𝑋5 = ( , −1, , −1, , … ) −1
3 4 5 3
Example 3.4.19:
(1) Find lim (−1)𝑛 and lim (−1)𝑛
𝑛→∞ 𝑛→∞
(2) Use part (1) to determine to show that the sequence ((−1)𝑛 ) diverges.
Solution:
(1)
inf 𝑥𝑚 = inf 𝑋𝑚 sup 𝑥𝑚 = sup 𝑋𝑚
𝑚≥𝑛 𝑚≥𝑛
2𝑛2 +1 2𝑛2 +1
Example 3.4.20: Find lim and lim .
𝑛→∞ 𝑛2 +𝑛−1 𝑛2 +𝑛−1
𝑛→∞
2𝑛2 +1 4𝑛 4
Solution: Since lim = lim = lim = 2 we have:
𝑛→∞ 𝑛2 +𝑛−1 𝑛→∞ 2𝑛+1 𝑛→∞ 2
2𝑛2 +1
The sequence (𝑛2 +𝑛−1) converges to 2 and so
∀𝜀 > 0, ⏞
if ⏞ ∃𝐻(𝜀) ∈ ℕ such that ⏞|𝑥𝑚 − 𝑥𝑛 | < 𝜀, ∀𝑚 > 𝑛 ≥ 𝐻(𝜀), where 𝑚, 𝑛 ∈ ℕ.
2) A sequence 𝑋 = (𝑥𝑛 ) of real numbers is not a Cauchy sequence
① ② ③
if ⏞ ∀𝐻(𝜀) ∈ ℕ and ⏞
∃𝜀0 > 0, ⏞ ∃𝑚0 > 𝑛0 ≥ 𝐻(𝜀) such that |𝑥𝑚0 − 𝑥𝑛0 | ≥ 𝜀0 .
Note: The value of the natural number 𝐻(𝜀) is usually depends on the value of 𝜀. For simplicity,
we will write 𝐻 instead of writing 𝐻(𝜀)
Example 3.5.2: Using the definition of Cauchy sequences, show that the sequence
1
𝑋 = (𝑛) is a Cauchy sequence.
1
Solution: Let 𝜀 > 0 be given and let 𝑥𝑛 = 𝑛.
1 1 𝜀 1 𝜀
Since lim 𝑛 = 0: ∃𝐾 ∈ ℕ such that |𝑛| < 2 , ∀𝑛 ≥ 𝐾 ⇒ 𝑛 < 2 , ∀𝑛 ≥ 𝐾 ……….②
Example 3.5.3: Using the definition of Cauchy sequences, show that the sequence
𝑋 = ((−1)𝑛 ) is not a Cauchy sequence.
Solution: Let 𝜀0 = 1 > 0 be given and 𝐻 ∈ ℕ. Want to find
𝑚0 > 𝑛0 ≥ 𝐻 such that |(−1)𝑚0 − (−1)𝑛0 | ≥ 𝜀0 = 1
Take 𝑚0 = 2𝐻 and 𝑛0 = 2𝐻 − 1 ⇒ 𝑚0 > 𝑛0 ≥ 𝐻 and
|(−1)𝑚0 − (−1)𝑛0 | = |(−1)2𝐻 − (−1)2𝐻−1 | = |1 − (−1)| = 2 > 1 = 𝜀0
⇒ ∃𝑚0 > 𝑛0 ≥ 𝐻 such that |(−1)𝑚0 − (−1)𝑛0 | ≥ 𝜀0
∴ 𝑋 = ((−1)𝑛 ) is not a Cauchy sequence.
56
Remark 3.5.4:
(1) A sequence (𝑥𝑛 ) is bounded iff ∃𝑀 > 0 such that |𝑥𝑚 | ≤ 𝑀, ∀𝑚 ∈ ℕ.
(2) The triangle inequality: |𝑎 − 𝑏| ≤ |𝑎| + |𝑏|
Suppose that (𝑥𝑛 ) is a Cauchy sequence. Want to show that (𝑥𝑛 ) converges.
Let (𝑥𝑛 ) be a Cauchy sequence ⇒ (𝑥𝑛 ) is bounded (by Theorem 4)
Bolzano-Weierstrass Theorem ⇒ 𝑋 = (𝑥𝑛 ) has a convergent subsequence 𝑋 ′ = (𝑥𝑛𝑘 )
that is lim 𝑥𝑛𝑘 = 𝑥 ⇒ Now, we show that lim 𝑥𝑛 = 𝑥
𝑘→∞ 𝑛→∞
Let 𝜀 > 0 be given.
Since (𝑥𝑛 ) is Cauchy seq.: ∃𝐻 ∈ ℕ such that
𝜀
|𝑥𝑚 − 𝑥𝑛 | < , ∀𝑚 > 𝑛 ≥ 𝐻………④
3
𝜀
Since lim 𝑥𝑛𝑘 = 𝑥: ⇒ ∃𝐾1 ∈ ℕ such that |𝑥𝑛𝑘 − 𝑥| < 3 , ∀𝑘 ≥ 𝐾1 …..⑤
𝑘→∞
Let 𝐾 = max{𝐻, 𝐾1 } and let 𝑚 ≥ 𝐾 ⇒ 𝑚 ≥ 𝐻 and 𝑚 ≥ 𝐾1
𝜀
In ④: Take 𝑛 = 𝐾⇒ |𝑥𝑚 − 𝑥𝐾 | < 3……….⑥
𝜀
In ⑤: Take 𝑘 = 𝐾⇒ |𝑥𝑛𝐾 − 𝑥| < 3………⑦
Also, since 𝑛𝐾 ≥ 𝐾 so in ④: taking 𝑚 = 𝑛𝐾 and 𝑛 = 𝐾 we have:
𝜀
|𝑥𝑛𝐾 − 𝑥𝐾 | < 3 ………⑧
Now,
|𝑥𝑚 − 𝑥| = |𝑥𝑚 − 𝑥𝐾 + 𝑥𝐾 − 𝑥𝑛𝐾 + 𝑥𝑛𝐾 − 𝑥|
⏟ |𝑥𝑚 − 𝑥𝐾 | + |𝑥𝐾 − 𝑥𝑛𝐾 | + |𝑥𝑛𝐾 − 𝑥|
≤
by the triangle inequality
= |𝑥𝑚 − 𝑥𝐾 | + ⏟
|𝑥𝑛𝐾 − 𝑥𝐾 | + |𝑥𝑛𝐾 − 𝑥|
since |𝑎|=|−𝑎|
𝜀 𝜀 𝜀
< + +
⏟
3 ⏟
3 ⏟
3
by ⑥ by ⑧ by ⑦
=𝜀
⇒ |𝑥𝑚 − 𝑥| < 𝜀, ∀𝑚 ≥ 𝐾 ⇒ 𝑋 converges.
Solution:
1) ((−1)𝑛 ) diverges ⇒ ((−1)𝑛 ) is not a Cauchy sequence.
𝑛 𝑛
2) (𝑛+1) converges to 1 ⇒ (𝑛+1) is a Cauchy sequence.
2 3𝑛 2 3𝑛
3) ((1 − 𝑛) ) converges to 𝑒 −6 ⇒ ((1 − 𝑛) ) is a Cauchy sequence.
4) (−√𝑛) diverges ⇒ (−√𝑛) is not a Cauchy sequence.
Definition 3.5.9: Let 𝑋 = (𝑥𝑛 ) be a sequence of real numbers. We say that a sequence 𝑋 of real
numbers is contractive if ∃𝐶 ∈ (0,1) such that
|𝑥𝑛+2 − 𝑥𝑛+1 | ≤ 𝐶|𝑥𝑛+1 − 𝑥𝑛 |, ∀𝑛 ∈ ℕ.
The number 𝐶 is called the constant of the contractive sequence.
58
Theorem 3.5.10: Every contractive sequence is a Cauchy sequence, and therefore is convergent.
Solution:
𝑥 +𝑥
1) |𝑥𝑛+2 − 𝑥𝑛+1 | = | 𝑛+12 𝑛 − 𝑥𝑛+1 |
𝑥𝑛+1 + 𝑥𝑛 2𝑥𝑛+1
=| − |
2 2
𝑥𝑛 − 𝑥𝑛+1 1
=| | = |𝑥𝑛+1 − 𝑥𝑛 |, ∀𝑛 ∈ ℕ
2 2
1
(𝑥𝑛 ) is contractive with 𝐶 = 2 ∈ (0,1).
1
(1) Show that 𝑥𝑛+1 = 1+𝑥 , 𝑛 ∈ ℕ.
𝑛
1
(2) Show that 2 ≤ 𝑥𝑛 ≤ 1, 𝑛 ∈ ℕ.
(3) Show that (𝑥𝑛 ) is contractive.
(4) Show that (𝑥𝑛 ) converges
(5) Find lim 𝑥𝑛 .
𝑛→∞
Solution:
𝑓𝑛+1
𝑓𝑛+1 𝑓𝑛+1 𝑓𝑛+1 1
1) 𝑥𝑛+1 = 𝑓 =𝑓 = 𝑓𝑛+1 𝑓 = 1+𝑥 , 𝑛 ∈ ℕ
𝑛+2 𝑛+1 +𝑓𝑛 + 𝑛 𝑛
𝑓𝑛+1 𝑓𝑛+1
1
2) To show that 2 ≤ 𝑥𝑛 ≤ 1, 𝑛 ∈ ℕ, we use the proof by induction:
1
Base step: At 𝑛 = 1: 𝑥1 = 1 ⇒ 2 ≤ 𝑥1 ≤ 1 is true
Inductive Step:
Suppose that the inequalities is true at 𝑛 = 𝑘 that is
1
≤ 𝑥𝑘 ≤ 1………..①
2
1
At 𝑛 = 𝑘 + 1: 𝑥𝑘+1 = 1+𝑥 (by part (1))
𝑘
⇒ 𝑥𝑘+1 < 1……..②
1 1 1
Also, 𝑥𝑘 ≤ 1 ⇒ 1 + 𝑥𝑘 ≤ 2 ⇒ 1+𝑥 ≥ 2 ⇒ 𝑥𝑘+1 ≥ 2………③
𝑘
1
②&③ ⇒ 2 ≤ 𝑥𝑘+1 ≤ 1 ⇒ the inequalities is true at 𝑛 = 𝑘 + 1
1
∴ 2 ≤ 𝑥𝑛 ≤ 1, 𝑛 ∈ ℕ
1 1 1+𝑥 −(1+𝑥 )
3) |𝑥𝑛+2 − 𝑥𝑛+1 | = |1+𝑥 − 1+𝑥 | = |(1+𝑥𝑛 )(1+𝑥
𝑛+1
|
𝑛+1 𝑛 𝑛+1 𝑛)
𝑥𝑛 − 𝑥𝑛+1
=| |
(1 + 𝑥𝑛+1 )(1 + 𝑥𝑛 )
|𝑥𝑛 −𝑥𝑛+1 | |𝑥𝑛+1 −𝑥𝑛 |
= = , ∀𝑛 ∈ ℕ…….①
(1+𝑥𝑛+1 )(1+𝑥𝑛 ) (1+𝑥𝑛+1 )(1+𝑥𝑛 )
1 3
Part (2) ⇒ 𝑥𝑛 ≥ 2 , ∀𝑛 ∈ ℕ ⇒ 1 + 𝑥𝑛 ≥ 2 , ∀𝑛 ∈ ℕ
1 2
⇒ 1+𝑥 ≤ 3 , ∀𝑛 ∈ ℕ……..②
𝑛
1 1
①&② ⇒ |𝑥𝑛+2 − 𝑥𝑛+1 | = 1+𝑥 |𝑥𝑛+1 − 𝑥𝑛 |
𝑛+1 1+𝑥𝑛
22
≤ |𝑥 − 𝑥𝑛 |
3 3 𝑛+1
4
= |𝑥 − 𝑥𝑛 |, ∀𝑛 ∈ ℕ
9 𝑛+1
4
(𝑥𝑛 ) is contractive with 𝐶 = 9 ∈ (0,1).
2 −1±√1−4(1)(−1) −1±√5
⇒ 𝑥(1 + 𝑥) = 1 ⇒ 𝑥 + 𝑥 − 1 = 0 ⇒ 𝑥 = =
2 2
−1+√5 −1−√5
⇒𝑥= or 𝑥 =
2 2
1 1 1
But 2 ≤ 𝑥𝑛 ≤ 1, 𝑛 ∈ ℕ ⇒ ≤ lim 𝑥𝑛 ≤ 1, 𝑛 ∈ ℕ ⇒ ≤ 𝑥 ≤ 1, 𝑛 ∈ ℕ
2 2
−1+√5
∴𝑥= 2
Nice videos that talk about relation between the Fibonacci sequence and the Golden ratio and the
golden spiral are with the following links:
1) https://youtu.be/qTw_qay54WI
2) https://youtu.be/SjSHVDfXHQ4
(1) ⏞ > 0, ⏞
We say that (𝑥𝑛 ) tends to +∞, written as lim 𝑥𝑛 = +∞ , if 𝛼 ∃𝐾(𝛼) ∈ ℕ such
𝑛→∞
③
that ⏞
𝑥𝑛 > 𝛼, ∀𝑛 ≥ 𝐾(𝛼).
① ②
(2) ⏞ > 0, ⏞
We say that (𝑥𝑛 ) tends to −∞, written as lim 𝑥𝑛 = −∞ , if 𝛽 ∃𝐾(𝛽) ∈ ℕ such
𝑛→∞
③
that ⏞
𝑥𝑛 < 𝛽, ∀𝑛 ≥ 𝐾(𝛽).
(3) We say that (𝑥𝑛 ) is properly divergent if either: lim 𝑥𝑛 = +∞ or lim 𝑥𝑛 = −∞.
𝑛→∞ 𝑛→∞
Remark 3.6.2:
1) lim 𝑥𝑛 = −∞ iff lim (−𝑥𝑛 ) = +∞
𝑛→∞ 𝑛→∞
So, we will focus our study on sequences (𝑥𝑛 ) with lim 𝑥𝑛 = +∞
𝑛→∞
2) The value of the natural number 𝐾(𝛼) is usually depends on the value of 𝛼. For
simplicity, we will write 𝐾 instead of writing 𝐾(𝛼).
3) To find the value of 𝐾 ∈ ℕ, we start as follows:
Let 𝛼 > 0 be given
Study 𝑥𝑛 by: 𝑥𝑛 > ⋯ > 𝑎𝑛𝑏 or 𝑎(𝑏 𝑛 ) with 𝑎, 𝑏 > 0 ()نقوم بعمل تصغير للمقدار
Take 𝑎(𝑏 𝑛 ) > 𝛼 or 𝑎𝑛𝑏 > 𝛼 ⇒ 𝑛 > 𝑀
Apply the Archimedean Property: ∃𝐾 ∈ ℕ such that 𝐾 > 𝑀
⇒ 𝑥𝑛 > 𝛼 ∀𝑛 ≥ 𝐾
Example 3.6.3:
7𝑛4 −3𝑛+1
(1) Using definition, show that lim = ∞.
𝑛→∞ 𝑛+1
(2) Based on your calculations in Part (1), Find the smallest value of 𝐾 that satisfies the
7𝑛4 −3𝑛+1
definition of lim = +∞ by taking 𝛼 = 54.
𝑛→∞ 𝑛+1
Solution:
7𝑛4 −3𝑛+1
(1) Let 𝛼 > 0 be given. Let 𝑥𝑛 = want to show that: 𝑥𝑛 > ⋯ > 𝑎𝑛𝑏 :
𝑛+1
⇒ 𝑥𝑛 ≥ 2𝑛3 ……..①
3 𝛼
Take 2𝑛3 > 𝛼 ⇔ 𝑛 > √ 2 ………②
62
3 𝛼
By the Archimedean Property: ∃𝐾 ∈ ℕ such that 𝐾 > √ 2 .
3 𝛼
If 𝑛 ≥ 𝐾 ⇒ 𝑛 > √ 2 ⇒ 2𝑛3 > 𝛼 (by ②) ⇒ 𝑥𝑛 > 𝛼 (by ①)
⇒ 𝑥𝑛 > 𝛼 ∀𝑛 ≥ 𝐾
7𝑛4 −3𝑛+1
⇒ lim = +∞.
𝑛→∞ 𝑛+1
3 𝛼 3 54
(2) By Part (1), 𝐾 ∈ ℕ such that 𝐾 > √ 2 ⇒ 𝐾 > √ 2 = 3 ⇒ 𝐾 > 3
⇒ The smallest value of 𝐾 is 𝐾 = 4.
Example 3.6.4:
(1) Using definition, show that lim 𝑛2 = ∞.
𝑛→∞
(2) Based on your calculations in Part (1), Find the smallest value of 𝐾 that satisfies the
definition of lim 𝑛2 = +∞ by taking 𝛼 = 16.
𝑛→∞
Solution:
(1) Let 𝛼 > 0 be given. Let 𝑥𝑛 = 𝑛2 want to show that: 𝑥𝑛 > ⋯ > 𝑎𝑛𝑏 :
𝑥𝑛 = 𝑛2 ……..①
ln 𝛼
If 𝑛 ≥ 𝐾 ⇒ 𝑛 > ⇒ 𝑥𝑛 > 𝛼 (by ①) ⇒ 𝑥𝑛 > 𝛼 ∀𝑛 ≥ 𝐾
ln 𝑐
⇒ lim 𝑐 𝑛 = ∞.
𝑛→∞
63
Theorem 3.6.9: Let (𝑥𝑛 ) and (𝑦𝑛 ) be a sequences of nonzero real numbers such that (𝑥𝑛 ) is
properly divergent and lim 𝑥𝑛 𝑦𝑛 = 𝐿 ∈ ℝ. Then lim 𝑦𝑛 = 0
𝑛→∞ 𝑛→∞
Comparison Test 3.6.10: Let (𝑥𝑛 ) and (𝑦𝑛 ) be two sequences of real numbers such that
𝑥𝑛 ≤ 𝑦𝑛 , ∀𝑛 ≥ 𝐾.
(1) If lim 𝑥𝑛 = +∞, then lim 𝑦𝑛 = +∞
𝑛→∞ 𝑛→∞
(2) If lim 𝑦𝑛 = −∞, then lim 𝑥𝑛 = −∞
𝑛→∞ 𝑛→∞
Proof:
(1) Suppose that lim 𝑥𝑛 = +∞. Want to show that lim 𝑦𝑛 = +∞ by using definition:
𝑛→∞ 𝑛→∞
Let 𝛼 > 0 be given
Since lim 𝑥𝑛 = +∞ ⇒ ∃𝐾 ∈ ℕ such that 𝑥𝑛 > 𝛼, ∀𝑛 ≥ 𝐾…①
𝑛→∞
But 𝑦𝑛 ≥ 𝑥𝑛 , ∀𝑛 ∈ ℕ ⇒ ⏟
𝑦𝑛 > 𝛼 , ∀𝑛 ≥ 𝐾
by ①
∴ lim 𝑦𝑛 = +∞
𝑛→∞
(2) Let lim 𝑦𝑛 = −∞ ⇒ lim (−𝑦𝑛 ) = +∞……….②
𝑛→∞ 𝑛→∞
𝑥𝑛 ≤ 𝑦𝑛 , ∀𝑛 ∈ ℕ ⇒ −𝑦𝑛 ≤ −𝑥𝑛 , ∀𝑛 ∈ ℕ……..③
Part (1) and ②+③: ⇒ lim (−𝑥𝑛 ) = +∞ ⇒ lim 𝑥𝑛 = −∞
𝑛→∞ 𝑛→∞
65
(3)
𝑥𝑛
(a) Suppose that lim = +∞ and lim 𝑥𝑛 = 0
𝑛→∞ 𝑦𝑛
1 1
⇒ lim = 0 and lim = +∞ (by Theorem 3.6.7)
𝑛→∞ 𝑥𝑛 /𝑦𝑛 𝑛→∞ 𝑥𝑛
1/𝑥𝑛 1
⇒ lim = 0 and lim = +∞
𝑛→∞ 1/𝑦𝑛 𝑛→∞ 𝑥𝑛
1
Part (2) (a) ⇒ lim = +∞ ⇒ lim 𝑦𝑛 = 0 (by Theorem 3.6.7)
𝑛→∞ 𝑦𝑛 𝑛→∞
𝑥𝑛
(b) Suppose that lim = +∞ and (𝑥𝑛 ) is bounded
𝑛→∞ 𝑦𝑛
𝑦𝑛
Theorem 4 ⇒ lim = 0 and (𝑥𝑛 ) is bounded
𝑛→∞ 𝑥𝑛
Part (2)(b) ⇒ lim 𝑦𝑛 = 0
𝑛→∞
Solution:
𝑥𝑛
(1) Since lim = 𝐿 with 0 < 𝐿 < ∞, then we apply The Limit Comparison Test Part (1). Let
𝑛→∞ 𝑛
𝑦𝑛 = 𝑛 we have lim 𝑦𝑛 = lim 𝑛 = +∞ ⇒ lim 𝑥𝑛 = +∞
𝑛→∞ 𝑛→∞ 𝑛→∞
𝑥𝑛 𝑛 𝑥 |𝑥𝑛 | 𝑥𝑛
(2) lim = 0 ⇒ lim |sin(𝑛)| = 0 ⇒ lim = 0 ⇒ lim =0
𝑛→∞ sin(𝑛) 𝑛→∞ 𝑛→∞ |sin(𝑛)| 𝑛→∞ |sin(𝑛)|
Now, we apply The Limit Comparison Test Part (2):
Let 𝑦𝑛 = |sin(𝑛)| ⇒ (𝑦𝑛 ) is bounded (since 0 ≤ |sin(𝑛)| ≤ 1)
⇒ lim 𝑥𝑛 = 0
𝑛→∞
|cos(𝑛)| 𝑥𝑛
(3) Since lim = +∞ ⇒ lim |cos(𝑛)|
= 0, then we apply The Limit Comparison Test Part
𝑛→∞ 𝑥𝑛 𝑛→∞
(2): Let 𝑦𝑛 = |cos(𝑛)|
⇒ (𝑦𝑛 ) is bounded (since 0 ≤ |cos(𝑛)| ≤ 1)
⇒ lim 𝑥𝑛 = 0.
𝑛→∞
1
|sin( )|
𝑛
(4) Since lim = +∞, then we apply The Limit Comparison Test Part (3): Let 𝑦𝑛 =
𝑛→∞ 𝑥𝑛
1 1
|sin(𝑛)| we have lim 𝑦𝑛 = lim |sin(𝑛)| = 0 ⇒ lim 𝑥𝑛 = 0
𝑛→∞ 𝑛→∞ 𝑛→∞
1
|sin( )| 𝑥𝑛
𝑛
Another Solution for Part (4): lim = +∞ ⇒ lim 1 =0
𝑛→∞ 𝑥𝑛 𝑛→∞ |sin(𝑛)|
Now, we apply The Limit Comparison Test Part (2):
1 1
Let 𝑦𝑛 = |sin(𝑛)| ⇒ (𝑦𝑛 ) is bounded (since 0 ≤ |sin(𝑛)| ≤ 1)
⇒ lim 𝑥𝑛 = 0
𝑛→∞
67
Remark 3.6.13: Observe that when we say that (𝑥𝑛 ) diverges means:
(𝑥𝑛 ) is properly diverges or diverges but not properly
For Example:
((−1)𝑛 ) diverges but it is not properly diverges.
(sin(𝑛)) diverges but it is not properly diverges.
(𝑛) diverges and it is properly diverges.
نظرا لعدم القدرة على اضافة أي قيمة في نهاية مجموعة الرموز هذه والمكتوبة على ً هذا "التعريف" يفتقر إلى الوضوح وذلك
.شكل صف حيث أن هذا التعريف يتطلب عددًا ال نهائي من عمليات الجمع التي يتعين القيام بها
Definition 3.7.1: If 𝑋 = (𝑥𝑛 ) is a sequence in ℝ, then the infinite series (or simply the series)
generated by 𝑋 is the sequence 𝑆 = (𝑠𝑛 ) defined by:
𝑠1 = 𝑥1
𝑠2 = 𝑠1 + 𝑥2 = 𝑥1 + 𝑥2
𝑠𝑘 = 𝑠𝑘−1 + 𝑥𝑛 = 𝑥1 + 𝑥2 + ⋯ + 𝑥𝑘
⋮
The numbers 𝑥1 , 𝑥2 , … are called the terms of the series and the numbers 𝑠1 , 𝑠2 , … are called the
partial sums of this series.
Remark 3.7.2:
(1) If lim 𝑠𝑛 exists, we say that this series is convergent and call this limit the sum or the value
𝑛→∞
of this series
(2) If this limit does not exist, we say that the series 𝑆 is divergent.
(3) It is convenient to use symbols such as:
∑(𝑥𝑛 ) or ∑ 𝑥𝑛 or ∑∞ 𝑛=1 𝑥𝑛 … … … … . . ①
to denote both the infinite series 𝑆 generated by the sequence 𝑋 = (𝑥𝑛 ) and also to denote
the value lim 𝑠𝑛 , in case this limit exists, that is ∑ 𝑥𝑛 = lim 𝑠𝑛 .
𝑛→∞ 𝑛→∞
(4) Just as a sequence may be indexed such that its first element is not 𝑥1 , but is 𝑥0 , or 𝑥5 or
𝑥99 , we will denote the series having these numbers as their first element by the symbols:
a) If 𝑥0 is the first term of the sequence 𝑋 = (𝑥𝑛 )∞ 𝑛=0 = (𝑥0 , 𝑥1 , 𝑥2 , … ):
The series 𝑆 = (𝑠𝑛 ) = (𝑠0 , 𝑠1 , 𝑠2 , … ) is denoted as ∑∞
𝑛=0 𝑥𝑛 , where
𝑠0 = 𝑥0 , 𝑠1 = 𝑥0 + 𝑥1 , … , 𝑠𝑘 = 𝑠𝑘−1 + 𝑥𝑘 , 𝑘 ≥ 1, …
b) If 𝑥5 is the first term of the sequence 𝑋 = (𝑥𝑛 )∞ 𝑛=5 = (𝑥5 , 𝑥6 , 𝑥7 , … )
The series 𝑆 = 𝑛 = (𝑠5 , 𝑠6 , 𝑠7 , … ) is denoted as ∑∞
(𝑠 ) 𝑛=5 𝑥𝑛 , where:
𝑠5 = 𝑥5 , 𝑠6 = 𝑥5 + 𝑥6 , … , 𝑠𝑘 = 𝑠𝑘−1 + 𝑥𝑘 , 𝑘 ≥ 6, …
Example 3.7.4:
1
(1) Show that if |𝑟| < 1, then the geometric series converges to 1−𝑟 , that is:
1
∑∞ 𝑛
𝑛=0 𝑟 = 1−𝑟
(2) Show that if |𝑟| ≥ 1, then the geometric series ∑∞ 𝑛
𝑛=0 𝑟 diverges.
1
Solution: Want to show that the sequence (𝑠𝑛 ) = (𝑠0 , 𝑠1 , 𝑠2 , … ) of partial sums converges to 1−𝑟
1
that is lim 𝑠𝑛 = 1−𝑟.
𝑛→∞
(𝑥𝑛 ) = (1, 𝑟, 𝑟 2 , … , 𝑟 𝑛 , … )
2 𝑛−1
𝑠𝑛 = 1 + 𝑟 + 𝑟 + ⋯ + 𝑟 + 𝑟𝑛
𝑟𝑠𝑛 = 𝑟 + 𝑟 2 + ⋯ + 𝑟 𝑛 + 𝑟 𝑛+1
𝑠𝑛 − 𝑟𝑠𝑛 = (1 + 𝑟 + 𝑟 2 + ⋯ + 𝑟 𝑛−1 + 𝑟 𝑛 ) − (𝑟 + 𝑟 2 + ⋯ + 𝑟 𝑛 + 𝑟 𝑛+1 )
= 1 − 𝑟 𝑛+1
1−𝑟 𝑛+1
⇒ 𝑠𝑛 (1 − 𝑟) = 1 − 𝑟 𝑛+1 ⇒ 𝑠𝑛 = (Closed Form of 𝑠𝑛 )
1−𝑟
1−𝑟 𝑛+1 1− lim 𝑟 𝑛+1
⇒ lim 𝑠𝑛 = lim = 𝑛→∞
……….①
𝑛→∞ 𝑛→∞ 1−𝑟 1−𝑟
1−𝑟 𝑛+1 1−0 1
(1) If |𝑟| < 1, then lim 𝑟 𝑛+1 = 0. ①: ⇒ lim 𝑠𝑛 = lim = 1−𝑟 = 1−𝑟
𝑛→∞ 𝑛→∞ 𝑛→∞ 1−𝑟
1 1
⇒ the series converges to 1−𝑟 that is ∑∞ 𝑛
𝑛=0 𝑟 = lim 𝑠𝑛 = 1−𝑟
𝑛→∞
(2) If |𝑟| ≥ 1, then lim 𝑟 𝑛+1 does not exist ⇒ lim 𝑠𝑛 does not exists
𝑛→∞ 𝑛→∞
⇒ (𝑠𝑛 ) diverges ⇒ ∑∞ 𝑛
𝑛=0 𝑟 diverges.
Remark 3.7.5: Let (𝑠𝑛 ) be the sequence of partial sums of the geometric series
∑∞ 𝑛
𝑛=𝑛0 𝑟 = 𝑟
𝑛0
+ 𝑟 1+𝑛0 + 𝑟 2+𝑛0 + ⋯ + 𝑟 𝑛+𝑛0 + ⋯
𝑟 𝑛0 −𝑟 𝑛+1 First Term−Term after last
The closed form of 𝑠𝑛 is: 𝑠𝑛 = 1−𝑟 (= )
1−Base
(1) If |𝑟| < 1, then
∑∞ 𝑛 𝑛0 2 𝑛
𝑛=𝑛0 𝑟 = 𝑟 (1 + 𝑟 + 𝑟 + ⋯ + 𝑟 + ⋯ )
1
= 𝑟 𝑛0 (∑∞ 𝑛
𝑛=0 𝑟 ) = 𝑟
𝑛0
( )
1−𝑟
𝑟 𝑛0 First Term
⇒ ∑∞
𝑛=𝑛0 𝑟 = 1−𝑟 (=
𝑛
)
1−Base
1 1 35 −1
1 1 1 ( 3 )−( 8 ) 35 −1 242
3 3 38
a) + 34 + ⋯ + 37 = 1 = 2 = 2(37 ) = 4374
33 1−
3 3
1 1 1
1 1 1 ( 𝑛)−( 𝑚 ) ( 𝑛) 1
2 2 2
b) Let 𝑚 > 𝑛. Then + 2𝑛+1 + ⋯ + 2𝑚−1 = 1 < 1 = 2𝑛−1
2𝑛 1− 1−
2 2
6 3
6 First Term ( 3) 3
24
c) ∑∞
𝑛=5 2𝑛−1 = = 1 = 2
1 =4
1−Base 1− ( )
2 2
70
1
Example 3.7.6: Determine whether the series ∑∞
𝑛=1 𝑛(𝑛+1) converges or diverges. If it converges
find its sum.
1
Solution: Let (𝑠𝑛 ) be the sequence of partial sums of ∑∞
𝑛=1 𝑛(𝑛+1). We must find a closed form of 𝑠𝑛 :
1 1 1 1 1
𝑠𝑛 = 1(2) + 2(3) + 3(4) + ⋯ + (𝑛−1)𝑛 + 𝑛(𝑛+1)
1 1 1 1 1 1 1 1 1 1
= (1 − 2) + (2 − 3) + (3 − 4) + ⋯ + (𝑛−1 − 𝑛) + (𝑛 − 𝑛+1) = 1 − 𝑛+1 (Closed Form)
1 1 1
⇒ lim 𝑠𝑛 = lim (1 − 𝑛+1) = 1 ⇒ ∑∞ ∞
𝑛=1 𝑛(𝑛+1) = lim 𝑠𝑛 = 1 ⇒ ∑𝑛=1 𝑛(𝑛+1) converges to 1
𝑛→∞ 𝑛→∞ 𝑛→∞
(−1)𝑛
Example 3.7.9: Using the Cauchy Criterion, show that the series ∑∞
𝑛=1 converges.
𝑛!
1 1
(Hint: 𝑛! ≤ 2𝑛−1 , ∀𝑛 ∈ ℕ)
Solution: We will apply the Cauchy Criterion for series:
Let 𝜀 > 0 be given and let 𝑚 > 𝑛:
(−1)𝑛+1 (−1)𝑚 (−1)𝑛+1 (−1)𝑚 1 1
| (𝑛+1)! + ⋯ + | ≤ | (𝑛+1)! | + ⋯ + | | = (𝑛+1)! + ⋯ + 𝑚!
⏟ 𝑚! ⏟ 𝑚! ⏟
𝑚−𝑛 terms by the triangle inequality 𝑚−𝑛 terms
1 1 1
≤ 2𝑛 + 2𝑛+1 + ⋯ + 2𝑚−1 (by the Hint)
1 1 1
( 𝑛)−( 𝑚 ) ( 𝑛) 1
2 2 2
= 1 < 1 = 2𝑛−1
1− 1−
2 2
(−1)𝑛+1 (−1)𝑚 1
⇒ | (𝑛+1)! + ⋯ + | < 2𝑛−1 ………①
𝑚!
1 1
Since lim = 0: Applying the definition of lim = 0, ∃𝐾 ∈ ℕ such that
𝑛→∞ 2𝑛−1 𝑛→∞ 2𝑛−1
1 1
|2𝑛−1 − 0| < 𝜀, ∀𝑛 ≥ 𝐾 ⇒ 2𝑛−1 < 𝜀, ∀𝑛 ≥ 𝐾……………②
1
Take 𝐻 = 𝐾 and let 𝑚 > 𝑛 ≥ 𝐻 ⇒ 𝑛 ≥ 𝐾 ⇒ ②: 2𝑛−1 < 𝜀, ∀𝑛 ≥ 𝐻………..③
(−1)𝑛+1 (−1)𝑚 1
① ⇒ | (𝑛+1)! + ⋯ + | < 2𝑛−1 < 𝜀, ∀𝑚 > 𝑛 ≥ 𝐻
𝑚!
(−1)𝑛
The Cauchy Criterion for series ⇒ ∑∞
𝑛=1 converges
𝑛!
1 1 1 1
Definition 3.7.10: The series ∑∞
𝑛=1 𝑛 = 1 + 2 + 3 + ⋯ + 𝑛 + ⋯ is called the Harmonic series.
71
1
Example 3.7.11: Show that the Harmonic series ∑∞ 𝑛=1 𝑛 diverges.
Proof: We will show that the Contrapositive of the Cauchy criterion for series holds:
1
Take 𝜀0 = 2, and let 𝐻 ∈ ℕ.
Take 𝑚0 , 𝑛0 ∈ ℕ such that 𝑛0 = 𝐻 and 𝑚0 = 2𝑛0 ⇒ 𝑚0 > 𝑛0 ≥ 𝐻
𝑛0 −terms
1 1 ⏞1 1 1 1
|𝑥𝑛0 +1 + 𝑥𝑛0 +2 + ⋯ + 𝑥𝑚0 | = |
⏟ + ⋯+ |= + ⋯+ ≥ 𝑛0 ( )=
𝑛0 + 1 2𝑛0 𝑛0 + 1 2𝑛0 2𝑛0 2
𝑚0 −𝑛0 =𝑛0
1
⇒ ∃𝑚0 > 𝑛0 ≥ 𝐻 such that |𝑥𝑛0 +1 + 𝑥𝑛0 +1 + ⋯ + 𝑥𝑚0 | ≥ 𝜀0 = 2
1 1
The Contrapositive of the Cauchy criterion for the series ∑∞ ∞
𝑛=1 𝑛 holds ⇒ ∑𝑛=1 𝑛 diverges.
⇒ 𝑥𝑛 = 𝑠𝑛 − 𝑠𝑛−1
⇒ lim 𝑥𝑛 = lim (𝑠𝑛 − 𝑠𝑛−1 ) = lim 𝑠𝑛 − lim 𝑠𝑛−1 = 𝑥 − 𝑥 = 0 lim 𝑥𝑛 = 0
𝑛→∞ 𝑛→∞ 𝑛→∞ 𝑛→∞ 𝑛→∞
The 𝒏𝒕𝒉 Term Test (Contraposition Form) 3.7.13: If lim 𝑥𝑛 ≠ 0, then ∑ 𝑥𝑛 diverges
Example 3.7.14:
𝑛
(1) Show that ∑∞𝑛=1 2𝑛+1 diverges
(2) Consider the series generated by ((−1)𝑛 ) that is ∑∞ 𝑛
𝑛=1(−1) . Show that the series
diverges.
Comparison Test for series 3.7.15: Let (𝑥𝑛 ) and (𝑦𝑛 ) be sequences of real numbers such that:
0 ≤ 𝑥𝑛 ≤ 𝑦𝑛 , ∀𝑛 ≥ 𝐾 for some 𝐾 ∈ ℕ.
(1) If ∑ 𝑦𝑛 converges, then ∑ 𝑥𝑛 converges.
(2) If ∑ 𝑥𝑛 diverges, then ∑ 𝑦𝑛 diverges.
Proof:
(1) Suppose that ∑ 𝑦𝑛 converges.
To show that ∑ 𝑥𝑛 converges we apply the Cauchy Criterion for series:
Let 𝜀 > 0 be given.
∑ 𝑦𝑛 converges ⇒ By the Cauchy Criterion applied to ∑ 𝑦𝑛 : ∃𝐻 ∈ ℕ such that
|𝑦𝑛+1 + 𝑦𝑛+2 + ⋯ + 𝑦𝑚 | < 𝜀, ∀𝑚 > 𝑛 ≥ 𝐻……①
⇒ 𝑦𝑛+1 + 𝑦𝑛+2 + ⋯ + 𝑦𝑚 < 𝜀, ∀𝑚 > 𝑛 ≥ 𝐻 (since 𝑦𝑛 ≥ 0, ∀𝑛 ∈ ℕ)
⇒ |𝑥𝑛+1 + 𝑥𝑛+2 + ⋯ + 𝑥𝑚 | = 𝑥𝑛+1 + 𝑥𝑛+2 + ⋯ + 𝑥𝑚
≤ 𝑦𝑛+1 + 𝑦𝑛+2 + ⋯ + 𝑦𝑚
< 𝜀, ∀𝑚 > 𝑛 ≥ 𝐻
⇒ |𝑥𝑛+1 + 𝑥𝑛+2 + ⋯ + 𝑥𝑚 | < 𝜀, ∀𝑚 > 𝑛 ≥ 𝐻
Cauchy Criterion for series ⇒ ∑ 𝑥𝑛 converges
(2) The contrapositive of part (1): If ∑ 𝑥𝑛 diverges, then ∑ 𝑦𝑛 diverges
Theorem 3.7.16: Let (𝑥𝑛 ) be a sequence of positive real numbers and let (𝑠𝑛 ) be the sequence of
partial sums of ∑ 𝑥𝑛 . Then the following are equivalent:
(1) ∑ 𝑥𝑛 converges
(2) The sequence (𝑠𝑛 ) is bounded.
(3) There is a bounded subsequence of (𝑠𝑛 ).
In this case: ∑ 𝑥𝑛 = lim 𝑠𝑛 = sup{𝑠𝑛 : 𝑛 ∈ ℕ}
𝑛→∞
Proof: Since (𝑥𝑛 ) be a sequence of positive real numbers, then (𝑠𝑛 ) is an increasing sequence:
𝑠𝑛 = 𝑠𝑛−1 + 𝑥𝑛 ≥ 𝑠𝑛−1 ⇒ 𝑠𝑛 ≥ 𝑠𝑛−1 , ∀𝑛 ∈ ℕ
(2) (1) Suppose that (𝑠𝑛 ) is bounded ⇒ (𝑠𝑛 ) bounded and increasing
Monotone Convergence Theorem MCT: ⇒ (𝑠𝑛 ) converges ⇒ ∑ 𝑥𝑛 converges
1 1 1 1
Definition 3.7.17: The series ∑∞
𝑛=1 𝑛𝑝 = 1 + 2𝑝 + 3𝑝 + ⋯ + 𝑛𝑝 + ⋯ is called the 𝒑-series.
73
Example 3.7.18:
1
(1) Show that if 𝑝 > 1, then ∑∞
𝑛=1 𝑛𝑝 converges
1
(2) Show that if 𝑝 ≤ 1, then ∑∞
𝑛=1 𝑛𝑝 diverges
Solution:
1
(1) Let 𝑝 > 1 and let (𝑠𝑛 ) be the sequence of partial sums of ∑∞
𝑛=1 𝑛𝑝 .
1
Since the series ∑∞
𝑛=1 𝑛𝑝 has positive terms, then we will show that Part (3) of Theorem 3.7.16
holds, that is we will find a bounded subsequence of (𝑠𝑛 ):
Let 𝑘𝑗 = 2𝑗 − 1, 𝑗 ∈ ℕ. Then:
𝑠𝑘1 = 𝑠1 = 1
1 1 1 1 1
𝑠𝑘2 = 𝑠3 = 1 + 2𝑝 + 3𝑝 ≤ 1 + 2𝑝 + 2𝑝 = 1 + 2𝑝−1
1 1 1 1 1 1 1 4 1 1
𝑠𝑘3 = 𝑠7 = 1
⏟+ 2𝑝 + 3𝑝 + ⏟ + 5𝑝 + 6𝑝 + 7𝑝 ≤ 1 + 2𝑝−1 + 4𝑝 = 1 + 2𝑝−1 + (2𝑝−1 )2
4𝑝
𝑠3 =𝑠𝑘2
1 1
⇒ 𝑠𝑘3 ≤ 1 + 2𝑝−1 + (2𝑝−1 )2
⋮
1 1 1 1 1
𝑠𝑘𝑛 ≤ 1 + 2𝑝−1 + (2𝑝−1 )2 + ⋯ + (2𝑝−1 )𝑛−1 ≤ ∑∞
𝑛=0 = 1
⏟ ⏟ (2𝑝−1 )𝑛 1− 𝑝−1
2
Finite geometric series 1
𝑟= 𝑝−1 ⇒0<𝑟<1 since 𝑝>1
2
1
⇒ 0 ≤ 𝑠𝑘𝑛 ≤ 1 , ∀𝑛 ∈ ℕ ⇒ (𝑠𝑘𝑛 ) is bounded
1− 𝑝−1
2
⇒ There is a bounded subsequence of (𝑠𝑛 ) ⇒ Part (3) of Theorem 3.7.16 holds
1
⇒ Part (1) of Theorem 3.7.16 holds ⇒ ∑∞𝑛=1 𝑝 converges 𝑛
𝑝 1 1
(2) Let 𝑝 ≤ 1. Then 𝑛 ≤ 𝑛, ∀𝑛 ∈ ℕ ⇒ ≤ 𝑛𝑝 , ∀𝑛 ∈ ℕ
𝑛
1 1
⇒ 0 ≤ 𝑛 ≤ 𝑛𝑝 , ∀𝑛 ∈ ℕ……..①
1
But the harmonic series ∑ diverges
𝑛
1
By ① and the Comparison Test: ⇒ ∑ 𝑛𝑝 diverges
Remark 3.7.19: Recall Theorem 3.6.7: Let (𝑥𝑛 ) be a sequence of positive numbers. Then
1
lim 𝑥𝑛 = ∞ iff lim 𝑥 = 0
𝑛→∞ 𝑛→∞ 𝑛
74
Limit Comparison Test for series 3.7.20: Let (𝑥𝑛 ) and (𝑦𝑛 ) be a sequence of positive real
𝑥
numbers such that lim 𝑦 𝑛 = 𝐿.
𝑛→∞ 𝑛
(1) If 0 < 𝐿 < ∞, then ∑ 𝑥𝑛 converges iff ∑ 𝑦𝑛 converges.
(2) If 𝐿 = 0 and ∑ 𝑦𝑛 converges, then ∑ 𝑥𝑛 converges.
(3) If 𝐿 = ∞ and ∑ 𝑥𝑛 converges, then ∑ 𝑦𝑛 converges
Proof:
𝑥𝑛 𝐿
(1) Suppose that 0 < 𝐿 < ∞. By the definition of the limit lim = 𝐿: Take 𝜀 = 2
𝑛→∞ 𝑦𝑛
𝑥𝑛 𝐿 𝐿 𝑥𝑛 𝐿
⇒ ∃𝐾 ∈ ℕ such that | 𝑦 − 𝐿| < 2 , ∀𝑛 ≥ 𝐾 ⇒ − 2 < − 𝐿 < 2 , ∀𝑛 ≥ 𝐾
𝑛 𝑦𝑛
𝐿 𝑥𝑛 3𝐿 𝐿 3𝐿
≤ ≤ , ∀𝑛 ≥ 𝐾 ⇒ (2) 𝑦𝑛 ≤ 𝑥𝑛 ≤ ( 2 ) 𝑦𝑛 , ∀𝑛 ≥ 𝐾………①
2 𝑦𝑛 2
Suppose that ∑ 𝑥𝑛 converges:
2
①: ⇒ 0 ≤ 𝑦𝑛 ≤ 𝐿 𝑥𝑛 , ∀𝑛 ≥ 𝐾……….②
②& ∑ 𝑥𝑛 converges & The Comparison Test for series 3.7.15: ⇒ ∑ 𝑦𝑛 converges
Suppose that ∑ 𝑦𝑛 converges:
3𝐿
①: ⇒ 0 ≤ 𝑥𝑛 ≤ ( 2 ) 𝑦𝑛 , ∀𝑛 ≥ 𝐾………③
③& ∑ 𝑦𝑛 converges & The Comparison Test for series 3.7.15: ⇒ ∑ 𝑥𝑛 converges
(2) Suppose that 𝐿 = 0 and ∑ 𝑦𝑛 converges.
𝑥
In the definition of the limit lim 𝑦 𝑛 = 0: Take 𝜀 = 1 ⇒ ∃𝐾 ∈ ℕ such that
𝑛→∞ 𝑛
𝑥𝑛 𝑥𝑛
| 𝑦 − 0| < 1, ∀𝑛 ≥ 𝐾 ⇒ < 1, ∀𝑛 ≥ 𝐾 ⇒ 𝑥𝑛 < 𝑦𝑛 , ∀𝑛 ≥ 𝐾
𝑛 𝑦𝑛
⇒ 0 ≤ 𝑥𝑛 < 𝑦𝑛 , ∀𝑛 ≥ 𝐾……④
④& ∑ 𝑦𝑛 converges & The Comparison Test for series 3.7.15: ⇒ ∑ 𝑥𝑛 converges
𝑥𝑛
(3) Suppose that 𝐿 = ∞ and ∑ 𝑥𝑛 converges. Since lim = ∞ we have from Theorem 3.6.7
𝑛→∞ 𝑦𝑛
1 𝑦𝑛
that lim 𝑥 = 0 ⇒ lim = 0.
𝑛→∞ ( 𝑦 𝑛) 𝑛→∞ 𝑥𝑛
𝑛
Since ∑ 𝑥𝑛 converges, it follows from Part (2) that ∑ 𝑦𝑛 converges.
Chapter 4: Limits
Definition 4.1.1: Let 𝐴 ⊆ ℝ. A point 𝑐 ∈ ℝ is a cluster point of 𝑨 if for every 𝛿 > 0 there exists
at least one point 𝑎 ∈ 𝐴, 𝑎 ≠ 𝑐 such that |𝑎 − 𝑐| < 𝛿, that is:
𝑐 is a cluster point of 𝐴 iff ∃𝑎 ∈ 𝐴, 𝑎 ≠ 𝑐 such that 𝑎 ∈ 𝑉𝛿 (𝑐) = (𝑐 − 𝛿, 𝑐 + 𝛿),∀𝛿 > 0
Remark 4.1.2:
1
(1) Taking 𝛿 = 𝑛 , 𝑛 ∈ ℕ we have:
(2) 𝑐 is a cluster point of 𝐴 iff there is a sequence (𝑎𝑛 ) in 𝐴 with lim 𝑎𝑛 = 𝑐 and 𝑎𝑛 ≠ 𝑐, ∀𝑛 ∈ ℕ
𝑛→∞
(3) To find the set of all cluster points of a set 𝐴: Take all sequences (with different terms) in 𝐴 that
are convergent. The set of all limit points of these sequences is the set of all cluster points of 𝐴.
Example 4.1.3:
(1) Let 𝐴 = {𝑎1 , … , 𝑎𝑛 } be a finite set ⇒ 𝐴 does not contain any sequence that converges
⇒ 𝐴 has no cluster points ⇒ The set of all cluster points of 𝐴 is the empty set ∅ = { }.
(2) The infinite set ℕ does not contain any sequence that converges ⇒ ℕ has no cluster points
⇒ The set of all cluster points of ℕ is the empty set ∅ = { }.
1 1
(3) Let 𝐴 = {𝑛 : 𝑛 ∈ ℕ} ⇒ 𝐴 contains sequence (𝑛) that converges to 0 ⇒ 𝐴 has only one cluster
point, which is 0 ⇒ The set of all cluster points of 𝐴 is {0}.
(4) The set of all cluster points of any one of the intervals [𝑎, 𝑏], (𝑎, 𝑏], [𝑎, 𝑏), (𝑎, 𝑏) is [𝑎, 𝑏].
𝜺-𝜹 Definition 4.1.4: Let 𝐴 ⊆ ℝ and 𝑐 ∈ ℝ be a cluster point of 𝐴, and let 𝑓: 𝐴 → ℝ be a function.
A real number 𝐿 is said to be a limit of 𝑓 at 𝑐, denoted by lim 𝑓(𝑥) = 𝐿 if ∀𝜀 > 0, ∃𝛿 > 0 such
𝑥→𝑐
that if 𝑥 ∈ 𝐴 and 0 < |𝑥 − 𝑐| < 𝛿, then |𝑓(𝑥) − 𝐿| < 𝜀.
That is lim 𝑓(𝑥) = 𝐿 iff ∀𝜀 > 0, ∃𝛿 > 0 such that if 𝑥 ∈ 𝑉𝛿 (𝑐) − {𝑐} ⇒ 𝑓(𝑥) ∈ 𝑉𝜀 (𝐿)
𝑥→𝑐
76
Example 4.1.5: Using the graph of the given function, if 𝜀 > 0 is given as in the figure then
approximate the larges possible value of 𝛿 that satisfies
the definition of the limit lim 𝑓 = 1
𝑥→0.5
Solution: lim 𝑓 = 1.
𝑥→0.5
𝜀
From the figure the largest possible value of 𝛿 is 𝛿 ≅ 3
Notations 4.1.6:
(1) lim 𝑓(𝑥) = 𝐿 may be written as:
𝑥→𝑐
lim 𝑓 = 𝐿
𝑥→𝑐
𝑓 converges to 𝐿 at 𝑐
𝑓(𝑥) approaches 𝐿 as 𝑥 approaches 𝑐.’’
(2) The definition: lim 𝑓(𝑥) = 𝐿 ∈ ℝ is equivalent to the following 3 conditions:
𝑥→𝑐
① ∀𝜀 > 0
② ∃𝛿 > 0
③ 𝑥 ∈ 𝐴 and 0 < |𝑥 − 𝑐| < 𝛿 ⇒ |𝑓(𝑥) − 𝐿| < 𝜀
𝑥 𝑐
Example 4.1.7: Using definition, show that if 𝑐 ∈ ℝ, then lim 2 = 2.
𝑥→𝑐
𝑥 𝑐
Solution: Let 𝜀 > 0 be given. 𝑓(𝑥) = 2 , 𝐿 = 2
𝑥 𝑐 |𝑥−𝑐|
|𝑓(𝑥) − 𝐿| = | − | = …….①
2 2 2
|𝑥−𝑐|
< 𝜀 ⇔ |𝑥 − 𝑐| < 2𝜀
2
Take 0 < 𝛿 ≤ 2𝜀.
|𝑥−𝑐|
Let 0 < |𝑥 − 𝑐| < 𝛿 ⇒ 0 < |𝑥 − 𝑐| < 2𝜀 ⇒ < 𝜀 ⇒ |𝑓(𝑥) − 𝐿| < 𝜀 (by ①)
2
0 < |𝑥 − 𝑐| < 𝛿 ⇒ |𝑓(𝑥) − 𝐿| < 𝜀
𝑥 𝑐
∴ lim 2 = 2.
𝑥→𝑐
Example 4.1.8: Using definition, show that if 𝑐 ∈ [0, ∞), then lim √𝑥 = √𝑐.
𝑥→𝑐
Solution: Let 𝜀 > 0 be given. 𝑓(𝑥) = √𝑥, 𝐿 = √𝑐
|𝑓(𝑥) − 𝐿| = |√𝑥 − √𝑐| ≤ √|𝑥 − 𝑐|…….①
√|𝑥 − 𝑐| < 𝜀 ⇔ |𝑥 − 𝑐| < 𝜀 2
Take 0 < 𝛿 ≤ 𝜀 2 .
Let 0 < |𝑥 − 𝑐| < 𝛿 ⇒ |𝑥 − 𝑐| < 𝜀 2 ⇒ √|𝑥 − 𝑐| < 𝜀 ⇒ |𝑓(𝑥) − 𝐿| < 𝜀 (by ①)
So, 0 < |𝑥 − 𝑐| < 𝛿 ⇒ |𝑓(𝑥) − 𝐿| < 𝜀
∴ lim √𝑥 = √𝑐.
𝑥→𝑐
77
3𝑥 2 −𝑥+2
Example 4.1.9: Using definition, show that lim = 4.
𝑥→1 𝑥 2 −𝑥+1
3𝑥 2 −𝑥+2 3𝑥 2 −𝑥+2
Solution: lim = 2 ⇒ 𝑐 = 1, 𝑓(𝑥) = and 𝐿 = 4 ⇒ |𝑥 − 𝑐| = |𝑥 − 1|
𝑥→1 𝑥 2 −𝑥+1 𝑥 2 −𝑥+1
Let 𝜀 > 0 be given.
3𝑥 2 −𝑥+2 3𝑥 2 −𝑥+2 4(𝑥 2 −𝑥+1) −𝑥 2 +3𝑥−2 𝑥 2 −3𝑥+2 (𝑥−1)(𝑥−2)
|𝑓(𝑥) − 𝐿| = | − 4| = | 𝑥 2 −𝑥+1 − |=| | = | 𝑥 2 −𝑥+1 | = | |
𝑥 2 −𝑥+1 𝑥 2 −𝑥+1 𝑥 2 −𝑥+1 𝑥 2 −𝑥+1
𝑥−2 𝑥−2
= |𝑥 − 𝑐| |⏟ | ⇒ |𝑓(𝑥) − 𝐿| = |𝑥 − 𝑐| |⏟ | …….①
𝑥 2 −𝑥+1 𝑥 2 −𝑥+1
يجب التخلص منه يجب التخلص منه
𝑥−2
Let 𝑔(𝑥) = 𝑥 2 −𝑥+1 be continuous on [𝑐 − 𝛿, 𝑐 + 𝛿] = [1 − 𝛿, 1 + 𝛿]:
مقترحة
⏞= 1 and let 𝑔(𝑥) = 𝑥 𝑥
Take 𝛿 , 𝑥 ∈ [𝑐 − 𝛿, 𝑐 + 𝛿] ⇒ 𝑔(𝑥) = 𝑥 2 −𝑥+1 , 𝑥 ∈ [−2,0]
𝑥 2 −𝑥+1
Now we find the absolute maximum value of |𝑔(𝑥)| on the interval [−2,0]
1−𝑥 2
𝑔′ (𝑥) = (𝑥 2 −𝑥+1)2 = 0 ⇒ 1 − 𝑥 2 = 0 ⇒ 𝑥 = ±1
⇒ |𝑔(𝑥)| ≤ max(|𝑔(−2)|, |𝑔(−1)|, |𝑔(0)|)
2 1 1
= max (7 , 3 , 0) = 3
1
⇒ |𝑔(𝑥)| ≤ 3 , ∀𝑥 ∈ [−2,0]………..②
1
① |𝑓(𝑥) − 𝐿| = |𝑥 − 𝑐| |𝑔(𝑥)|
⏟ ≤ 3 |𝑥 − 𝑐| (by ②)
يجب التخلص منه
1
⇒ |𝑓(𝑥) − 𝐿| ≤ 3 |𝑥 − 𝑐|…………③
1
Now, 3 |𝑥 − 𝑐| < 𝜀 ⇔ |𝑥 − 𝑐| < 3𝜀
مقترحة محسوبة
1
Let 0 < |𝑥 − 𝑐| < 𝛿 ⇒ 0 < |𝑥 − 𝑐| < min(1,3𝜀) ⇒ 0 < |𝑥 − 𝑐| < 3𝜀 ⇒ 3 |𝑥 − 𝑐| < 𝜀
⇒ |𝑓(𝑥) − 𝐿| < 𝜀 (by ③). So, 0 < |𝑥 − 𝑐| < 𝛿 ⇒ |𝑓(𝑥) − 𝐿| < 𝜀
3𝑥 2 −𝑥+2
∴ lim =4
𝑥→1 𝑥 2 −𝑥+1
𝑥 3 +3 5
Example 4.1.10: Using definition, show that lim = − 4.
𝑥→−2 𝑥 2 −𝑥−2
𝑥 3 +3 5
Solution: Let 𝑐 = −2, 𝑓(𝑥) = 𝑥 2 −𝑥−2, and 𝐿 = − 4 ⇒ |𝑥 − 𝑐| = |𝑥 + 2|
Let 𝜀 > 0 be given
𝑥3 + 3 5 𝑥3 + 3 5
|𝑓(𝑥) − 𝐿| = | 2
− (− )| = | 2
+ |
𝑥 −𝑥−2 4 𝑥 −𝑥−2 4
4𝑥 3 + 5𝑥 2 − 5𝑥 + 2
4(𝑥 3 +3)+5(𝑥 2 −𝑥−2) 4𝑥 3 +5𝑥 2 −5𝑥+2 4 5 −5 2
=| |=| |
4(𝑥 2 −𝑥−2) 4(𝑥 2 −𝑥−2) −8 6 −2 −2
(𝑥+2)(4𝑥 2 −3𝑥+1)
=| | 4 −3 1 0
4(𝑥 2 −𝑥−2)
2
4𝑥 − 3𝑥 + 1
|𝑥+2| 4𝑥 2 −3𝑥+1 |𝑥−𝑐| 4𝑥 2 −3𝑥+1
⇒ |𝑓(𝑥) − 𝐿| = | |= | |
4 𝑥 2 −𝑥−2 4 𝑥 2 −𝑥−2
78
|𝑥−𝑐| 4𝑥 2 −3𝑥+1
⇒ |𝑓(𝑥) − 𝐿| = |⏟𝑥 2 −𝑥−2 | ……①
4
نريد التخلص منه
بالحصول على القيمة
العظمى المطلقة له
4𝑥 2 −3𝑥+1
Let 𝑔(𝑥) = be continuous on [𝑐 − 𝛿, 𝑐 + 𝛿] = [−2 − 𝛿, −2 + 𝛿]
𝑥 2 −𝑥−2
مقترحة
⏞ 1
Take 𝛿 = 2……..② (why we cannot take 𝛿 = 1?)
1 1
⇒ 𝑔(𝑥) is continuous on [𝑐 − 𝛿, 𝑐 + 𝛿] = [−2 − , −2 + ] = [−2.5, −1.5]
2 2
Want to find the absolute maximum of |𝑔(𝑥)| on the interval [−2.5, −1.5]:
−𝑥 2 −18𝑥+7
𝑔′ (𝑥) = (𝑥 2 −𝑥−2)2
(How?)
18±√352
𝑔′ (𝑥) = 0 ⇒ −𝑥 2 + 18𝑥 + 7 = 0 ⇒ 𝑥 2 − 18𝑥 − 7 = 0 ⇒ 𝑥 = 2
18−√352 18+√352
⇒ = 9 − √88 ≅ −0.4 and = 9 + √88 ≅ 18.4
2 2
|𝑥−𝑐| |𝑥−𝑐| 58 29
①&③: ⇒ |𝑓(𝑥) − 𝐿| = |𝑔(𝑥)| ≤ ( 7 ) = 14 |𝑥 − 𝑐|, ∀𝑥 ∈ [−2.5, −1.5]
4 4
29
⇒ |𝑓(𝑥) − 𝐿| ≤ 14 |𝑥 − 𝑐|, ∀𝑥 ∈ [−2.5, −1.5] ……④
29 14𝜀
Now, take 14 |𝑥 − 𝑐| < 𝜀 ⇔ |𝑥 − 𝑐| < 29
مقترحة محسوبة
⏞
1 ⏞
14𝜀
Take 0 < 𝛿 ≤ min ( , ). So, 0 < |𝑥 − 𝑐| < 𝛿 ⇒ |𝑓(𝑥) − 𝐿| < 𝜀
2 29
𝑥 3 +3 5
∴ lim = − 4.
𝑥→−2 𝑥 2 −𝑥−2
Let 𝜀 > 0 be given. By the definition of lim 𝑓 = 𝐿: ∃𝛿 > 0 such that ∀𝑥 ∈ 𝐴 we have
𝑥→𝑐
The contraposition of the Sequential Criterion for Limits implies the following divergence criteria:
Divergence Criteria 4.1.13: Let 𝑓: 𝐴 → ℝ be a function and let 𝑐, 𝐿 ∈ ℝ such that 𝑐 is a cluster
point of 𝐴. Then
(1) lim 𝑓 ≠ 𝐿 iff ∃(𝑥𝑛 ) ⊆ 𝐴 with 𝑥𝑛 ≠ 𝑐, ∀𝑛 ∈ ℕ and 𝑥𝑛 → 𝑐 such that lim 𝑓(𝑥𝑛 ) ≠ 𝐿.
𝑥→𝑐 𝑛→∞
(2) lim 𝑓 does not exist iff ∃(𝑥𝑛 ) ⊆ 𝐴 with 𝑥𝑛 ≠ 𝑐, ∀𝑛 ∈ ℕ and 𝑥𝑛 → 𝑐 such that (𝑓(𝑥𝑛 )) diverges.
𝑥→𝑐
(3) lim 𝑓 does not exist iff ∃(𝑥𝑛 ) ⊆ 𝐴, ∃(𝑦𝑛 ) ⊆ 𝐴 with 𝑥𝑛 ≠ 𝑐, 𝑦𝑛 ≠ 𝑐, ∀𝑛 ∈ ℕ and 𝑥𝑛 → 𝑐,
𝑥→𝑐
𝑦𝑛 → 𝑐 such that lim 𝑓(𝑥𝑛 ) ≠ lim 𝑓(𝑦𝑛 ).
𝑛→∞ 𝑛→∞
80
1
Example 4.1.14: Using the divergence criterion, show that lim 𝑥 does not exist
𝑥→0
1 1
Proof: Let 𝑓(𝑥) = 𝑥 , 𝑥 ∈ ℝ − {0} and let 𝑐 = 0 and 𝑥𝑛 = 𝑛 , 𝑛 ∈ ℕ
1 1 1
⇒ (𝑛) ⊆ ℝ − {0}, 𝑛 ≠ 0, ∀𝑛 ∈ ℕ and 𝑛 → 0
1 1
Now, (𝑓(𝑥𝑛 )) = (𝑥 ) = ( 1 ) = (𝑛) is properly diverges (diverges to +∞)
𝑛
𝑛
1
Part (b) of the Divergence Criteria ⇒ lim 𝑥 does not exist
𝑥→0
1
Example 4.1.15: Using the divergence criterion, show that lim sin (𝑥) does not exist
𝑥→0
1 1 1
Proof: Let 𝑥𝑛 = 𝑛 , 𝑛 ∈ ℕ ⇒ 𝑛 ≠ 0 and 𝑛 → 0
1
Now, (sin (𝑥 )) = (sin(𝑛)) diverges (See Example 3.4.7)
𝑛
1
Divergence Criteria Part (b): ⇒ lim sin (𝑥) does not exist
𝑥→0
Example 4.1.16: Show that lim sgn(𝑥) does not exist, where sgn(𝑥) is the signum function
𝑥→0
1 ,𝑥 > 0
defined by sgn(𝑥) = { 0 , 𝑥 = 0}
−1 , 𝑥 < 0
1 1
Solution: We will apply Part (c) of the Divergence Criteria: Let 𝑥𝑛 = 𝑛 and 𝑦𝑛 = − 𝑛, 𝑛 ∈ ℕ
⇒ 𝑥𝑛 ≠ 0, 𝑦𝑛 ≠ 0, ∀𝑛 ∈ ℕ and 𝑥𝑛 → 0, 𝑦𝑛 → 0.
1
(sgn(𝑥𝑛 )) = (sgn (𝑛)) = (1) = (1,1,1, … ) ⇒ lim sgn(𝑥𝑛 ) = 1
1
(sgn(𝑦𝑛 )) = (sgn (− 𝑛)) = (−1) = (−1, −1, −1, … ) ⇒ lim sgn(𝑦𝑛 ) = −1
⇒ lim sgn(𝑥𝑛 ) ≠ lim sgn(𝑦𝑛 )
Part (c) of the Divergence Criteria ⇒ lim sgn(𝑥) does not exist
𝑥→0
1 1
Exercise 4.1.17: Let 𝑓: ℝ → ℝ be a bounded function, let 𝐼𝑛 = (− , ) , 𝑛 ∈ ℕ, and let
𝑛 𝑛
𝑎𝑛 = sup 𝑓(𝐼𝑛 ) and 𝑏𝑛 = inf 𝑓(𝐼𝑛 ) , 𝑛 ∈ ℕ.
(1) Show that (𝑎𝑛 ) is a decreasing sequence
(2) Show that (𝑏𝑛 ) is an increasing sequence
(3) Show that if lim 𝑎𝑛 = lim 𝑏𝑛 , then lim 𝑓 exists
𝑛→∞ 𝑛→∞ 𝑥→0
Definition 4.2.1: Let 𝐴 ⊆ ℝ, 𝑓: 𝐴 → ℝ is a function and let 𝑐 ∈ ℝ be a cluster point of 𝐴. The function
𝑓 is called bounded on a neighborhood of 𝑐 if there exists a 𝛿-neighborhood 𝑉𝛿 (𝑐) = (𝑐 − 𝛿, 𝑐 + 𝛿)
and a constant 𝑀 > 0 such that:
|𝑓(𝑥)| ≤ 𝑀, ∀𝑥 ∈ 𝐴 ∩ 𝑉𝛿 (𝑐).
Lemma 4.2.2: Let 𝐴 ⊆ ℝ, 𝑓: 𝐴 → ℝ is a function and let 𝑐, 𝐿 ∈ ℝ. If lim 𝑓 exists, then 𝑓 is bounded
𝑥→𝑐
𝑓 lim 𝑓 1 1
(5) Quotient Rule: If lim 𝑔 ≠ 0, then lim 𝑔 = 𝑥→𝑐 . In particular, lim 𝑔 =
𝑥→𝑐 lim 𝑔𝑥→𝑐 𝑥→𝑐 lim 𝑔
𝑥→𝑐 𝑥→𝑐
We will use the Sequential Criterion for Limits 4.1.12. So, let (𝑥𝑛 ) be a sequence in 𝐴 satisfying:
𝑥𝑛 ≠ 𝑐, ∀𝑛 ∈ ℕ and 𝑥𝑛 → 𝑐.
Since lim 𝑓 = 𝐿1 and lim 𝑔 = 𝐿2 , then the Sequential Criterion for Limits 4.1.12 implies that:
𝑥→𝑐 𝑥→𝑐
and
lim 𝑔(𝑥𝑛 ) = 𝐿2 ……….②
𝑛→∞
(1) lim (𝑏𝑓)(𝑥𝑛 ) = lim 𝑏𝑓(𝑥𝑛 ) = 𝑏 lim 𝑓(𝑥𝑛 ) = 𝑏𝐿1 ⇒ lim (𝑏𝑓)(𝑥𝑛 ) = 𝑏𝐿1
𝑛→∞ 𝑛→∞ 𝑛→∞ 𝑛→∞
(2) lim (𝑓 + 𝑔)(𝑥𝑛 ) = lim (𝑓(𝑥𝑛 ) + 𝑔(𝑥𝑛 )) = lim 𝑓(𝑥𝑛 ) + lim 𝑔(𝑥𝑛 ) = 𝐿1 + 𝐿2
𝑛→∞ 𝑛→∞ 𝑛→∞ 𝑛→∞
⇒ lim (𝑓 + 𝑔)(𝑥𝑛 ) = 𝐿1 + 𝐿2
𝑛→∞
(4) lim (𝑓𝑔)(𝑥𝑛 ) = lim (𝑓(𝑥𝑛 )𝑔(𝑥𝑛 )) = ( lim 𝑓(𝑥𝑛 )) ( lim 𝑔(𝑥𝑛 )) = 𝐿1 𝐿2
𝑛→∞ 𝑛→∞ 𝑛→∞ 𝑛→∞
⇒ lim (𝑓𝑔)(𝑥𝑛 ) = 𝐿1 𝐿2
𝑛→∞
𝑓 𝐿 lim 𝑓
𝑥→𝑐
∴ The Sequential Criterion for Limits 4.1.12⇒ lim 𝑔 = 𝑀 =
𝑥→𝑐 lim 𝑔
𝑥→𝑐
83
1 lim 1 1
(2) lim 𝑥 −𝑛 = lim 𝑥 𝑛 = 𝑥→𝑐
⏟= 𝑐 𝑛 = 𝑐 −𝑛
𝑥→𝑐 ⏟ 𝑥→𝑐 lim 𝑥 𝑛
𝑥→𝑐
by Part (1)
by the Quotient Rule
Solution:
(1) Let 𝑝(𝑥) = 𝑎𝑛 𝑥 𝑛 + 𝑎𝑛−1 𝑥 𝑛−1 + ⋯ + 𝑎1 𝑥 + 𝑎0 and
𝑞(𝑥) = 𝑏𝑚 𝑥 𝑚 + 𝑏𝑚−1 𝑥 𝑚−1 + ⋯ + 𝑏1 𝑥 + 𝑏0
lim𝑞(𝑥) = lim(𝑏𝑚 𝑥 𝑚 + 𝑏𝑚−1 𝑥 𝑚−1 + ⋯ + 𝑏1 𝑥 + 𝑏0 )
𝑥→𝑐 𝑥→𝑐
= 𝑏𝑚 𝑐 𝑚 + 𝑏𝑚−1 𝑐 𝑚−1 + ⋯ + 𝑏1 𝑐 + 𝑏0
⏟
by Example 7
= 𝑞(𝑐)
≠0
Similarly, lim𝑝(𝑥) = 𝑝(𝑐).
𝑥→𝑐
1 lim 1 1
(2) Observe that lim𝑥 = 𝑐 ≠ 0 ⇒ lim 𝑥 = 𝑥→𝑐 =𝑐
𝑥→𝑐 ⏟
𝑥→𝑐 lim 𝑥
𝑥→𝑐
by the Quotient Rule
𝑥 2 −4 0
(3) lim 𝑥 3 −8 = 0 ⇒ We cannot apply the Quotient Rule
𝑥→2
𝑥 2 −4 (𝑥−2)(𝑥+2) 𝑥+2 4 1
lim = lim (𝑥−2)(𝑥 2 +2𝑥+4) = ⏟
lim = 12 = 3
𝑥→2 𝑥 3 −8 𝑥→2 𝑥→2 𝑥 2 +2𝑥+4
by the Quotient Rule
1 1
(4) Since lim 𝑥 = 0 ⇒ We cannot apply the Quotient Rule
𝑥→0
1
Let 𝛿 > 0 and let 𝑓(𝑥) = 𝑥 , 𝑥 ∈ ℝ∗
1
⇒ is unbounded on ℝ∗ ∩ 𝑉𝛿 (0), ∀𝛿 > 0
𝑥 -3 -2 -1 1 2
1
⇒ is unbounded on (−𝛿, 0) ∪ (0, 𝛿), ∀𝛿 > 0
𝑥
We will use the sequential Criterion for Limits 4.1.12 to prove Parts (1)&(2):
Let (𝑥𝑛 ) be a sequence in 𝐴 satisfying: 𝑥𝑛 ≠ 𝑐, ∀𝑛 ∈ ℕ and 𝑥𝑛 → 𝑐.
Since lim 𝑓 = 𝐿………..①
𝑥→𝑐
①& The Sequential Criterion for Limits 4.1.12 ⇒ lim 𝑓(𝑥𝑛 ) = 𝐿……..②
𝑛→∞
③& The Sequential Criterion for Limits 4.1.12 ⇒ lim |𝑓(𝑥)| = |𝐿|
𝑥→𝑐
∴ lim|𝑓| = |lim𝑓|
𝑥→𝑐 𝑥→𝑐
(2)
lim|𝑓| = |lim𝑓| = |0| = 0 ⇒ lim|𝑓| = 0
Suppose that lim 𝑓 = 0 ⇒ ⏟
𝑥→𝑐 𝑥→𝑐 𝑥→𝑐 𝑥→𝑐
by Part (1)
Suppose that lim|𝑓| = 0 ⇒ 0 |𝑓| = |lim𝑓| ⇒ |lim 𝑓| = 0 ⇒ lim𝑓 = 0
𝑥→𝑐 ⏟= lim 𝑥→𝑐 𝑥→𝑐 𝑥→𝑐 𝑥→𝑐
by Part (1)
∴ lim √𝑓 = √lim𝑓
𝑥→𝑐 𝑥→𝑐
86
𝑎 ≤ lim 𝑓 ≤ 𝑏.
𝑥→𝑐
Proof: Since lim 𝑓 exists ⇒ let lim 𝑓 = 𝐿. We will use the sequential Criterion for Limits 4.1.12:
𝑥→𝑐 𝑥→𝑐
①& the Sequential Criterion for Limits 4.1.12⇒ lim 𝑓(𝑥𝑛 ) = 𝐿………..②
𝑛→∞
Proof: Since lim 𝑓 and lim ℎ both exist and lim 𝑓 = lim ℎ ⇒ let lim 𝑓 = lim ℎ = 𝐿. We will use
𝑥→𝑐 𝑥→𝑐 𝑥→𝑐 𝑥→𝑐 𝑥→𝑐 𝑥→𝑐
The Sequential Criterion for Limits 4.1.12 ⇒ lim 𝑔 = 𝐿 ⇒ lim 𝑔 = lim 𝑓 = lim ℎ
𝑥→𝑐 𝑥→𝑐 𝑥→𝑐 𝑥→𝑐
87
Example 4.2.13:
(1) Show that lim sin 𝑥 = 0 (Hint: |sin 𝑥| ≤ |𝑥| for all 𝑥 ∈ ℝ)
𝑥→0
𝑥2
(2) Show that lim cos 𝑥 = 1 (Hint: 1 − ≤ cos 𝑥 ≤ 1 for all 𝑥 ∈ ℝ)
𝑥→0 2
cos 𝑥−1 𝑥2
(3) Show that lim = 0 (Hint: 1 − ≤ cos 𝑥 ≤ 1 for all 𝑥 ∈ ℝ)
𝑥→0 𝑥 2
sin 𝑥 𝑥2 sin 𝑥
(4) Show that lim = 1 (Hint: 1 − ≤ ≤ 1 for all 𝑥 ∈ ℝ∗ )
𝑥→0 𝑥 6 𝑥
Proof:
(1) |sin 𝑥| ≤ |𝑥| for 𝑥 ∈ ℝ ⇒ 0 ≤ |sin 𝑥| ≤ |𝑥| for 𝑥 ∈ ℝ
The squeeze theorem ⇒ lim 0 ≤ lim |sin 𝑥| ≤ lim|𝑥| ⇒ 0 ≤ lim|sin 𝑥| ≤ 0
𝑥→0 𝑥→0 𝑥→0 𝑥→0
𝑥2 𝑥2
(3) 1− ≤ cos 𝑥 ≤ 1 for 𝑥 ∈ ℝ ⇒ − ≤ cos 𝑥 − 1 ≤ 0 for 𝑥 ∈ ℝ
2 2
𝑥2 𝑥2
⇒ − ≤ cos 𝑥 − 1 ≤ for 𝑥 ∈ ℝ
2 2
𝑥2 |𝑥|2
⇒ |cos 𝑥 − 1| ≤ = for 𝑥 ∈ ℝ (dividing by |𝑥|)
2 2
|cos 𝑥−1| |𝑥| cos 𝑥−1 |𝑥|
⇒ |𝑥|
≤ for 𝑥 ∈ ℝ∗ ⇒ 0 ≤ | |≤ for 𝑥 ∈ ℝ∗
2 𝑥 2
(4) Exercise
88
Solving Problems:
Q 4.2.14)
(1) Let 𝑐 ∈ ℝ be a cluster point of 𝐴 ⊆ ℝ and 𝑓, 𝑔 are functions on 𝐴.
Show that if lim 𝑓 = 0 and 𝑔 is bounded on 𝐴, then lim 𝑓𝑔 = 0.
𝑥→𝑐 𝑥→𝑐
1
(2) Using Part (1), show that lim 𝑥sin (𝑥) = 0.
𝑥→0
𝑥−1
(3) Using Part (1), show that lim 𝑥 2 cos (𝑥ln(𝑥)) = 0
𝑥→0
Solution:
(1) lim 𝑓 = 0 ⇒ lim |𝑓(𝑥)| = 0
𝑥→𝑐 𝑥→𝑐
𝑔 is bounded on 𝐴 ⇒ ∃𝑀 > 0 such that |𝑔(𝑥)| ≤ 𝑀, ∀𝑥 ∈ 𝐴 multiply by |𝑓(𝑥)|
⇒ |𝑓(𝑥)||𝑔(𝑥)| ≤ 𝑀|𝑓(𝑥)|, ∀𝑥 ∈ 𝐴 ⇒ |𝑓(𝑥)𝑔(𝑥)| ≤ 𝑀|𝑓(𝑥)|, ∀𝑥 ∈ 𝐴
⇒ 0 ≤ |𝑓(𝑥)𝑔(𝑥)| ≤ 𝑀|𝑓(𝑥)|, ∀𝑥 ∈ 𝐴
⇒ lim 0 ≤ lim|𝑓(𝑥)𝑔(𝑥)| ≤ lim 𝑀|𝑓(𝑥)|
𝑥→𝑐 𝑥→𝑐 𝑥→𝑐
⇒ 0 ≤ lim|𝑓(𝑥)𝑔(𝑥)| ≤ 𝑀lim |𝑓(𝑥)| ⇒ 0 ≤ lim|𝑓(𝑥)𝑔(𝑥)| ≤ 𝑀(0) = 0
𝑥→𝑐 𝑥→𝑐 𝑥→𝑐
⇒ lim|𝑓(𝑥)𝑔(𝑥)| = 0 ⇒ lim𝑓(𝑥)𝑔(𝑥) = 0 ⇒ lim 𝑓𝑔 = 0
𝑥→𝑐 𝑥→𝑐 𝑥→𝑐
(2) lim 𝑥 = 0
𝑥→0
1 1
|sin (𝑥)| ≤ 1, ∀𝑥 ∈ ℝ∗ ⇒ sin (𝑥) is bounded on ℝ∗
1
Part (1) ⇒ lim 𝑥sin ( ) = 0
𝑥→0 𝑥
(3) lim 𝑥 2 = 0
𝑥→0
𝑥−1 𝑥−1
|cos (𝑥ln(𝑥))| ≤ 1, ∀𝑥 ∈ ℝ − {0,1} ⇒ cos (𝑥ln(𝑥)) is bounded on ℝ − {0,1}
𝑥−1
Part (1) ⇒ lim 𝑥 2 cos (𝑥ln(𝑥)) = 0
𝑥→0
Q 4.2.17) Let 𝑓: ℝ → ℝ such that 𝑓(𝑥 + 𝑦) = 𝑓(𝑥) + 𝑓(𝑦), ∀𝑥, 𝑦 ∈ ℝ and let lim 𝑓 be exists.
𝑥→0
(1) Show that lim 𝑓 = 0
𝑥→0
(2) Show that lim 𝑓 = 𝑓(𝑐), ∀𝑐 ∈ ℝ
𝑥→𝑐
1
(Hint: For Part (1): Show that 𝑓(𝑛𝑥) = 𝑛𝑓(𝑥), ∀𝑛 ∈ ℕ and then replace 𝑥 by 𝑛 and
then use the Sequential Criterion for Limits 4.1.12)
Remark 4.3.2:
(1) The limits lim+ 𝑓 and lim− 𝑓 are called one-sided limits of 𝑓 at 𝑐.
𝑥→𝑐 𝑥→𝑐
(2) For the values of lim+ 𝑓 and lim− 𝑓, it is possible that:
𝑥→𝑐 𝑥→𝑐
(a) Both of them exist and equal: lim+ 𝑓 = lim− 𝑓 = 𝐿 ∈ ℝ
𝑥→𝑐 𝑥→𝑐
(b) Both of them exist and are different: lim+ 𝑓 ≠ lim− 𝑓
𝑥→𝑐 𝑥→𝑐
(c) Both of them does not exist
(d) One of them may be exist and the other does not exist.
Theorem 4.3.3: Let 𝑐 ∈ ℝ be a cluster point of both the sets 𝐴 ∩ (𝑐, ∞) and
𝐴 ∩ (−∞, 𝑐) and let 𝑓: 𝐴 → ℝ.
Then lim 𝑓 = 𝐿 iff lim+ 𝑓 = 𝐿 lim− 𝑓 = 𝐿
𝑥→𝑐 𝑥→𝑐 𝑥→𝑐
91
Definition 4.3.5:
(1) lim 𝑓(𝑥) = ∞ iff ∀𝛼 > 0, ∃𝛿 > 0 such that [𝑥 ∈ 𝐴 and 0 < |𝑥 − 𝑐| < 𝛿 ⇒ 𝑓(𝑥) > 𝛼].
𝑥→𝑐
(2) lim 𝑓(𝑥) = ∞ iff ∀𝛼 > 0, ∃𝛿 > 0 such that [𝑥 ∈ 𝐴 and 0 < 𝑥 − 𝑐 < 𝛿 ⇒ 𝑓(𝑥) > 𝛼].
𝑥→𝑐 +
(3) lim 𝑓(𝑥) = ∞ iff ∀𝛼 > 0, ∃𝛿 > 0 such that [𝑥 ∈ 𝐴 and 0 < 𝑐 − 𝑥 < 𝛿 ⇒ 𝑓(𝑥) > 𝛼].
𝑥→𝑐 −
(4) lim 𝑓(𝑥) = 𝐿 iff ∀𝜀 > 0, ∃𝑀 > 0 such that [𝑥 ∈ 𝐴 and 𝑥 > 𝛼 ⇒ |𝑓(𝑥) − 𝐿| < 𝜀].
𝑥→∞
(5) lim 𝑓(𝑥) = ∞ iff ∀𝛼 > 0, ∃𝑀 > 0 such that [𝑥 ∈ 𝐴 and 𝑥 > 𝑀 ⇒ 𝑓(𝑥) > 𝛼].
𝑥→∞
92
(1) Let 𝑐 ∈ ℝ be a cluster point of the set {𝑥 ∈ 𝐴: 𝑥 > 𝑐} and let 𝑓: 𝐴 → ℝ be a function.
If 𝐿 ∈ ℝ or 𝐿 = ∞ or 𝐿 = −∞, then the following are equivalent:
(a) lim+ 𝑓 = 𝐿
𝑥→𝑐
(b) ∀ sequence (𝑥𝑛 ) in 𝐴 with 𝑥𝑛 → 𝑐 + we have lim 𝑓(𝑥𝑛 ) = 𝐿.
𝑛→∞
(2) Let 𝑐 ∈ ℝ be a cluster point of the set {𝑥 ∈ 𝐴: 𝑥 < 𝑐} and let 𝑓: 𝐴 → ℝ be a function.
If 𝐿 ∈ ℝ or 𝐿 = ∞ or 𝐿 = −∞, then the following are equivalent:
(a) lim− 𝑓 = 𝐿
𝑥→𝑐
(b) ∀ sequence (𝑥𝑛 ) in 𝐴 with 𝑥𝑛 → 𝑐 − we have lim 𝑓(𝑥𝑛 ) = 𝐿.
𝑛→∞
93
Theorem 4.3.8: Let 𝑐 ∈ ℝ and let 𝑓: (𝑐, ∞) → ℝ such that 𝑓(𝑥) > 0, ∀𝑥 ∈ (𝑐, ∞). Then
1
(1) lim+ 𝑓 = ∞ iff lim+ 𝑓 = 0
𝑥→𝑐 𝑥→𝑐
1
(2) lim+ 𝑓 = 0 iff lim+ 𝑓 = ∞
𝑥→𝑐 𝑥→𝑐
Proof:
(1) lim+ 𝑓 = ∞ iff ∀ sequence (𝑥𝑛 ) in 𝐴 with 𝑥𝑛 → 𝑐 + we have lim 𝑓(𝑥𝑛 ) = ∞
𝑥→𝑐 𝑛→∞
1
iff ∀ sequence (𝑥𝑛 ) in 𝐴 with 𝑥𝑛 → 𝑐 + we have lim =0
𝑛→∞ 𝑓(𝑥𝑛 )
1
iff lim+ 𝑓 = 0
𝑥→𝑐
1 1
(2) lim+ 𝑓 = ∞ ⏟ iff lim 1 = 0 iff lim+ 𝑓 = 0
𝑥→𝑐 𝑥→𝑐 + 𝑓 𝑥→𝑐
by Part (1)
Example 5.1.2: Using the given graph of a function 𝑓, if 𝜀 > 0 is as in the figure then approximate
the larges value of 𝛿 that satisfies the definition of the continuity of the function at 𝑥 = 0.5.
0.5 1 1.5
Solution: 𝑓 is continuous at 𝑥 = 0.5 ⇔ lim 𝑓 = 𝑓(0.5) = 1. From the figure we find that the
𝑥→0.5
𝜀
largest value of 𝛿 is 𝛿 ≅ 3.
Remark 5.1.3: Recall that
𝑎−𝑏 𝑎+𝑏
(1) sin 𝑎 − sin 𝑏 = 2 sin ( ) cos ( )
2 2
𝑎−𝑏 𝑎+𝑏
(2) cos 𝑎 − cos 𝑏 = 2 sin ( ) sin ( )
2 2
95
Definition 5.1.5:
(1) a point 𝑐 ∈ ℝ is called a cluster point of a set 𝑨 if ∃(𝑥𝑛 ) in 𝐴 such that 𝑥𝑛 ≠ 𝑐, ∀𝑛 ∈ ℕ
and 𝑥𝑛 → 𝑐.
(2) a point 𝑐 ∈ ℝ is called an isolated point of a set 𝑨 if 𝑐 is not a cluster point of 𝐴.
Remark 5.1.6: 𝐵 is the set of all cluster points of a set 𝐴 iff ℝ − 𝐵 is the set of all isolated points of 𝐴.
Example 5.1.7: Find the sets of all cluster points and the set of all isolated points of the following sets:
(1) 𝐴 = [0,1] (2) 𝐴 = (0,1) (3) 𝐴 = {0} ∪ (1, ∞).
Solution:
(1) The sets of all cluster points of 𝐴 is [0,1] = 𝐴
The set of all isolated points of 𝐴 is ℝ\[0,1] = (−∞, 0) ∪ (1, ∞).
(2) The sets of all cluster points of 𝐴 is [0,1]
The set of all isolated points of 𝐴 is ℝ\[0,1] = (−∞, 0) ∪ (1, ∞).
(3) The sets of all cluster points of 𝐴 is [1, ∞)
The set of all isolated points of 𝐴 is ℝ\[1, ∞) = (−∞, 1).
1 ,𝑥 = 0
Example 5.1.9: Determine whether 𝑓 is continuous at 𝑥 = 0, where 𝑓(𝑥) = { }
𝑥2 ,𝑥 ≥ 1
4
3
Solution: Observe that 0 ∈ Dom(𝑓) = {0} ∪ (1, ∞)
⇒ 𝑥 = 0 is an isolated point of Dom(𝑓) 2
⇒ 𝑓 continuous at 𝑥 = 0 (by Theorem 4).. 1
1 2
Note
) في مجاالت االقترانات هي دراسة ذات اهمية تطبيقيةisolated points( يجب العلم بأن دراسة االتصال عند النقاط المعزولة
أما النقاط غير المعزولة (أي النقاط العنقودية، وذلك الى كون االقترانات دائما متصلة عند هذه النقاط،قليلة
لذلك سنركز في دراستنا على االتصال عند النقاط،)) فيمكن أن يكون االقتران متصال أو غير متصل عندهاcluster points(
.العنقودية في مجاالت االقترانات
بما أن مفهوم النهاية مرتبط بالنقاط العنقودية وكذلك مرتبط بمفهوم الجوار فسنعمل في النظرية التالية على ربط مفهوم االتصال
.بمفهوم النهاية عند النقاط العنقودية فقط
Remark 5.1.12: To study the continuity of a function 𝑓 at a point 𝑐, we have two possible cases
for 𝑐:
Case 1: 𝑐 ∈ Domain (𝑓): We have two cases:
Case 1.1: 𝑐 is an isolated point in Dom (𝑓) ⇒ 𝑓 is continuous at 𝑐.
Case 1.2: 𝑐 is a cluster point of Dom (𝑓) ⇒ We have to compare the values of
𝑓(𝑐) and lim 𝑓
𝑥→𝑐
If lim 𝑓 = 𝑓(𝑐) ⇒ 𝑓 is continuous at 𝑐
𝑥→𝑐
If lim 𝑓 DNE or lim 𝑓 ≠ 𝑓(𝑐) ⇒ 𝑓 is discontinuous at 𝑐
𝑥→𝑐 𝑥→𝑐
Case 2: 𝑐 ∉ Domain (𝑓), to say 𝑓 is discontinuous at 𝑐 we must have 𝑐 a cluster point of
Dom (𝑓) otherwise we do not talk about the continuity of functions at isolated points not in
its domain (see the following example):
97
-2
Step 1: Isolated points in 𝐃𝐨𝐦 (𝒇): 𝑥 = 1 is the only one isolated point of Dom (𝑓) which is in
Dom (𝑓) ⇒ 𝑓 is continuous at 𝑥 = 1.
Step 2: The remaining points in Dom(𝑓) are cluster points of Dom (𝑓) which are in Dom (𝑓):
⇒ 𝑓 is continuous on (−∞, −2) ∪ (−2, −1) ∪ (−1,0) ∪ (0,0.5] ∪ [1.5, ∞)
Conclusion: 𝑓 is continuous on (−∞, −2) ∪ (−2, −1) ∪ (−1,0) ∪ (0,0.5] ∪ {1} ∪ [1.5, ∞)
Remark 5.1.15: From now on (ً )من اآلن فصاعداwhen we talk about the continuity of a function
𝑓 at a point 𝑐, we are assuming that 𝑐 is a cluster point of Dom(𝑓).
1 ,𝑥 > 0
Example 5.1.16: Show that the signum function sgn(𝑥) = { 0 , 𝑥 = 0} is discontinuous at 𝑥 = 0.
−1 , 𝑥 < 0
Solution: 0 ∈ Dom(sgn(𝑥)) = ℝ and 𝑥 = 0 is a cluster point of Dom(sgn(𝑥)):
lim+ sgn(𝑥) = 1 & lim− sgn(𝑥) = −1 ⇒ lim sgn(𝑥) DNE ⇒ 𝑓 is discontinuous at 𝑥 = 0.
𝑥→0 𝑥→0 𝑥→0
Remark 5.1.19: Observe that lim 𝑓(𝑥𝑛 ) ≠ 𝑓(𝑐) has two meanings:
𝑛→∞
It might be lim 𝑓(𝑥𝑛 ) DNE (does not exist) so we have lim 𝑓(𝑥𝑛 ) ≠ 𝑓(𝑐)
𝑛→∞
It might be lim 𝑓(𝑥𝑛 ) exists but does not equal to 𝑓(𝑐) so we have lim 𝑓(𝑥𝑛 ) ≠ 𝑓(𝑐)
𝑛→∞
1
sin (𝑥) ,𝑥 ≠ 0
Example 5.1.20: Determine whether the function 𝑓(𝑥) = { } is continuous at 𝑥 = 0.
1 ,𝑥 = 0
Solution: 𝑓(0) = 1 and 𝑥 = 0 is a cluster point of Dom(𝑓) = ℝ.
1
Observe that lim 𝑓 = lim sin (𝑥) DNE (by using the Sequential Criterion for discontinuity:
𝑥→0 𝑥→0
1 1 1
Consider the sequence (𝑛) ⇒ 𝑛 ∈ Dom(𝑓) and 𝑛 → 0
1 1
∴ lim 𝑓 (𝑛) = lim sin ( 1 ) = lim sin(𝑛) DNE (by Example 7 of Section 3.4)
𝑛→∞ 𝑛→∞ 𝑛
𝑛→∞
1
⇒ lim 𝑓 (𝑛) ≠ 𝑓(0) ⇒ 𝑓 is discontinuous at 𝑥 = 0.
𝑛→∞
𝑥3 + 𝑥 ,𝑥 ∈ ℚ
Example 5.1.22: Find all points in ℝ at which the function 𝛹(𝑥) = { } is
−2𝑥 2 , 𝑥 ∈ ℚ𝑐
continuous.
Solution: 𝛹 is continuous at the points when 𝑥 3 + 𝑥 = −2𝑥 2 ⇒ 𝑥(𝑥 + 1)2 = 0
⇒ 𝑥 = 0 or 𝑥 = −1 ⇒ 𝛹 is continuous only at 𝑥 = 0 and 𝑥 = −1
1 , 𝑥 is rational
Example 5.1.23: Show that Dirichlet’s 𝛹(𝑥) = { } function is nowhere
0 , 𝑥 is irrational
continuous.
Solution: 𝛹 is continuous at the points when 1 = 0 ⇒ 𝛹 is discontinuous at all points of ℝ ⇒ 𝛹
is nowhere continuous
99
Solving Problems:
Problem 5.1.25: Let 𝑓: 𝐴 → ℝ be continuous at 𝑐 ∈ 𝐴. Show that ∀𝜀 > 0, ∃𝛿 > 0 such that
if 𝑥, 𝑦 ∈ 𝐴 with |𝑥 − 𝑐| < 𝛿 and |𝑦 − 𝑐| < 𝛿, then |𝑓(𝑥) − 𝑓(𝑦)| < 𝜀
Proof: Let 𝜀 > 0 be given.
𝜀
Since 𝑓 is continuous at 𝑐, we apply the definition of continuity for 2: ⇒ ∃𝛿 > 0 such that
𝜀
If 𝑡 ∈ 𝐴 with |𝑡 − 𝑐| < 𝛿 , then |𝑓(𝑡) − 𝑓(𝑐)| < 2………①
Let 𝑥, 𝑦 ∈ 𝐴 with |𝑥 − 𝑐| < 𝛿 and |𝑦 − 𝑐| < 𝛿
𝜀 𝜀
① ⇒ |𝑓(𝑥) − 𝑓(𝑐)| < 2 and |𝑓(𝑦) − 𝑓(𝑐)| < 2……….②
Now,
|𝑓(𝑥) − 𝑓(𝑦)| = |(𝑓(𝑥) − 𝑓(𝑐)) − (𝑓(𝑦) − 𝑓(𝑐))|
≤ |𝑓(𝑥) − 𝑓(𝑐)| + |𝑓(𝑦) − 𝑓(𝑐)| (by the triangle inequality)
𝜀 𝜀
< + (by ②)
2 2
=𝜀
Proof: We will prove part (1). The proof of part (2) is similar.
(1) suppose that 𝑓(𝑐) > 0.
𝑓(𝑐)
Since 𝑓 is continuous at 𝑐, we apply the definition of continuity for 𝜀 = 2 :
𝑓(𝑐)
⇒ ∃𝛿 > 0 such that if 𝑥 ∈ 𝐴 with |𝑥 − 𝑐| < 𝛿, then |𝑓(𝑥) − 𝑓(𝑐)| < 𝜀 = 2
𝑓(𝑐) 𝑓(𝑐)
⇒ − < 𝑓(𝑥) − 𝑓(𝑐) < , ∀𝑥 ∈ 𝐴 ∩ 𝑉𝛿 (𝑐)
2 2
𝑓(𝑐) 3𝑓(𝑐)
⇒ < 𝑓(𝑥) < , ∀𝑥 ∈ 𝐴 with |𝑥 − 𝑐| < 𝛿
2 2
𝑓(𝑐)
⇒ 𝑓(𝑥) > > 0, ∀𝑥 ∈ 𝐴 ∩ 𝑉𝛿 (𝑐) ⇒ 𝑓(𝑥) > 0, ∀𝑥 ∈ 𝐴 ∩ 𝑉𝛿 (𝑐)
2
In this section all points that we discuss the continuity at are assumed to be cluster points to the
domains of the functions
⇒ 𝑓 + 𝑔 is continuous at 𝑐
𝑓 lim 𝑓 𝑓(𝑐) 𝑓 𝑓
lim = 𝑥→𝑐 = = ( ) (𝑐) ⇒ is continuous at 𝑐
𝑥→𝑐 𝑔 lim 𝑔
𝑥→𝑐
𝑔(𝑐) 𝑔 𝑔
Theorem 5.2.3: Let 𝐴, 𝐵 ⊆ ℝ and let 𝑓: 𝐴 → ℝ and 𝑔: 𝐵 → ℝ be functions such that 𝑓(𝐵) ⊆ 𝐴.
(1) If 𝑓 is continuous at 𝑐 ∈ 𝐴 and 𝑔 is continuous at 𝑏 = 𝑓(𝑐) ∈ 𝐵, then the composition function
𝑔 ∘ 𝑓: 𝐴 → ℝ is continuous at 𝑐.
(2) If 𝑓 is continuous on 𝐴 and 𝑔 is continuous on 𝐵, then the composition function
𝑔 ∘ 𝑓: 𝐴 → ℝ is continuous on 𝐴.
Proof:
(1) Let 𝑐 ∈ 𝐴 and let 𝑏 = 𝑓(𝑐).
𝑓 is continuous at 𝑐 ⇒ lim 𝑓(𝑥) = 𝑓(𝑐)
𝑥→𝑐
1 1
𝑓(𝑥) = 𝑥 is unbounded on (0, ∞) 𝑓(𝑥) = 1+𝑥 2 is bounded on ℝ
since 𝑓((0, ∞)) = (0, ∞) has no UB since 1 is an UB and 0 is a LB of
𝑓(ℝ) = Range(𝑓) = (0,1]
Remark 5.3.3: If 𝑓 is continuous on an open bounded interval (𝑎, 𝑏), then 𝑓 need not be bounded
on (𝑎, 𝑏). Consider the following example:
𝜋 𝜋
Example 5.3.4: Let 𝑓(𝑥) = tan 𝑥 is continuous on the open bounded interval (− 2 , 2 ). In fact 𝑓
𝜋 𝜋
is unbounded on (− 2 , 2 ) (see the graph of the function 𝑓 given in the figure)
Question 5.3.5: Let 𝑓: 𝐼 → ℝ be a function that is continuous on an interval 𝐼 and 𝐼 is not closed or
unbounded. When 𝑓 is bounded on 𝐼? The answer is given in the following Theorem:
Theorem 5.3.6: Let 𝐼 be an interval with end points 𝑎, 𝑏 with 𝑎 < 𝑏, and let 𝑓: 𝐼 → ℝ be a
function such that:
(1) 𝑓 is continuous on 𝐼.
(2) lim 𝑓 and lim 𝑓 both exist.
𝑥→𝑎 𝑥→𝑏
Then 𝑓 is bounded on 𝐼, where 𝐼 is any of the following intervals:
[𝑎, 𝑏], ⏟
(𝑎, 𝑏], [𝑎, 𝑏), (𝑎, 𝑏) , ⏟
(−∞, 𝑏], (−∞, 𝑏), [𝑎, ∞), (𝑎, ∞), ℝ = (−∞, ∞).
not closed unbounded
104
Since 𝑓 is continuous on the closed bounded interval [𝑎, 𝑏] ⇒ 𝑓 is bounded on [𝑎, 𝑏].
⇒ 𝑓(𝐴) = Range(𝑓) = {𝑓(𝑥): 𝑥 ∈ [𝑎, 𝑏]} has an UB and has a LB. So, Let 𝑠 ∗ = sup 𝑓([𝑎, 𝑏])
Claim 1: ∃𝑥 ∗ ∈ [𝑎, 𝑏] such that 𝑓(𝑥 ∗ ) = 𝑠 ∗ :
1
Pf of Claim 1: Since 𝑠 ∗ = sup 𝑓(𝐴) ⇒ 𝑠 ∗ − 𝑛 is not an UB of Range(𝑓), ∀𝑛 ∈ ℕ.
1
⇒ ∃𝑥𝑛 ∈ [𝑎, 𝑏] such that 𝑠 ∗ − 𝑛 < 𝑓(𝑥𝑛 ) ≤ 𝑠 ∗ , ∀𝑛 ∈ ℕ.
⇒ (𝑥𝑛 ) is a sequence in [𝑎, 𝑏] and since [𝑎, 𝑏] is a bounded interval we have: (𝑥𝑛 ) is a
bounded sequence
The Bolzano-Weierstrass Theorem ⇒ (𝑥𝑛 ) has a convergent subsequence (𝑥𝑛𝑘 ).
Since (𝑥𝑛𝑘 ) is a sequence in [𝑎, 𝑏] with lim 𝑥𝑛𝑘 = 𝑢 and 𝑓 continuous at 𝑢, then Sequential
𝑘→∞
Criterion implies that lim 𝑓(𝑥𝑛𝑘 ) = 𝑓(𝑢)…….①
𝑘→∞
1
Since 𝑠 ∗ − < 𝑓(𝑥𝑛 ) ≤ 𝑠 ∗ , ∀𝑛 ∈ ℕ and (𝑥𝑛𝑘 ) is a subsequence of (𝑥𝑛 ) we have:
𝑛
1 1
𝑠 ∗ − 𝑛 < 𝑓(𝑥𝑛𝑘 ) ≤ 𝑠 ∗ , ∀𝑘 ∈ ℕ ⇒ lim (𝑠 ∗ − 𝑛 ) < lim 𝑓(𝑥𝑛𝑘 ) ≤ lim 𝑠 ∗
𝑘 𝑘→∞ 𝑘 𝑘→∞ 𝑘→∞
⇒ ∃𝑥∗ ∈ [𝑎, 𝑏] such that 𝑓(𝑥∗ ) = inf 𝑓(𝐴) ≡ The absolute min. of 𝑓
∴ 𝑓 has its absolute minimum on [𝑎, 𝑏].
Example 5.3.9: Show that if the function 𝑓 is continuous on the closed bounded interval [𝑎, 𝑏] and
𝑓(𝑥) > 0, ∀𝑥 ∈ [𝑎, 𝑏], then ∃𝛼 > 0 such that 𝑓(𝑥) ≥ 𝛼, ∀𝑥 ∈ [𝑎, 𝑏]
Proof: 𝑓 is continuous on the closed bounded interval [𝑎, 𝑏] ⇒ 𝑓 has its absolute minimum point at
some 𝑥∗ ∈ [𝑎, 𝑏] that is 𝑓(𝑥) ≥ 𝑓(𝑥∗ ), ∀𝑥 ∈ [𝑎, 𝑏]………①
Since 𝑥∗ ∈ [𝑎, 𝑏] and 𝑓(𝑥) > 0, ∀𝑥 ∈ [𝑎, 𝑏] ⇒ 𝑓(𝑥∗ ) > 0……….②
Take 𝛼 = 𝑓(𝑥∗ ) ⇒ 𝛼 > 0 (by ②) and 𝑓(𝑥) ≥ 𝛼, ∀𝑥 ∈ [𝑎, 𝑏] (by ①).
Location of Roots Theorem 5.3.10: Let 𝑓 be continuous on the closed bounded interval [𝑎, 𝑏] such
𝑓(𝑎)𝑓(𝑏) < 0. Then ∃𝑐 ∈ (𝑎, 𝑏) such that 𝑓(𝑐) = 0, that is the equation 𝑓(𝑥) = 0 has a real root
that lies between 𝑎 and 𝑏.
Since 𝑐 = sup 𝐵 ⇒ ∃(𝑥𝑛 ) in 𝐵 such that 𝑥𝑛 → 𝑐. But lim 𝑓 = 𝑓(𝑐), so by the Sequential Criterion
𝑥→𝑐
we have: lim 𝑓(𝑥𝑛 ) = 𝑓(𝑐)
𝑛→∞
Claim: 𝑓(𝑐) = 0
Pf of the Claim: We will prove it by contradiction:
Assume that 𝑓(𝑐) ≠ 0 ⇒ 𝑓(𝑐) < 0 (by ②)
Since 𝑓 continuous at 𝑐 and 𝑓(𝑐) < 0 we have: ∃𝛿 > 0 such that
𝑓(𝑥) < 0, ∀𝑥 ∈ (𝑐 − 𝛿, 𝑐 + 𝛿) ⊂ [𝑎, 𝑏]
𝛿
Since 𝑑 = 𝑐 + 2 ∈ (𝑐 − 𝛿, 𝑐 + 𝛿) ⇒ 𝑓(𝑑) < 0 ⇒ 𝑑 ∈ 𝐵 ⇒ 𝑑 ≤ sup 𝐵 = 𝑐
𝛿 𝛿
⇒ 𝑐 + ≤ 𝑐 ⇒ ≤ 0 ⇒ 𝛿 ≤ 0 contradiction
2 2
Example 5.3.11: Show that the function 𝑓(𝑥) = 2 ln 𝑥 + √𝑥 − 2 has a zero in [1,2].
Solution:
𝑓 is continuous on the closed bounded interval [1,2]
𝑓(1) = −1 < 0 and 𝑓(2) = 2 ln 2 + √2 − 2 > 0 ⇒ 𝑓(1)𝑓(2) < 0
Location of Roots Theorem ⇒ ∃𝑐 ∈ (1,2) such that 𝑓(𝑐) = 0
∴ 𝑓 has a zero in [1,2]
1
Example 5.3.12: Let 𝑓 be continuous on [0,1] such that 𝑓(0) = 𝑓(1). Prove that ∃𝑐 ∈ [0, 2] such
1
that 𝑓(𝑐) = 𝑓 (𝑐 + 2).
Solution:
1 1 1
Case 1: If 𝑓(0) = 𝑓 (2), take 𝑐 = 0 and so ∃𝑐 ∈ [0, 2] such that 𝑓(𝑐) = 𝑓 (𝑐 + 2) which is 𝑐 = 0.
1 1 1
Case 2: If 𝑓(0) ≠ 𝑓 ( ), then let 𝑔(𝑥) = 𝑓(𝑥) − 𝑓 (𝑥 + ) , 𝑥 ∈ [0, ]
2 2 2
1
𝑔 continuous on the closed bounded interval [0, 2].
1 1 1 1
𝑔(0) = 𝑓(0) − 𝑓 (2) and 𝑔 (2) = 𝑓 (2) − 𝑓(1) = 𝑓 (2) − 𝑓(0) (since 𝑓(0) = 𝑓(1))
2
1 1 1
⇒ 𝑔(0)𝑔 (2) = − (𝑓(0) − 𝑓 (2)) < 0 (since 𝑓(0) ≠ 𝑓 (2))
1
⇒ 𝑔(0)𝑔 (2) < 0
1
The Location of Roots Theorem: ⇒ ∃𝑐 ∈ [0, 2] such that 𝑔(𝑐) = 0
1 1
⇒ 𝑓(𝑐) − 𝑓 (𝑐 + 2) = 0 ⇒ 𝑓(𝑐) = 𝑓 (𝑐 + 2).
107
Corollary 5.3.14: Let 𝑓 be continuous on the closed bounded interval [𝑎, 𝑏].
(1) If inf 𝑓([𝑎, 𝑏]) ≤ 𝑘 ≤ sup 𝑓([𝑎, 𝑏]), then ∃𝑐 ∈ [𝑎, 𝑏] such that 𝑓(𝑐) = 𝑘.
(2) 𝑓([𝑎, 𝑏]) is a closed bounded interval, that is 𝑓([𝑎, 𝑏]) = [𝑚, 𝑀], where:
𝑚 = inf 𝑓([𝑎, 𝑏]) and 𝑀 = sup 𝑓([𝑎, 𝑏])
Corollary 5.3.15: Let 𝑓 be continuous on an interval 𝐼. Then 𝑓(𝐼) is an interval, where the
intervals 𝐼 and 𝑓(𝐼) need not be closed and bounded.
1
Remark 5.4.2: Let 𝑓(𝑥) = 𝑥 , 𝑥 > 0. Let 𝜀 > 0 be fixed:
1
Taking 𝑦 = 2 ⇒ 𝑓(𝑦) = 𝑓(2) = 2:
⇒ ∃𝛿1 > 0 satisfying the definition
1 1
Taking 𝑦 = 2 ⇒ 𝑓(𝑦) = 𝑓(2) = 2:
⇒ ∃𝛿2 > 0 satisfying the definition
Observe that for a fixed 𝜀 > 0, changing the point 𝑦 implies that 𝛿 may be changes.
Definition 5.4.3:
A function 𝑓: 𝐴 → ℝ is said to be uniformly continuous on 𝑨 if ∀𝜀 > 0, ∃𝛿 > 0 such that
∀𝑥, 𝑦 ∈ 𝐴 with |𝑥 − 𝑦| < 𝛿 ⇒ |𝑓(𝑥) − 𝑓(𝑦)| < 𝜀.
فإن هذا اإلقتران يكون اتصاله منتظما ً على المجموعة 𝐴 إذا لم يحدث الوضع،𝐴 ليكن لدينا اقتران 𝑓 متصالً على مجموعة:مالحظة
𝑓 فإذا حدث هذا الوضع فإن اإلقتران. ال يوجد مماسات لالقتران 𝑓 تقترب من الوضع العامودي بشكل مستمر دون توقف:التالي
ً يكون اتصاله على المجموعة 𝐴 غير منتظم وفي هذه الحالة نبحث عن مجموعة جزئية داخل 𝐴 والتي يكون اإلتصال عليها منتظما
(أي ال يحدث الوضع السابق فيها) ولتحقيق ذلك علينا أن نضع شرطا ً على المجموعة 𝐴 نمنع فيها اإلقتراب من الوضع العامودي
.𝑓 لمماسات المنحنى
109
Example 5.4.5: Find the largest set on which the function is uniformly continuous:
1
(1) 𝑓(𝑥) = 𝑥 (2) 𝑓(𝑥) = 𝑥 2 (3) 𝑓(𝑥) = 𝑒 𝑥
𝑥 3 −𝑥
(4) 𝑓(𝑥) = 𝑥 3 +𝑥 (5) 𝑓(𝑥) = √𝑥 (6) 𝑓(𝑥) = 𝑥
1
(7) 𝑓(𝑥) = ln 𝑥 (8) 𝑓(𝑥) = sin (𝑥)
Solution:
(1) 𝑓 is continuous on ℝ\{0} ⇒ 𝑓 is not uniformly continuous on ℝ\{0}
because approaching 0 leads to a vertical tangent line
So, prevent 𝑥 → 0− and 𝑥 → 0+ : Let 𝑎 > 0. Then
𝑓 is uniformly continuous on (−∞, −𝑎] ∪ [𝑎, ∞),∀𝑎 > 0.
1
The function 𝑓(𝑥) = 𝑥 is uniformly continuous on each of the following sets:
(−∞, −5], (−3, −1), (0.1,2], (1, ∞), and (−5, −2] ∪ [−1, −0.1) ∪ (1,7)
1
The function 𝑓(𝑥) = 𝑥 is not uniformly continuous on each of the following sets:
(−∞, 0), (0,1), (0,2], (0, ∞), and (−5, −2] ∪ (0,4] ∪ (1, ∞)
𝑥 3 −𝑥
(4) 𝑓(𝑥) = 𝑥 3 +𝑥 is continuous on ℝ\{−1,0}.
⇒ 𝑓 is not uniformly continuous on ℝ\{−1,0} because approaching −1 (only) leads to a vertical
tangent line. So, prevent 𝑥 → −1 (from right and from left: 𝑥 ↛ −1− and 𝑥 ↛ −1+ )
Let 𝑎 > 1, 0 < 𝑏 < 1. Then 𝑓 is uniformly continuous on (−∞, −𝑎] ∪ [−𝑏, 𝑏] ∪ [𝑎, ∞).
(5) 𝑓(𝑥) = √𝑥 is continuous on [0, ∞)
⇒ 𝑓 is uniformly continuous on [0, ∞).
1
(8) 𝑓(𝑥) = sin (𝑥) is continuous on ℝ\{0}.
Its graph is given in the figure.
⇒ 𝑓 is not uniformly continuous on ℝ\{0}
Let 𝑎 > 0. Then 𝑓 is uniformly continuous on
(−∞, −𝑎] ∪ [𝑎, ∞)
Remark 5.4.6: Recall that a function 𝑓: 𝐴 → ℝ is said to be uniformly continuous on A iff ∀𝜀 > 0,
∃𝛿 > 0 such that ∀𝑥, 𝑦 ∈ 𝐴 with |𝑥 − 𝑦| < 𝛿 ⇒ |𝑓(𝑥) − 𝑓(𝑦)| < 𝜀.
Example 5.4.7: Using definition, show that 𝑓(𝑥) = 𝑥 4 is uniformly continuous on [−5,4].
Solution: Let 𝜀 > 0 be given and let 𝑥, 𝑦 ∈ [−5,4].
|𝑓(𝑥) − 𝑓(𝑦)| = |𝑥 4 − 𝑦 4 |
= |(𝑥 2 + 𝑦 2 )(𝑥 2 − 𝑦 2 )|
= |(𝑥 2 + 𝑦 2 )(𝑥 + 𝑦)(𝑥 − 𝑦)|
≤ |(𝑥 2 + 𝑦 2 )(𝑥 + 𝑦)||𝑥 − 𝑦|
≤ ((|𝑥 2 | + |𝑦 2 |)(|𝑥| + |𝑦|)|𝑥 − 𝑦|
= ((𝑥 2 + 𝑦 2 )(|𝑥| + |𝑦|)|𝑥 − 𝑦|
≤ ((25 + 25)(5 + 5))|𝑥 − 𝑦|
= 500|𝑥 − 𝑦|
⇒ |𝑓(𝑥) − 𝑓(𝑦)| ≤ 500|𝑥 − 𝑦|.
𝜀
Now, let 500|𝑥 − 𝑦| < 𝜀 ⇒ |𝑥 − 𝑦| < 500
𝜀
Take 0 < 𝛿 ≤ 500. So, ∀𝑥, 𝑦 ∈ [−5,4] with |𝑥 − 𝑦| < 𝛿 ⇒ |𝑓(𝑥) − 𝑓(𝑦)| < 𝜀
⇒ 𝑓(𝑥) = 𝑥 4 is uniformly continuous on [−5,4]
Example 5.4.9: Using definition, show that 𝑓(𝑥) = √𝑥 is uniformly continuous on [0, ∞).
Solution: Let 𝜀 > 0 be given and let 𝑥, 𝑦 ∈ [0, ∞).
|𝑓(𝑥) − 𝑓(𝑦)| = |√𝑥 − √𝑦|
To simplify dealing with uniform continuity we start with the following Theorems:
Uniform Continuity Theorem 5.4.10: If 𝑓: [𝑎, 𝑏] → ℝ is continuous on the closed bounded
interval [𝑎, 𝑏], then 𝑓 is uniformly continuous on [𝑎, 𝑏].
Example 5.4.12:
(1)
1
𝑓(𝑥) = 𝑥 is uniformly continuous on [3,10] (since 𝑓 is continuous on the closed
bounded interval [3,10]).
𝑓 is uniformly continuous on (3,7] ∪ (8,9) since 𝑓 is uniformly continuous on [3,10]
and (3,7] ∪ (8,9) ⊆ [3,10].
(2) If 𝑃(𝑥) is a polynomial, then 𝑃(𝑥) is uniformly continuous on any closed bounded interval
[𝑎, 𝑏] (since 𝑃(𝑥) is continuous on [𝑎, 𝑏] and [𝑎, 𝑏] is a closed bounded interval).
For examples: 𝑓(𝑥) = 𝑥 3 − 𝜋𝑥 + 1 and 𝑔(𝑥) = −0.1𝑥 5 − √2𝑥 4 are uniformly
continuous on the closed bounded intervals [−5,0] and [1,2.5].
(3) 𝑓(𝑥) = sin 𝑥 and 𝑔(𝑥) = cos 𝑥 are uniformly continuous on any closed bounded interval
[𝑎, 𝑏] (because they are continuous on [𝑎, 𝑏] and [𝑎, 𝑏] is a closed bounded interval).
Example 5.4.14: Using the Nonuniform Continuity Criterion, show that 𝑓(𝑥) = 𝑥 2 is not uniformly
continuous on ℝ.
Proof:
1
Let 𝑥𝑛 = 𝑛 + 𝑛 and 𝑦𝑛 = 𝑛, 𝑛 ∈ ℕ ⇒ (𝑥𝑛 ), (𝑦𝑛 ) are sequences in ℝ.
113
1 1
lim (𝑥𝑛 − 𝑦𝑛 ) = lim (𝑛 + 𝑛 − 𝑛) = lim =0
𝑛→∞ 𝑛→∞ 𝑛→∞ 𝑛
1 2 1 1 1
|𝑓(𝑥𝑛 ) − 𝑓(𝑦𝑛 )| = |(𝑛 + 𝑛) − 𝑛2 | = |𝑛2 + 2 + 𝑛2 − 𝑛2 | = |2 + 𝑛2 | = 2 + 𝑛2 ≥ 2, ∀𝑛 ∈ ℕ
1
Example 5.4.15: Using the Nonuniform Continuity Criterion, show that 𝑓(𝑥) = sin (𝑥) is not uniformly
continuous on (0, ∞).
Solution:
2 1
Let 𝑥𝑛 = (4𝑛+1)𝜋 and 𝑦𝑛 = 𝑛𝜋 ⇒ (𝑥𝑛 ), (𝑦𝑛 ) are in (0, ∞).
2 1
lim (𝑥𝑛 − 𝑦𝑛 ) = lim ((4𝑛+1)𝜋 − 𝑛𝜋) = 0
𝑛→∞ 𝑛→∞
1 1 (4𝑛+1)𝜋
|𝑓(𝑥𝑛 ) − 𝑓(𝑦𝑛 )| = |sin ( 2 ) − sin ( 1 ) | = |sin ( ) − sin(𝑛𝜋) | = 1, ∀𝑛 ∈ ℕ
2
(4𝑛+1)𝜋 𝑛𝜋
Take 𝜀0 = 1. The nonuniform Continuity Criterion holds ⇒ 𝑓 is not uniformly continuous on (0, ∞).
Remark 5.4.18:
(1) The Mean-Value Theorem says that: If 𝑓: [𝑎, 𝑏] → ℝ is a function continuous on the closed
bounded interval [𝑎, 𝑏] and differentiable on (𝑎, 𝑏), then ∃𝑐 ∈ (𝑎, 𝑏) such that
𝑓(𝑏)−𝑓(𝑎)
𝑓 ′ (𝑐) = . Equivalently, 𝑓(𝑏) − 𝑓(𝑎) = 𝑓 ′ (𝑐)(𝑏 − 𝑎)
𝑏−𝑎
1
Example 5.4.20: Show that the function 𝑓(𝑥) = 1+𝑥 2 , 𝑥 ∈ ℝ is uniformly continuous on ℝ. (Hint:
Show that 𝑓 is Lipschitz on ℝ).
Solution: We want to find 𝑓 ′ and to show it is bounded on ℝ:
2𝑥 |𝑥| 1 |𝑥| |𝑥|
|𝑓 ′ (𝑥)| = |(1+𝑥 2 )2 | = 2 (1+𝑥 2 )2 = 2 ( 2 ) ( 2 ) ≤ 2(1) ( 2 ) ≤ 2, ∀𝑥 ∈ ℝ
1+𝑥 1+𝑥 1+𝑥
Example 5.4.23:
sin 𝑥 sin 𝑥
(1) The function 𝑓(𝑥) = 𝑥 is continuous on ℝ\{0} and lim 𝑓 = lim 𝑥 = 1
𝑥→0 𝑥→0
𝑓(𝑥) , 𝑥 ∈ ℝ − {0}
⇒ 𝐹(𝑥) = { }
1 ,𝑥 = 0
sin 𝑥
, 𝑥 ∈ ℝ − {0}
𝐹(𝑥) = { 𝑥 } is the continuous extension of 𝑓.
1 ,𝑥 = 0
𝑥
(2) The function 𝑓(𝑥) = 𝑥 2 −𝑥 is continuous on ℝ\{0,1}:
𝑥 𝑥 1
lim 𝑓 = lim = lim = lim doesn’t exist
𝑥→1 𝑥→1 𝑥 2 −𝑥 𝑥→1 𝑥(𝑥−1) 𝑥→1 𝑥−1
⇒ The function 𝑓 has no continuous extension.
115
Example 5.4.26:
sin 𝑥
(a) Show that 𝑓(𝑥) = 𝑥 is uniformly continuous on (0,1].
1
(b) Show that 𝑓(𝑥) = 𝑥 is uniformly continuous on (0,1).
Solution:
(a)
(1) 𝑓 continuous on the bounded interval (0,1]
sin 𝑥 sin 𝑥
(2) lim+ = 1 exists and lim− = sin 1 exists
𝑥→0 𝑥 𝑥→1 𝑥
sin 𝑥
,0 < 𝑥 ≤ 1
⇒ 𝑓 has the continuous extension on [0,1] which is 𝐹(𝑥) = { 𝑥 }
1 ,𝑥 = 0
So, 𝑓 is uniformly continuous on (0,1].
1
(b) lim+ 𝑥 does not exist ⇒ 𝑓 hasn’t a continuous extension on [0,1].
𝑥→0
⇒ 𝑓 is not uniformly continuous on (0,1).
Example 5.4.28: Give examples of two functions 𝑓 and 𝑔 that are uniformly continuous on a set 𝐴
but 𝑓𝑔 is not uniformly continuous on 𝐴.
Solution: Let 𝑓(𝑥) = 𝑔(𝑥) = 𝑥 ⇒ 𝑓 and 𝑔 are uniformly continuous on ℝ
But (𝑓𝑔)(𝑥) = 𝑥 2 is not uniformly continuous on ℝ.
116
Theorem 5.4.29: Let 𝑓 and 𝑔 be uniformly continuous on 𝐴. If 𝑓 and 𝑔 are bounded on 𝐴, then 𝑓𝑔
is uniformly continuous on 𝐴.
Proof: Let 𝜀 > 0 be given. 𝑓 and 𝑔 are bounded on 𝐴 ⇒ ∃𝑀 > 0 such that
|𝑓(𝑥)| ≤ 𝑀, |𝑔(𝑥)| ≤ 𝑀, ∀𝑥 ∈ 𝐴………..①
𝑓 and 𝑔 be uniformly continuous on 𝐴 ⇒ ∃𝛿 > 0 such that
𝜀 𝜀
𝑥, 𝑦 ∈ 𝐴 and |𝑥 − 𝑦| < 𝛿 ⇒ |𝑓(𝑥) − 𝑓(𝑦)| < 2𝑀 and |𝑔(𝑥) − 𝑔(𝑦)| < 2𝑀…………②
|(𝑓𝑔)(𝑥) − (𝑓𝑔)(𝑦)| = |𝑓(𝑥)𝑔(𝑥) − 𝑓(𝑥)𝑔(𝑦) + 𝑓(𝑥)𝑔(𝑦) − 𝑓(𝑦)𝑔(𝑦)|
≤ |𝑓(𝑥)𝑔(𝑥) − 𝑓(𝑥)𝑔(𝑦)| + |𝑓(𝑥)𝑔(𝑦) − 𝑓(𝑦)𝑔(𝑦)|
= |𝑓(𝑥)| |𝑔(𝑥) − 𝑔(𝑦)| + |𝑔(𝑦)| |𝑓(𝑥) − 𝑓(𝑦)|
≤ 𝑀 |𝑔(𝑥) − 𝑔(𝑦)| + 𝑀 |𝑓(𝑥) − 𝑓(𝑦)| (by①)
𝜀 𝜀
≤ 𝑀 2𝑀 + 𝑀 2𝑀 (by ②)
=𝜀
𝑥, 𝑦 ∈ 𝐴 and |𝑥 − 𝑦| < 𝛿 ⇒ |(𝑓𝑔)(𝑥) − (𝑓𝑔)(𝑦)| < 𝜀
⇒ 𝑓𝑔 is uniformly continuous on 𝐴
Example 5.4.31: Show that 𝑓(𝑥) = |√𝑥 − 1| is uniformly continuous on [0, ∞).
Solution: Let 𝑔(𝑥) = √𝑥 and ℎ(𝑥) = |𝑥 − 1|. Then 𝑔 is uniformly continuous on [0, ∞) and ℎ is
uniformly continuous on ℝ ⇒ ℎ ∘ 𝑔 is uniformly continuous on [0, ∞)
But (𝑔 ∘ ℎ)(𝑥) = 𝑔(ℎ(𝑥)) = |ℎ(𝑥) − 1| = |√𝑥 − 1| = 𝑓(𝑥)
∴ 𝑓 is uniformly continuous on [0, ∞).
Theorem 5.6.6: Let 𝐼 ⊆ ℝ be an interval and let 𝑓 ∶ 𝐼 → ℝ be monotone on 𝐼. Then the set of
discontinuities of 𝑓 on 𝐼 is a countable set.
118
Example 5.6.7: True or false: There a function 𝑓 ∶ [0,1) → ℝ decreasing on [0,1) and
discontinuous on [0,1) ∩ ℚ𝑐 . Justify.
Solution: The answer is No. Because assume that there is a function 𝑓 on [0,1) discontinuous on
[0,1) ∩ ℚ𝑐 . Theorem 6 ⇒ [0,1) ∩ ℚ𝑐 is a countable set which is a contradiction since [0,1) ∩ ℚ𝑐
is uncountable ⇒ our assumption is false
⇒ there is no such function.
Remark 5.6.8:
(1) A function 𝑓∶𝐴→ℝ is injective (one-to-one) iff whenever
𝑥1 , 𝑥2 ∈ 𝐴 with 𝑥1 ≠ 𝑥2 we have 𝑓(𝑥1 ) ≠ 𝑓(𝑥2 ). Equivalently, 𝑓 is injective iff whenever
𝑥1 , 𝑥2 ∈ 𝐴 with 𝑓(𝑥1 ) = 𝑓(𝑥2 ) we have 𝑥1 = 𝑥2 .
(2) If a function 𝑓 ∶ 𝐴 → ℝ is injective, then its inverse function 𝑓 −1 exists.
(3) If a function 𝑓 ∶ 𝐴 → ℝ is strictly monotone, then 𝑓 is injective and so its inverse function 𝑓 −1 exists.
Chapter 6: Differentiation
Definition 6.1.1: Let 𝐼 be an interval, let 𝑓: 𝐼 → ℝ and let 𝑐 ∈ 𝐼. We say that a real number 𝐿 is
𝑓(𝑥)−𝑓(𝑐)
the derivative of 𝒇 at 𝒄, if lim 𝑥−𝑐 = 𝐿. In this case we say that 𝑓 is differentiable at 𝑐 and we
𝑥→𝑐
𝑓(𝑥)−𝑓(𝑐)
write 𝑓 ′ (𝑐) = 𝐿, that is: 𝑓 ′ (𝑐) = lim
𝑥→𝑐 𝑥−𝑐
Proof: Since 𝑓 has a derivative at 𝑐, we have 𝑓 ′ (𝑐) exists and is a real number.
𝑓(𝑥) − 𝑓(𝑐)
lim(𝑓(𝑥) − 𝑓(𝑐)) = lim × (𝑥 − 𝑐)
𝑥→𝑐 𝑥→𝑐 𝑥−𝑐
𝑓(𝑥) − 𝑓(𝑐)
= lim × lim(𝑥 − 𝑐) = 𝑓 ′ (𝑐) × 0 = 0
𝑥→𝑐 𝑥−𝑐 𝑥→𝑐
⇒ lim(𝑓(𝑥) − 𝑓(𝑐)) = 0 ⇒ lim 𝑓(𝑥) = 𝑓(𝑐) ⇒ 𝑓 is continuous at 𝑐.
𝑥→𝑐 𝑥→𝑐
Remark 6.1.6: The contraposition of Theorem 5 says that: Let 𝐼 be an interval in ℝ
If 𝑓 is discontinuous at 𝑐 ∈ 𝐼, then 𝑓 is not differentiable at 𝑐.
The Chain Rule 6.1.8: Let 𝐼, 𝐽 be intervals in ℝ, let 𝑔: 𝐼 → ℝ, 𝑓: 𝐽 → ℝ be functions such that
𝑓(𝐽) ⊆ 𝐼, and let 𝑐 ∈ 𝐽. If 𝑓 is differentiable at 𝑐 and 𝑔 is differentiable at 𝑓(𝑐), then the composition
function 𝑔 ∘ 𝑓 is differentiable at 𝑐 with
(𝑔 ∘ 𝑓 )′ (𝑐) = 𝑔′ (𝑓(𝑐)) ∙ 𝑓 ′ (𝑐)
122
Theorem 6.1.9: Let 𝑓 and 𝑔 be functions defined on ℝ that are differentiable at 𝑐 ∈ ℝ. Then
𝑓(𝑥) , 𝑥 ∈ ℚ
𝛹(𝑥) = { }
𝑔(𝑥) , 𝑥 ∈ ℚ𝑐
is differentiable at 𝑐 with 𝛹 ′ (𝑐) = 𝐿 iff ⏟
𝑓(𝑐) = 𝑔(𝑐) and 𝑓 ′ (𝑐) = 𝑔′ (𝑐) = 𝐿.
𝛹 is continuous at 𝑐
Example 6.1.11: Find all values of 𝑥 at which the function 𝛹 is differentiability, where
𝑥 3 + 𝑥 , 𝑥 rational
𝛹(𝑥) = { }
−2𝑥 2 , 𝑥 irrational
Solution: Let 𝑓(𝑥) = 𝑥 3 + 𝑥 and 𝑔(𝑥) = −2𝑥 2 .
Step 1: We study the continuity of 𝛹:
𝑓(𝑥) = 𝑔(𝑥) ⇒ 𝑥 3 + 𝑥 = −2𝑥 2 ⇒ 𝑥 3 + 2𝑥 2 + 𝑥 = 0 ⇒ 𝑥(𝑥 2 + 2𝑥 + 1) = 0
⇒ 𝑥(𝑥 + 1)2 = 0 ⇒ 𝑥 = 0, −1 ⇒ 𝛹 is continuous only at 𝑥 = 0, −1.
Step 2: We study the differentiability at the points at which 𝛹 is continuous:
Now: 𝑓 ′ (𝑥) = 3𝑥 2 + 1 and 𝑔′ (𝑥) = −4𝑥:
At 𝑥 = 0: 𝑓 ′ (0) = 1 and 𝑔′ (0) = 0 ⇒ 𝑓 ′ (0) ≠ 𝑔′ (0)
⇒ 𝛹 is not differentiable at 𝑥 = 0.
At 𝑥 = −1: 𝑓 ′ (−1) = 4 and 𝑔′ (−1) = 4 ⇒ 𝑓 ′ (−1) = 𝑔′ (−1) = 4
⇒ 𝛹 is differentiable at 𝑥 = −1
⇒ 𝛹 is differentiable only at 𝑥 = −1 with 𝛹 ′ (−1) = 4.
123
Remark 6.2.2: Recall Bolzano’s Intermediate Value Theorem 5.3.13 which says the following:
If 𝑓: [𝑎, 𝑏] → ℝ is continuous on [𝑎, 𝑏] and 𝑓(𝑎)𝑓(𝑏) < 0, then ∃𝑐 ∈ (𝑎, 𝑏) such that 𝑓(𝑐) = 0
Example 6.2.3: Show that the equation 𝑥 5 + 𝑥 + 1 = 0 has exactly one real root in ℝ.
Solution: Let 𝑓(𝑥) = 𝑥 5 + 𝑥 + 1, 𝑥 ∈ ℝ
Step 1: First we show that the equation 𝑥 5 + 𝑥 + 1 = 0 has a real root in ℝ
⇒ 𝑓 is continuous on ℝ.
𝑓(−1)𝑓(1) = −1(1) < 0
Bolzano’s Intermediate Value Theorem ⇒ ∃𝑐 ∈ (−1,1) such that 𝑓(𝑐) = 0
𝑐 5 + 𝑐 + 1 = 0 ⇒ 𝑐 is a real root of the equation 𝑥 5 + 𝑥 + 1 = 0
⇒ the equation 𝑥 5 + 𝑥 + 1 = 0 has a real root in ℝ
Step 2: Second we show that the equation 𝑥 5 + 𝑥 + 1 = 0 has a unique real root
We use proof by contradiction:
Assume that there is another real root 𝑐1 of the equation 𝑥 5 + 𝑥 + 1 = 0 other than 𝑐.
⇒ 𝑓(𝑐) = 0 and 𝑓(𝑐1 ) = 0. Let 𝐼 be the interval with end points 𝑐 and 𝑐1.
Now,
(1) 𝑓 is continuous
(2) differentiable on 𝐼
(3) 𝑓(𝑐) = 0 and 𝑓(𝑐1 ) = 0
Roll’s Theorem 6.2.1⇒ ∃𝑐2 ∈ 𝐼 such that 𝑓 ′ (𝑐2 ) = 0
⇒ 5(𝑐2 )4 + 1 = 0 which is impossible
⇒ Our assumption is false ⇒ The equation has exactly one real root 𝑐.
Corollary 6.2.5: Let 𝑓: [𝑎, 𝑏] → ℝ be continuous on [𝑎, 𝑏] and differentiable on (𝑎, 𝑏).
(1) If 𝑓 ′ (𝑥) = 0, ∀𝑥 ∈ (𝑎, 𝑏), then 𝑓(𝑥) = 𝐶, ∀𝑥 ∈ [𝑎, 𝑏], where 𝐶 is a constant.
(2) If 𝑓 ′ (𝑥) ≥ 0, ∀𝑥 ∈ (𝑎, 𝑏), then 𝑓 is increasing on [𝑎, 𝑏].
(3) If 𝑓 ′ (𝑥) ≤ 0, ∀𝑥 ∈ (𝑎, 𝑏), then 𝑓 is decreasing on [𝑎, 𝑏].
Proof:
(1) Let 𝑥 ∈ (𝑎, 𝑏] and consider the function 𝑓 on the interval [𝑎, 𝑥]. Then
𝑓 is continuous on [𝑎, 𝑥] (since 𝑓 continuous on [𝑎, 𝑏] and [𝑎, 𝑥] ⊆ [𝑎, 𝑏])
𝑓 differentiable on (𝑎, 𝑏) (since 𝑓 differentiable on (𝑎, 𝑏) and (𝑎, 𝑥) ⊆ (𝑎, 𝑏))
The Mean Value Theorem ⇒ ∃𝑐 ∈ (𝑎, 𝑥) such that
𝑓(𝑥)−𝑓(𝑎)
𝑓 ′ (𝑐) = 𝑥−𝑎 ………①
Since 𝑓 ′ (𝑥) = 0, ∀𝑥 ∈ (𝑎, 𝑏) and 𝑐 ∈ (𝑎, 𝑏) ⇒ 𝑓 ′ (𝑐) = 0
𝑓(𝑥)−𝑓(𝑎)
The relation ① ⇒ 𝑥−𝑎 = 0 ⇒ 𝑓(𝑥) = 𝑓(𝑎), ∀𝑥 ∈ (𝑎, 𝑏].
Take 𝐶 = 𝑓(𝑎) ⇒ 𝑓(𝑥) = 𝐶, ∀𝑥 ∈ [𝑎, 𝑏].
(2) Let 𝑥1 , 𝑥2 ∈ [𝑎, 𝑏] with 𝑥1 < 𝑥2 . Then
𝑓 is continuous on [𝑥1 , 𝑥2 ]
differentiable on (𝑥1 , 𝑥2 )
The Mean Value Theorem ⇒ ∃𝑐 ∈ (𝑥1 , 𝑥2 ) ⊆ (𝑎, 𝑏) such that
𝑓(𝑥 )−𝑓(𝑥1 )
𝑓 ′ (𝑐) = 2 ……..②
𝑥2 −𝑥1
Since 𝑓 ′ (𝑥) ≥ 0, ∀𝑥 ∈ (𝑎, 𝑏) and 𝑐 ∈ (𝑎, 𝑏) ⇒ 𝑓 ′ (𝑐) ≥ 0
𝑓(𝑥 )−𝑓(𝑥 )
The relation ② ⇒ 𝑥2 −𝑥 1 ≥ 0 ⇒ 𝑓(𝑥2 ) ≥ 𝑓(𝑥1 ).
2 1
⇒ 𝑓 is increasing on [𝑎, 𝑏].
125
Relative Extrema Theorem 6.2.10: Let 𝑛 ∈ ℕ and 𝑓: [𝑎, 𝑏] → ℝ be such that 𝑓 ′ , 𝑓 ′′ , … , 𝑓 (𝑛) are
continuous on a neighborhood 𝑉𝛿 (𝑥0 ) = (𝑥0 − 𝛿, 𝑥0 + 𝛿) of some point 𝑥0 ∈ (𝑎, 𝑏) and
𝑓 ′ (𝑥0 ) = 0, 𝑓 ′′ (𝑥0 ) = 0, … , 𝑓 (𝑛−1) (𝑥0 ) = 0 and 𝑓 (𝑛) (𝑥0 ) ≠ 0.
(1) If 𝑛 is even and 𝑓 (𝑛) (𝑥0 ) > 0, then 𝑓 has a relative minimum at 𝑥0 .
(2) If 𝑛 is even and 𝑓 (𝑛) (𝑥0 ) < 0, then 𝑓 has a relative maximum at 𝑥0 .
(3) If 𝑛 is odd, then 𝑓 has neither a relative maximum nor a relative minimum at 𝑥0 .
126
Example 6.2.11: Determine whether the function has a relative maximum, a relative minimum, or
neither a relative extremum at 𝑥 = 0:
1
(1) 𝑓(𝑥) = sin2 (𝑥) + 3 𝑥 4 − 𝑥 2
𝑥3
(2) 𝑓(𝑥) = 6
− 𝑥 + sin(𝑥)
𝑥2
(3) 𝑓(𝑥) = 1 + 2
+ cos(𝑥)
Solution:
4 4
(1) 𝑓 ′ (𝑥) = 2sin𝑥 cos𝑥 + 3 𝑥 3 − 2𝑥 = sin(2𝑥) + 3 𝑥 3 − 2𝑥 ⇒ 𝑓 ′ (0) = 0
𝑓 ′′ (𝑥) = 2cos(2𝑥) + 4𝑥 2 − 2 ⇒ 𝑓 ′′ (0) = 0
𝑓 ′′′ (𝑥) = −4sin(2𝑥) + 8𝑥 ⇒ 𝑓 ′′′ (0) = 0
𝑓 (4) (𝑥) = −8cos(2𝑥) + 8 ⇒ 𝑓 (4) (0) = 0
𝑓 (5) (𝑥) = 16sin(2𝑥) ⇒ 𝑓 (5) (0) = 0
𝑓 (6) (𝑥) = 32cos(2𝑥) ⇒ 𝑓 (6) (0) = 32 ≠ 0
⇒ 𝑛 = 6 is even and 𝑓 (6) (0) > 0 ⇒ 𝑓 has a relative minimum at 0.
𝑥2
(2) 𝑓 ′ (𝑥) = 2 − 1 + cos(𝑥) ⇒ 𝑓 ′ (0) = 0
𝑓 ′′ (𝑥) = 𝑥 − sin(𝑥) ⇒ 𝑓 ′′ (0) = 0
𝑓 ′′′ (𝑥) = 1 − cos(𝑥) ⇒ 𝑓 ′′′ (0) = 0
𝑓 (4) (𝑥) = sin(𝑥) ⇒ 𝑓 (4) (0) = 0
𝑓 (5) (𝑥) = cos(𝑥) ⇒ 𝑓 (5) (0) = 1 ≠ 0
⇒ 𝑛 = 5 is odd ⇒ 𝑓 has neither a relative max. nor a relative min. at 0
(3) Is an exercise
127
𝑓(𝑥)
Example 6.3.2: Find lim 𝑔(𝑥). Justify your answer, where
𝑥→0
2
1
𝑓(𝑥) = 𝑥 sin ( ) for 𝑥 ≠ 0 and 𝑓(0) = 0 and let 𝑔(𝑥) = sin𝑥, 𝑥 ∈ ℝ
𝑥
1
𝑥 2 sin( ) 0
𝑥
Solution: Observe that lim+ =
𝑥→0 sin 𝑥 0
So, we think of using L’Hospital’s Rule (1) to find this limit
Since 𝑥 → 0 ⇒ Take the intervals (−1,0) ∪ (0,1)
1
Let 𝑓(𝑥) = 𝑥 2 sin (𝑥) and 𝑔(𝑥) = sin𝑥 are diff. on (−1,0) ∪ (0,1)
1
(1) 𝑓(𝑥) = 𝑥 2 sin (𝑥) and 𝑔(𝑥) = sin𝑥 are diff. on (−1,0) ∪ (0,1)
𝑑
(2) 𝑔′ (𝑥) = 𝑑𝑥 (sin𝑥) = cos𝑥 ≠ 0, ∀𝑥 ∈ (−1,0) ∪ (0,1)
1
(3) lim 𝑓(𝑥) = lim𝑥 2 sin (𝑥) = 0 (by the squeeze Theorem)
𝑥→0 𝑥→0
lim 𝑔(𝑥) = lim sin𝑥 = 0
𝑥→0 𝑥→0
1 −1 1 1 1
𝑓 ′ (𝑥) 𝑥 2 cos( )( 2 )+2𝑥sin( ) −cos( )+2𝑥sin( )
𝑥 𝑥 𝑥 𝑥 𝑥
(4) lim = lim = lim
𝑥→0 𝑔′ (𝑥) 𝑥→0 cos𝑥 𝑥→0 cos𝑥
1 1
−lim cos( )+2lim 𝑥sin( ) DNE+2(0)
𝑥→0 𝑥 𝑥→0 𝑥
= == = DNE ∉ ℝ ∪ {±∞}
lim cos𝑥 1
𝑥→0
⇒ L’Hospital’s Rule (1) fails to answer this question. We use another method:
1
𝑓(𝑥) 𝑥 2 sin( )
𝑥
lim = lim
𝑥→0 𝑔(𝑥) 𝑥→0 sin𝑥
𝑥 1
= lim (sin𝑥) (𝑥sin (𝑥))
𝑥→0
𝑥 1
= lim (sin𝑥) lim (𝑥sin (𝑥))
𝑥→0 𝑥→0
= 1(0) = 0
𝑓(𝑥)
⇒ lim 𝑔(𝑥) = 0
𝑥→0
128
ln(cos𝑥)
Example 6.3.3: Find lim+ . Justify your answer.
𝑥→0 𝑥
ln(cos𝑥) 0
Solution: Observe that lim+ =0
𝑥→0 𝑥
So, we think of using L’Hospital’s Rule (1) to find this limit
𝜋
Since 𝑥 → 0+ , take the interval (0, 4 )
𝜋
Let 𝑓(𝑥) = ln(cos(𝑥)), 𝑔(𝑥) = 𝑥, 𝑥 ∈ (0, 4 ):
𝜋
(1) 𝑓(𝑥) = ln(cos𝑥) and 𝑔(𝑥) = 𝑥 are diff. on (0, 4 )
𝜋
(2) 𝑔′ (𝑥) = 1 ≠ 0, ∀𝑥 ∈ (0, 4 )
(3) lim+𝑓(𝑥) = lim+ln(cos𝑥) = ln(1) = 0 and lim+ 𝑔(𝑥) = lim+ x = 0
𝑥→0 𝑥→0 𝑥→0 𝑥→0
sin(𝑥)
𝑓′ (𝑥) (− ) −sin(𝑥) 0
cos(𝑥)
(4) lim = lim+ = lim = 1 = 0 ∈ ℝ ∪ {±∞}
𝑥→0 𝑔′ (𝑥) 𝑥→0 1 𝑥→0+ cos(𝑥)
ln(cos𝑥) 𝑓(𝑥) 𝑓′ (𝑥)
By L’Hospital’s Rule (1) we have lim+ = lim+ 𝑔(𝑥) = lim =0
𝑥→0 𝑥 𝑥→0 𝑥→0 𝑔′ (𝑥)
L’Hospital’s Rule (2) 6.3.4: Let−∞ ≤ 𝑎 < 𝑏 ≤ ∞ and let 𝑓, 𝑔 be functions satisfying the following:
(1) 𝑓, 𝑔 are differentiable on (𝑎, 𝑏)
(2) 𝑔′ (𝑥) ≠ 0, ∀𝑥 ∈ (𝑎, 𝑏)
(3) lim+ 𝑔(𝑥) = ∞ or − ∞
𝑥→𝑎
𝑓 ′ (𝑥)
(4) lim+ 𝑔′ (𝑥) = 𝐿 ∈ ℝ ∪ {−∞, ∞}.
𝑥→𝑎
𝑓(𝑥) 𝑓 ′ (𝑥)
Then lim+ 𝑔(𝑥) = lim+ 𝑔′ (𝑥) = 𝐿
𝑥→𝑎 𝑥→𝑎
𝑥+sin𝑥
Example 6.3.5: Find lim . Justify your answer.
𝑥→∞ 𝑥−sin𝑥
𝑥+sin𝑥 ∗
Solution: Observe that lim =∞.
𝑥→∞ 𝑥−sin𝑥
ln(sin𝑥)
Example 6.3.6: Find lim+ . Justify your answer.
𝑥→0 ln𝑥
ln(sin𝑥) ∗
Solution: Observe that lim+ = −∞
𝑥→0 ln𝑥